Sei sulla pagina 1di 276

Safety, Risk and Reliability

[D21SR]

School of Energy, Geoscience, Infrastructure and Society

© Heriot-Watt University 2015


Address for correspondence:

School of the Built Environment

Heriot-Watt University,
Riccarton,
Edinburgh
EH14 4AS
UK
Tel.: +44 (0) 131-451-3153
Fax.: +44 (0) 131-451-4617

Module leader:

Prof. Dimitri Val

William Arrol Building,


School of the Built Environment
Heriot-Watt University,
Riccarton,
Edinburgh
EH14 4AS, UK
Tel.: +44 (0) 131-451-4622
Fax.: +44 (0) 131-451-4617
E-mail: D.Val@hw.ac.uk

Module Author:

Prof. Dimitri Val

School of Energy, Geoscience, Infrastructure and Society


Heriot-Watt University,

First published January 2010

Safety, Risk and Reliability, Heriot-Watt University 2


TABLE OF CONTENTS

STUDY GUIDE ............................................................................................................................................ 12

Overview and objectives ....................................................................................................................... 12

Materials ............................................................................................................................................... 12

VISION ................................................................................................................................................... 13

Study methods ...................................................................................................................................... 13

Communication with other students .................................................................................................... 13

Assessment ........................................................................................................................................... 14

Assistance and feedback ....................................................................................................................... 14

Further information .............................................................................................................................. 15

1 INTRODUCTION TO THE CONCEPTS OF RISK AND SAFETY IN STRUCTURAL ENGINEERING .................... 17

1.1 Introduction .................................................................................................................................... 17

1.2 Definition of Risk ............................................................................................................................. 17

1.3 Risk Management Process .............................................................................................................. 18

1.3.1 Establishing the context..................................................................................................... 18

1.3.2 Definition of the system .................................................................................................... 20

1.3.3 Identification of hazards and consequences ...................................................................... 20

1.3.4 Risk analysis ....................................................................................................................... 21

1.3.5 Risk evaluation................................................................................................................... 22

1.3.6 Consideration of risk treatment alternatives ..................................................................... 22

1.3.7 Implementation of risk treatment ..................................................................................... 24

1.3.8 Communication and consultation ...................................................................................... 24

Safety, Risk and Reliability, Heriot-Watt University 3


1.3.9 Monitoring and review ...................................................................................................... 24

1.4 Risk in society .................................................................................................................................. 25

1.5 Risk of structural failures................................................................................................................. 26

1.6 Examples of structural failures ........................................................................................................ 27

1.6.1 Failures due to human error .................................................................................................... 27

1.6.2 Failures due to natural hazards ................................................................................................ 29

1.6.3 Failures due man-made hazards .............................................................................................. 33

1.6.4 Failures due to deterioration ................................................................................................... 36

Revision Questions ................................................................................................................................ 37

SUMMARY............................................................................................................................................. 38

References ............................................................................................................................................ 38

2 BASICS OF PROBABILITY THEORY............................................................................................................ 39

2.1 Introduction .................................................................................................................................... 39

2.2 Elements of Set Theory ................................................................................................................... 39

2.3 Definition of Probability .................................................................................................................. 41

2.3.1 Classical definition ................................................................................................................... 41

2.3.2 Relative-frequency definition .................................................................................................. 41

2.3.3 Bayesian definition – probability as a measure of belief ......................................................... 42

2.3.4 Axiomatic definition ................................................................................................................. 42

2.4 Basic rules and theorems ................................................................................................................ 43

2.4.1 Union of not mutually exclusive events ................................................................................... 43

2.4.2 Multiplication rule and conditional probability ....................................................................... 44

2.4.3 Theorem of total probability .................................................................................................... 47

2.4.4 Bayes’ theorem ........................................................................................................................ 48

Exercises ............................................................................................................................................... 50

Revision Questions ................................................................................................................................ 52

SUMMARY............................................................................................................................................. 53

3 UNCERTAINTY MODELLING .................................................................................................................... 54

Safety, Risk and Reliability, Heriot-Watt University 4


3.1 Types of uncertainty in structural engineering ............................................................................... 54

3.2 Random variables ............................................................................................................................ 54

3.3 Probability distributions .................................................................................................................. 55

3.3.1 Discrete random variables ....................................................................................................... 55

3.3.2 Continuous random variables .................................................................................................. 56

3.4 Expected value ................................................................................................................................ 57

3.4.1 Expected value of a function .................................................................................................... 57

3.4.2 Moments of a random variable ............................................................................................... 57

3.4.3 Properties of the expectation operation.................................................................................. 59

3.5 Commonly used probability distributions ....................................................................................... 60

3.5.1 Discrete random variables ....................................................................................................... 60

3.5.1.1 Binomial distribution ........................................................................................................ 60

3.5.1.2 Poisson distribution .......................................................................................................... 61

3.5.2 Continuous random variables .................................................................................................. 63

3.5.2.1 Normal or Gaussian distribution ....................................................................................... 63

3.5.2.2 Lognormal distribution ..................................................................................................... 66

3.5.2.3 Uniform distribution ......................................................................................................... 68

3.5.2.4 Exponential distribution ................................................................................................... 68

3.5.2.5 Other probability distributions ......................................................................................... 69

3.6 Extreme value distributions ............................................................................................................ 70

3.6.1 Modelling extremes ................................................................................................................. 70

3.6.2 Gumbel distribution ................................................................................................................. 71

3.6.3 Weibull distribution ................................................................................................................. 73

3.7 Multiple random variables .............................................................................................................. 74

3.7.1 Joint probability distributions .................................................................................................. 74

3.7.2 Marginal distributions.............................................................................................................. 75

3.7.3 Conditional distributions.......................................................................................................... 76

3.7.4 Covariance and correlation ...................................................................................................... 76

Safety, Risk and Reliability, Heriot-Watt University 5


3.7.5 Common multivariate distribution........................................................................................... 79

3.8 Central limit theorem ...................................................................................................................... 79

3.9 Estimation of distribution parameters ............................................................................................ 80

Exercises ............................................................................................................................................... 81

Revision Questions ................................................................................................................................ 84

SUMMARY............................................................................................................................................. 84

4 FUNDAMENTALS OF STRUCTURAL RELIABILITY ...................................................................................... 85

4.1 Definition of failure and limit states ................................................................................................ 85

4.2 Deterministic and semi-probabilistic approaches to structural safety ............................................ 86

4.3 Probabilistic approach to structural safety ..................................................................................... 89

4.4 Special case: C and D are normal random variables ........................................................................ 90

4. Return period .................................................................................................................................... 94

Exercises ............................................................................................................................................... 94

Revision Questions ................................................................................................................................ 96

SUMMARY............................................................................................................................................. 96

5 FIRST ORDER METHODS OF RELIABILITY ANALYSIS ................................................................................ 97

5.1 First-Order Second-Moment (FOSM) method ................................................................................. 97

5.1.1 Introduction ............................................................................................................................. 97

5.1.2 The FOSM method ................................................................................................................... 97

5.1.3 The use of FOSM and the ‘lack of invariance’ problem .......................................................... 101

5.1.4 Designing to achieve a given reliability using FOSM .............................................................. 104

5.2 Hasofer-Lind safety index and AFOSM method............................................................................. 105

5.2.1 Hasofer-Lind safety index ...................................................................................................... 105

5.2.2 The AFOSM method ............................................................................................................... 112

5.2.3 Designing to achieve given reliability level using AFOSM method ......................................... 115

5.3 First-Order Reliability Method (FORM) ......................................................................................... 119

5.3.1 Equivalent normal distribution transformation ..................................................................... 120

5.3.2 Iterative procedure for FORM ................................................................................................ 122

Safety, Risk and Reliability, Heriot-Watt University 6


5.4 Sensitivity factors .......................................................................................................................... 126

5.5 Correlated random variables ........................................................................................................ 127

5.5.1 Correlated normal random variables ..................................................................................... 127

5.5.2 Correlated non-normal random variables ............................................................................. 129

Exercises ............................................................................................................................................. 130

Revision Questions .............................................................................................................................. 131

SUMMARY........................................................................................................................................... 132

References .......................................................................................................................................... 133

6 MONTE CARLO SIMULATION ................................................................................................................ 134

6.1 Introduction .................................................................................................................................. 134

6.2 Basic steps of Monte Carlo simulation .......................................................................................... 135

6.3 Advantages and disadvantages of Monte Carlo simulation .......................................................... 136

6.4 Accuracy of simulation .................................................................................................................. 137

6.5 Generation of random numbers uniform distributed on [0,1) ...................................................... 138

6.6 Generating other random numbers .............................................................................................. 140

6.6.1 The inverse transform method .............................................................................................. 140

6.6.2 Generating normal and lognormal random variables ............................................................ 141

6.7 Variance reduction techniques - importance sampling ................................................................. 146

6.8 Handling correlated random variables .......................................................................................... 150

Exercises ............................................................................................................................................. 150

Revision Questions .............................................................................................................................. 150

SUMMARY........................................................................................................................................... 150

References .......................................................................................................................................... 151

7 LOAD AND RESISTANCE MODELLING.................................................................................................... 152

7.1 Introduction .................................................................................................................................. 152

7.2 Load modelling .............................................................................................................................. 152

7.2.1 Introduction ........................................................................................................................... 152

7.2.2 Dead (or permanent) loads .................................................................................................... 153

Safety, Risk and Reliability, Heriot-Watt University 7


7.2.3 Live loads in buildings ............................................................................................................ 153

7.2.3.1 Sustained loads ............................................................................................................... 154

7.2.3.2 Transient loads ............................................................................................................... 156

7.2.4 Wind loads ............................................................................................................................. 158

7.2.5 Wave loads ............................................................................................................................ 159

7.2.6 Extreme values of repeated loads.......................................................................................... 164

7.3 Resistance modelling .................................................................................................................... 167

7.3.1 Introduction ........................................................................................................................... 167

7.3.2 Structural steel....................................................................................................................... 167

7.3.3 Reinforcing steel .................................................................................................................... 169

7.3.4 Concrete ................................................................................................................................ 169

7.3.5 Use of extreme value distributions for resistance modelling................................................. 170

Exercises ............................................................................................................................................. 172

Revision Questions .............................................................................................................................. 173

SUMMARY........................................................................................................................................... 173

References .......................................................................................................................................... 174

8 RELIABILITY ANALYSIS OF STRUCTURAL SYSTEMS ................................................................................ 175

8.1 Introduction .................................................................................................................................. 175

8.2 Load combinations ........................................................................................................................ 175

8.3 System reliability ........................................................................................................................... 178

8.3.1 Introduction ........................................................................................................................... 178

8.3.2 Series systems ........................................................................................................................ 180

8.3.3 Parallel systems ..................................................................................................................... 182

8.3.3.1 Parallel system with perfectly ductile components ........................................................ 183

8.3.3.2 Parallel system with brittle components ........................................................................ 184

8.3.4 Hybrid (combined) systems ................................................................................................... 185

8.3.5 System reliability bounds ....................................................................................................... 188

8.3.5.1 Series system with positive correlation .......................................................................... 188

Safety, Risk and Reliability, Heriot-Watt University 8


8.3.5.2 Parallel system of ductile components with positive correlation ................................... 188

Exercises ............................................................................................................................................. 189

Revision Questions .............................................................................................................................. 190

SUMMARY........................................................................................................................................... 190

References .......................................................................................................................................... 191

9 RELIABILITY-BASED CODE CALIBRATION ............................................................................................... 192

9.1 Introduction .................................................................................................................................. 192

9.2 Levels of reliability-based design .................................................................................................. 192

9.3 Basic elements of code development ........................................................................................... 193

9.3.1 Establishing scope of the code ............................................................................................... 193

9.3.2 Selection of calibration points ............................................................................................... 194

9.3.3 Establishing target safety levels ............................................................................................. 194

9.3.4 Code formats ......................................................................................................................... 196

9.4 Derivation of partial factors by FOSM/FORM................................................................................ 197

9.5 Code calibration procedure........................................................................................................... 198

9.6 Example of code calibration (adapted from Melchers (1999)) ...................................................... 200

Revision Questions .............................................................................................................................. 205

SUMMARY........................................................................................................................................... 205

References .......................................................................................................................................... 206

10 RELIABILITY-BASED ASSESSMENT OF EXISTING STRUCTURES............................................................. 207

10.1 Introduction ................................................................................................................................ 207

10.2 Dealing with structural deterioration ...................................................................................... 208

10.3 Corrosion modelling .................................................................................................................... 209

10.3.1 Types of corrosion ............................................................................................................... 209

10.3.2 Corrosion and minimum remaining strength ....................................................................... 210

10.4 Fracture ....................................................................................................................................... 216

10.4.1 Types of fracture mechanism............................................................................................... 216

10.4.2 Notches and stress concentrations, cracks and the stress intensity factor .......................... 216

Safety, Risk and Reliability, Heriot-Watt University 9


10.4.3 Toughness and brittle or ductile fracture ............................................................................ 218

10.5 Fatigue ........................................................................................................................................ 220

10.5.1 Introduction ......................................................................................................................... 220

10.5.2 S-N curves ............................................................................................................................ 221

10.5.3 Stress ranges and the use of Miner’s rule ............................................................................ 222

10.5.4 Factors affecting fatigue life ................................................................................................ 224

10.5.5 Fracture mechanics and crack growth ................................................................................. 225

10.6 Modelling structural deterioration and fatigue cracking ............................................................ 227

10.7 Structural inspection ............................................................................................................... 229

10.8 Updating in light of inspection................................................................................................. 231

Exercises ............................................................................................................................................. 236

Revision Questions .............................................................................................................................. 236

SUMMARY........................................................................................................................................... 237

References .......................................................................................................................................... 238

APPENDIX 1 – BASICS OF STRENGTH OF MATERIALS .............................................................................. 239

A1.1 Introduction............................................................................................................................. 239

A1.2 Elastic Theory .......................................................................................................................... 239

A1.3 Theory of Thin Shells ............................................................................................................... 239

General points ............................................................................................................................ 243

Exercises ..................................................................................................................................... 243

A1.4 Beam theory – elastic bending of beams ................................................................................. 243

Exercises ..................................................................................................................................... 246

A1.5 Plastic theory for beams .......................................................................................................... 246

A1.5.1 Collapse mechanisms....................................................................................................... 248

Exercises ..................................................................................................................................... 250

A1.6 Buckling ................................................................................................................................... 250

A1.6.1 Elastic buckling of columns .................................................................................................. 250

Exercises ..................................................................................................................................... 253

Safety, Risk and Reliability, Heriot-Watt University 10


A1.6.2 Plastic buckling of columns.................................................................................................. 253

Exercises ..................................................................................................................................... 256

Revision Questions .............................................................................................................................. 256

APPENDIX 2 - Numerical values of the standard normal CDF, (u). ....................................................... 258

APPENDIX 3 – ANSWERS TO EXERCISES FROM APPENDIX 1 .................................................................... 260

APPENDIX 4 – INFORMATION SHEET ....................................................................................................... 266

Basic Mechanics of Materials ......................................................................................................... 266

Geometrical Formulae .................................................................................................................... 266

Probability Density Functions ......................................................................................................... 267

Reliability and Simulation ............................................................................................................... 268

APPENDIX 5 – PAST EXAM PAPERS .......................................................................................................... 269

Safety, Risk and Reliability, Heriot-Watt University 11


STUDY GUIDE
Overview and objectives
The desire to ensure that structures are safe and will not collapse even under the most extreme
loading has always been with us. The course starts by introducing you to the concept of risk in
engineering design in general and in structural engineering in particular. The basics of probability theory
needed for the rest of the course are then introduced. The main concepts used in structural reliability
including capacity, demand, limit state functions, the safety index and probabilities of failure are
explained before going on to look at the ways in which structural reliability may be estimated. You will
acquire skills in estimating structural reliability first through hand calculations, before moving onto the
use of Excel spreadsheets for iterative calculations and computer programs for Monte Carlo simulations.
You will look at the uncertainties in the strength of structures and loading to which they are subjected
and how these are modelled. At the design stage lifetime extremes of loading and strength often need
to be estimated and you will learn how to do this through the use of extreme value statistics. Many
structures deteriorate significantly with time and it is necessary to ensure their adequate safety over the
whole lifetime. You will be introduced to risk-based inspection strategies and learn how to update
structural models in the light of inspection results. Generally, structures are designed to Codes of
Practice and you will understand how these are developed and calibrated.

Materials
The materials provided with the Safety, Risk and Reliability course should be sufficient to complete
this course. If your background in mechanics of materials is limited then you need to consult some
introductory texts in this area to supplement the material provided in the Appendix 1. In addition to the
text you will receive an Excel spreadsheet pro-forma for doing iterative calculations for structural
reliability that may be adapted for the coursework exercises. You will also be given a simple Monte Carlo
simulation program in Visual Basic that runs in Excel. Again this can be adapted to undertake
coursework exercises.

All text-based materials produced by Heriot-Watt will be available as colour pdf files downloadable
from the VISION (see below). These will be useful if you need to find words or phrases via the search
function. Details of the VISION will be provided when you register for the course. Should you spot any
errors in the course please let us know. Any changes to the course materials will be posted online.

Additional resources and links may also be provided online through VISION or can be accessed via the
library. The library does not offer a postal service to distance learners but there are many resources

Safety, Risk and Reliability, Heriot-Watt University


Study Guide
12
available electronically. As a Heriot-Watt University student you will have free access to electronic
materials, which are accessible via the Heriot-Watt library website (you may need your normal VISION
login and password).

VISION
The University’s virtual learning environment VISION can be found on https://vision.hw.ac.uk/. You
should be able to login in to this site with your HW e-mail username and password. If you know that
material is available on VISION for courses you are studying but still cannot see them then you need to
check with your programme leader or the Student Office that your course registration is correct.

Distance learning students should have their accounts already activated and their login details and
computing information forwarded to them by our Student Office when they first begin the course. If this
is your first course on the programme you will need to work this out to log in. If you have already used it
before and your registration details are correct on our database then you should see this course
automatically.

On-campus students will need to follow the computing guidelines provided at the induction session
to set up their HW e-mail accounts before they can access VISION.

Study methods
We recommend that you work through the text and follow all the worked examples and exercises. At
the end of sections there is a selection of additional exercises and revision questions that should help
you check your understanding of the materials. Each section also has a summary that should help
remind you of the content of that section but these summaries do not include all the material you will
need to know.

Communication with other students


Collaboration and discussion with your student peers will help your progress. Therefore we strongly
encourage you to communicate with your fellow students. With this in mind we aim to allow you access
to talk to each other on-line through VISION. There are several means of communication available.

In the first place we suggest you use the Discussion Boards to compare notes and help each other on
the topics in this course. It may take a while for any discussions to get going. If you have a more urgent
problem contact the course leader by e-mail. If the answer is of use to all students we will post it on the
Discussion Boards.

Safety, Risk and Reliability, Heriot-Watt University 13


Study Guide
You will also notice that there is a ‘Send Email’ function. This tool can be used to communicate
directly with individuals at their HW e-mail addresses but feel free to release further contact
information so that you can, if you wish, communicate privately in small groups. Heriot-Watt University
cannot release any personal data about you to others without your permission. However, please do not
use the send e-mail to write to all students. Our database of students is not always up to date and there
may be some students listed that are not currently doing the course. So only use this for personal
communication when you know who you need to talk to.

Assessment
The course assessment includes coursework. On-campus students should submit it in accordance to
the deadline set by the course leader in the course schedule. DL students should submit the
coursework in accordance to the deadline given in their Handbook. Solution of the coursework
questions involves hand calculations and may also involve adapting and using Excel spreadsheets and
Monte Carlo simulation program for structural reliability calculations. Full solutions of the coursework
questions with detailed explanations should be provided including, where it is relevant, print outs of
Excel spreadsheets. Please note that it is not necessary to type out the hand calculations and a clear
hand-written report is quite acceptable. DL students should also include into their submissions a CD-
ROM with Excel files of the solution spreadsheets. DL students cannot submit their coursework
electronically on-line; it should be sent either by e-mail to the course leader or by mail to the Student
Office.

Formal feedback will be provided when the submitted coursework solutions are marked. However,
do feel free to contact the course leader at any time if you have any problems. We also recommend that
you communicate with other students and work in small groups as this will help you to assess your own
work.

The course is also assessed by means of a two hour exam at the end of the year in addition to the
coursework submitted. Examples of exam papers are provided in the appendices and also available from
the School Website. The worked examples in the text are similar to exam questions.

Assistance and feedback


The course leader, Prof. Dimitri Val, is always willing to help with any problems you may have and
provide feedback on your progress where appropriate. The best method of contacting him is by e-mail.
Note also that all communication from the course leader will be through your HW e-mail account as
most HW staff do not have any access to your alternative e-mail addresses. Alternatively, you may try
the contact details listed inside the front cover. He should usually respond within a few days but

Safety, Risk and Reliability, Heriot-Watt University 14


Study Guide
occasionally this may be longer if he is away. If you do not get a reply, please try again in case your
message did not get through. You may also contact the School Office or Programme Leader if you have
any difficulties. Any questions raised and responses provided that might be of wider interest will be
circulated to all studying the course. Please note, rather than type in an analytical query it is probably
easier to write it by hand and then scan it and send it as an attached file.

For any difficulties you have with the administration or running of the course please contact the
School Office or your Programme Leader.

Further information
If you have limited background knowledge of mechanics of materials the following textbook, or
something similar, may be very helpful.

Benham, PP, RJ Crawford, CG Armstrong (1996) “Mechanics of Engineering Materials”, 2nd Edition, Longman. ISBN
0-582-25164-8

If you want to explore the course subject further, the following books should be useful. Most of these
books are all available from Heriot-Watt library. Thoft-Christensen and Baker (1982) and Melchers
(1999) are probably the most relevant and easily understood.

Ang, AHS, WH Tang (1984) Probability Concepts in Engineering Planning and Design. Volume I. John Wiley and
Sons. ISBN 0-471-03200-X. –Volume 1 is a good text on applied probability and statistics. Volume II is out of print.

Blockley, D (1992) “Engineering Safety”, McGraw Hill – covers many of the broader issues, mainly discursive – little
maths.

Haldar, A, S Mahadevan (2000) “Probability, Reliability, and Statistical Methods in Engineering Design”, John Wiley
& Sons – a good introductory text covering most of the module

Lind, NC (1969) Deterministic formats for the probabilistic design of structures, An Introduction to Structural
optimisation, SM Study No. 1, Kachaturian, N. (ed), University of Waterloo, Ontario, pp121-142.

Madsen HO, S Krenk, NC Lind (1986) “Methods of Structural Safety” More advanced text – mathematically quite
intense

Melchers, RE (1999) “Structural Reliability, Analysis and Prediction”, 2nd edition Wiley, Chichester – an
introductory text covering much of the module and a lot more besides

Nowak, AS, KR Collins (2000) “Reliability of Structures”, McGraw-Hill – a basic textbook on structural reliability

Thoft-Christensen P, MJ Baker (1982) “Structural Reliability Theory and its Applications”, Springer Verlag – an older
text – covers all the basic material

Additional references are provided at the end of each chapter. Note that you are not required to
read either the above references or the references appearing further in the text.

Sections of the library of interest include:

Safety, Risk and Reliability, Heriot-Watt University 15


Study Guide
660.2804 SAFETY TECHNOLOGY (Part of CHEMICAL ENG = 660)

693.82 BUILDING

624.17 STRUCTURAL THEORY (part of CIVIL ENGINEERING = 624)

627.9 OFFSHORE (Includes Piper Alpha Inquiry Report)

519 PROBABILITY & STATISTICS

363.12 PUBLIC SAFETY (Includes inquiry reports etc., other than for Piper Alpha)

614 HEALTH AND SAFETY; GENERAL

The library also carries the journal “Structural Safety” (available on-line) which you may wish to look
at.

The library does not offer a postal service to distance learners but there are many resources available
electronically. Should you spot any errors in the course please let us know. Any changes to the course
materials will be posted on the course website.

Safety, Risk and Reliability, Heriot-Watt University 16


Study Guide
1 INTRODUCTION TO THE CONCEPTS OF RISK AND SAFETY
IN STRUCTURAL ENGINEERING

1.1 Introduction
One of the main responsibilities of a civil/structural engineer is to provide safety of civil engineering
facilities (e.g., buildings, bridges, power plants, dams, offshore structures, etc.) over their entire life. In
other words, to prevent failures of these facilities which can be a result of numerous causes such as
natural and human-made hazards, human errors in design, construction and maintenance, deterioration
of the facilities under service conditions, etc. The safety provision is essential for

 preventing human injury, including loss of life;


 protecting environment; and
 ensuring economic feasibility.

In a wider sense, the safety provision is being dealt with within the process of risk management. This
process is based on recognition of the fact that it is impossible to guarantee perfect safety of an
engineering facility – constructing, operating and then decommissioning a new engineering facility (as
any other activities) are always associated with risk, i.e., probability that something will go wrong
(failure) which will lead to harmful consequences (e.g., injuries, loss of life, economic losses,
environmental damage). Usually, there are numerous uncertainties affecting failure of an engineering
facility (system) and consequences of this failure, e.g., uncertainties associated with material properties,
loads acting on the facility, environmental conditions, etc. Thus, risk management is basically a process
of decision making under uncertainty.

In the following, a formal definition of the term “risk” will be provided. The process of risk
management will be considered in more detail and its main elements will be explained. Statistical
information on risks to human life posed by various activities will be provided. Finally, causes and
examples of failures of civil engineering facilities will be considered.

1.2 Definition of Risk


In common language risk may mean a source of danger (or harm), the possibility of suffering harm
(or loss) and is often used interchangeably with such words as chance, danger, probability, or hazard. As
a special term it has different definitions depending on the area of its application (e.g., engineering,
finance, auditing, etc.). In engineering risk is defined quantitatively as

Safety, Risk and Reliability, Heriot-Watt University 17


1. Introduction to the Concept of Risk and Safety
Risk = (Probability of event/failure) × (Consequences of the event) (1.1)

Consequences of the event/failure are usually expressed in monetary terms. The definition means
that risk represents the expected consequences of a certain event/failure.

1.3 Risk Management Process


The process of risk management typically consists of (1) establishing the context, (2) risk assessment,
(3) risk treatment, (4) communication and consultation, and (5) monitoring and review, as illustrated in
Figure 1.1 (AS/NZS 4360 1999). It should be noted that risk management is not a one way process but
an iterative process.

1.3.1 Establishing the context

At this stage the basic parameters within which risks must be managed should be defined and
guidance for decisions within more detailed risk management studies should be provided. This may be
considered as the most important stage of the risk management process since it sets the scope for the
rest of it. This stage consists of:

 Definition of the role of risk assessment in the framework of the risk management process.
Typically, this may be performed for (i) new design, (ii) assessment of existing
system/structure, (iii) assessment of exceptional structures and/or typical structures
subjected to extraordinary events, and (iv) risk-based decision making.

 Identification of the internal and external stakeholders, and consideration of their objectives,
by taking into account their perceptions and establishing communication policies with these
parties.

 Identification of factors which may have negative influence on the risk management process.

 Definition of the extent and comprehensiveness of the risk management activities to be


carried out. This should be done with full consideration of the need to balance costs, benefits
and opportunities.

Safety, Risk and Reliability, Heriot-Watt University 18


1. Introduction to the Concept of Risk and Safety
1) Establish the context

2) Risk assessment

Definition of the system

Identification of hazards and


consequences
4) Communication and consultation

5) Monitoring and review


Estimation of probability
Risk analysis
Estimation of consequences

Risk evaluation No
Should risk be treated?

Yes

Consideration of risk treatment alternatives

3) Implementation of risk
treatment

- Scope of risk assessment

Figure 1.1 Flow of generic risk management process.

 Determination of the criteria against which risk is to be evaluated. Decisions concerning risk
acceptability and risk treatment may be based on operational, technical, financial, legal,
social, humanitarian or other criteria. These often depend on intended functions of the
engineering system and the interests of stakeholders. Criteria may be affected by internal and
external perceptions and legal requirements. Although risk criteria are initially developed as
part of establishing the risk management context, they may be further developed and refined
subsequently as particular risks are identified and risk analysis techniques are chosen, i.e. the

Safety, Risk and Reliability, Heriot-Watt University 19


1. Introduction to the Concept of Risk and Safety
risk criteria must correspond to the type of risks and the way in which risk levels are
expressed.

1.3.2 Definition of the system

At this stage the boundaries of the engineering system, to which the risk management process is
being applied, should be defined, i.e., it should be clearly described what parts/elements belong to the
considered system and what parts/elements are outside of it. The system should be fully described
including types of its structure(s), codes and standards used in its design (if such information is
available), its use, importance, location and working life. The limit states of the system should also be
specified. Justification of the system definition should be provided. The system definition will affect the
level of detail in the risk analysis and this aspect should also be addressed at this stage.

1.3.3 Identification of hazards and consequences

At this stage possible failure modes of the system (and associated with them consequences) and
possible causes of the failures (hazards) should be identified. For a potential hazard, scenarios should be
identified as a sequence or combination of events or processes leading to failure of the system. A
scenario may include collapse of the whole system, failure or damage of its subsystems/components,
loss of the system functionality, loss of human life or injury, and other economic and/or social losses. In
order to perform this task it may be necessary to divide the system into subsystems/components.

Consequences resulting from the hazards and following events should be identified. They may be
described in terms of several measures, e.g., monetary loss, human fatalities and environmental
damage. Some consequences can be identified by scenario analyses considering the extent of the
influences due to failure of the system in time and space.

Although all possible hazards should be taken into consideration, hazards important to the system
should be selected on the basis of their significance and incorporated in the risk assessment. The criteria
for the hazard screening are, in principle, based on the magnitude of the risk from the preliminary risk
estimation. Frequency of the hazard and/or significance of the relevant consequences are also useful
criteria. Hazards with obviously negligible risk compared with the acceptable risk level can be screened
out. The hazard screening criteria should be clearly described in terms of frequency of the event and
magnitude of its consequence. They may be based on the past experience, human perception, and
relevant values specified elsewhere.

Safety, Risk and Reliability, Heriot-Watt University 20


1. Introduction to the Concept of Risk and Safety
1.3.4 Risk analysis

Risk analysis depends on its purpose, required degree of details, and data and resources available.
The types of risk analysis can broadly be divided into the three categories: qualitative, semi-quantitative
and quantitative.

In qualitative analysis, risk is subjectively estimated and ranked in a descriptive manner. Qualitative
estimation can be used: (i) as an initial screening activity to identify risks which require more detailed
estimation; (ii) when it provides sufficient information for decision making; or 3) where available data or
resources are insufficient for a quantitative estimation. As a result of such analysis, risk can be rated, for
example, as high, moderate, or low.

In semi-quantitative analysis, a more expanded ranking scale than the one usually employed in
qualitative estimation is adopted. It should be noted that values used in such analysis are often chosen
subjectively and, therefore, may not properly reflect relativities, which can lead to inconsistent or even
incorrect outcomes.

In quantitative analysis, the evaluation of both consequences and probability is based on data from a
variety of sources:
 Past records
 Collected field data
 Experimental data (including prototype testing)
 Relevant published data
 Data obtained using analytical/numerical models, and
 Expert opinions.

The quality of the estimation depends on the accuracy and completeness of the data and the validity
of the models used. A sensitivity analysis should be carried out to investigate the effect of uncertainty in
assumptions, models, and data on results of the estimation. A high sensitivity suggests that more care
and/or effort are required in obtaining relevant data or more accurate models need to be employed.
Sensitivity analysis is also a way of examining the appropriateness and effectiveness of potential
controls and risk treatment options.

Estimation of probability

The quantitative estimation of the probability of failure of an engineering system (or its
subsystems/components) depends on the type of subsystem/components and available information
about their failures. Two approaches are possible: (i) the system reliability approach in which the
probability of failure an engineering system is estimated using data on failure rates of the system
Safety, Risk and Reliability, Heriot-Watt University 21
1. Introduction to the Concept of Risk and Safety
subsystems/components (the data are often available for electronic and mechanical parts); (ii) the
structural reliability approach in which the probability of failure is evaluated by a direct probabilistic
analysis of the system (or its subsystem/component) (this is the main topic of this module and will be
considered in detail in later chapters).

Estimation of consequences

The consequences to be estimated should be consistent with the ones considered in the acceptance
criteria. Typically, they should include loss of lives and injuries, economic consequences and
environmental effects. Consequences can be determined by modelling the outcomes of a hazard or a set
of events, or by judging from experimental studies or past observations. The estimation of
consequences requires thorough understanding of the considered system and its interaction with
environment and, therefore, is better performed in collaboration with experts, who have hands-on
experience in this type of activity.

1.3.5 Risk evaluation

After the risk is estimated, it should be determined whether the risk level is acceptable or not by
comparing it with the acceptance criteria determined at the stage of establishing the context. If the risk
is unacceptable, it shall be treated appropriately.

1.3.6 Consideration of risk treatment alternatives

There are four approaches to risk treatment:

Avoidance – aimed at avoiding the risk by not undertaking or discontinuing activity that may
generate it (i.e., preventing occurrence of hazard). In order to use this approach, particular hazards need
to be identified. Measures associated with this approach may include changes in the structure site or
access to it (e.g., not building near a seismic fault or imposing a minimum stand-off distance by placing
barriers or other similar devices), by preventing the use or storage of hazardous substances within the
structure (e.g., not using natural gas). Although such measures are often the most simple ones and
usually do not require special structural engineers' services it is important to stress that avoiding the risk
also prevents achieving benefits associated with it.

Reduction – aimed at reducing the likelihood/probability of the hazard and/or its consequences if it
occurs. Reducing the likelihood of hazard will not eliminate the risk if it is inherent to a particular
process associated with the engineering system use.

Safety, Risk and Reliability, Heriot-Watt University 22


1. Introduction to the Concept of Risk and Safety
Measures aimed at reducing the likelihood of a hazard may include:
 personnel training
 review of design specifications and requirements
 quality control at the stages of design and construction
 regular inspection during the system service life
 control for processes associated with the system use
 preventative maintenance (e.g., repair, replacement of damaged components)
 strengthening and retrofit of existing structures
 structural protective measure (e.g., protection of columns)
 non-structural protective measures (e.g., installing a fire sprinkler system)
 improving techniques for design, construction, and maintenance through research and
development.

Measures aimed at reducing consequences may include:


 provision of evacuation routes
 training building occupants how to behave in emergency situations
 response planning
 confinement of the hazard (e.g. prevention of fire spread by compartmentalisation)
 limiting the extent of failure (e.g. by improving robustness of the system).

Transfer – aimed at passing some of the risk to other parties. It can be achieved via insurance or
finding a partner(s) to share the burden of the risk. The most basic risk-transfer instrument is insurance,
where the policyholder pays a small premium in return for claim payments from the insurer should he or
she suffer losses from a disaster.

Retention – to accept the risk and be ready to deal with its consequences if it occurs. Plans need to
be prepared for dealing with the risk consequences, including identifying possible sources for covering
losses. All risks that are not avoided or transferred are retained by default. Risk retention may be a
viable option for small risks where the cost of insuring against the risk would be greater over time than
the total losses sustained (this can be assessed by cost-benefit analysis). This also includes risks that are
so large or catastrophic that they either cannot be insured against or the premiums would be infeasible.
War is an example since most property and risks are not insured against it. Any amount of potential loss
(consequences) over the amount insured is related to the retained risk.

The approaches are not mutually exclusive and in most cases their combination will provide the most
efficient solution. It is important to stress that measures for the risk treatment should be addressed as
part of the initial risk assessment during the planning, design and commissioning stages. Many of risk

Safety, Risk and Reliability, Heriot-Watt University 23


1. Introduction to the Concept of Risk and Safety
treatment measures may be impossible or costly to implement once the structure has been
commissioned. It should be checked that measures undertaken against certain risks would not
inadvertently increase others.

1.3.7 Implementation of risk treatment

After the consideration of alternatives for risk treatment, the most appropriate ones should be
selected and implemented. Since new risks could be introduced by the risk treatment, they should be
identified, assessed, treated and monitored. After treatment, a decision should be made on whether to
retain the residual risk or repeat the risk treatment process.

1.3.8 Communication and consultation

Communication and consultation are important considerations at each step of the risk management
process. It is important to develop a communication plan for both internal and external stakeholders at
the earliest stage of the process. This plan should address issues relating to both the risk itself and the
process to manage it. Effective internal and external communication is important to ensure that those
responsible for implementing risk management, and those with a vested interest understand the basis
on which decisions are made and why particular actions are required. Perceptions of risk can vary due to
differences in assumptions, concepts, needs, issues and concerns of stakeholders, who are likely to
make judgments of the acceptability of a risk based on their own perception of the latter. Since
stakeholders can have a significant impact on the decisions made, it is important that their perceptions
of risk, as well as their perceptions of benefits, be identified and documented and the underlying
reasons for them understood and addressed.

1.3.9 Monitoring and review

Risks and the effectiveness of control measures need to be monitored to ensure changing
circumstances do not alter risk priorities since few risks remain static. For many engineering systems
such as offshore oil platforms, large construction projects, bridges, etc. monitoring is an essential part of
the risk management process, which provides new data (including data about effects of the risk
treatment options implemented during the service life of the system) for updating the risk analysis.

On-going review is essential to ensure that the management plan remains relevant. Factors which
may affect the likelihood and consequences of a hazard may change, as the factors which affect the

Safety, Risk and Reliability, Heriot-Watt University 24


1. Introduction to the Concept of Risk and Safety
suitability or cost of the various treatment options. It is therefore necessary to regularly repeat the risk
management cycle. Review is an integral part of the risk management process.

1.4 Risk in society


One of the ways to set acceptable levels of risk for activities associated with construction, operation,
and decommission of engineering systems is by comparing this risk with other risks in society, whose
estimates are shown in Table 1.1 with consequences expressed in terms of loss of human lives
(Melchers 1999).

Table 1.1 Estimates of risk associated with various activities (adapted from Melchers 1999)
Activity Approximate death rate Typical exposure1 Expected risk of
(×10-9 deaths/hour (hours/year) death
exposure) (×10-6 /year)
Alpine climbing 30000-40000 50 1500-2000
Boating 1500 80 120
Swimming 3500 50 170
Cigarette smoking 2500 400 1000
Air travel 1200 20 24
Car travel 700 300 200
Train travel 80 200 15
Coal mining 210 1500 300
Construction work 70-200 2200 150-440
Manufacturing 20 2000 40
Building fires2 1-3 8000 8-24
2
Structural failures 0.02 6000 0.1
1
For those involved in the activity (estimated)
2
Exposure for an average person (estimated)

As can be seen, the risk due structural failures is much smaller compared with the other activities. It
is also necessary to note that there should be a difference between the acceptable level of risk for
“voluntary” and “involuntary” activities. The use of engineering structures is usually considered as an
“involuntary” activity and as such the acceptable level of risk associated with it should be much lower
(by at least one order of magnitude or more) than that associated with “voluntary” activities such as
alpine climbing, cigarette smoking, or construction work.

Additional information about acceptable levels of risk is also given in Table 1.2 (Otway et al. 1970).

Safety, Risk and Reliability, Heriot-Watt University 25


1. Introduction to the Concept of Risk and Safety
Table 1.2 Indicative estimates of acceptable risk (adapted from Otway et al. 1970)
Risk of death per Characteristic response
person per year
10-3 Uncommon accidents; immediate action is taken to reduce the risk
10-4 People spend money, especially public money to control the risk
(e.g., traffic signs, police, laws)
10-5 Mothers warn their children of the risk (e.g., fire, drowning,
firearms, poisons), also air travel avoidance
10-6 Not of great concern to average person; aware of the risk but not
of personal nature; act of God

1.5 Risk of structural failures


Based on a total of about 700 reported structural failures and errors Matousek and Schneider (1976)
have reported a detailed review of their causes. Figure 1.2 shows at what stage of the project the
failures and errors were discovered for different types of structures.

As can be seen from Figure 1.2, on average failures and errors were distributed equally between the
construction and usage stages. However, there are some differences in the distribution for the different
types of structures. Industrial structures suffer mostly from failures and errors during the operation,
whereas the dam structures experience the largest part of failures and errors at the stage of
construction. This can be explained by differences in the operational use of these types of structures.

Figure 1.3 provides information about various causes of structural failures (Stewart and Melchers
1997). As can be seen, the main causes of structural failures are poor construction procedures,
inadequate connection between elements, and inadequate load behaviour.

100%
90%
80%
70% Retrofit/disposal
60% Usage
50%
Construction
40%
30%
20%
10%
0%
)

1)
5)

2)

)
62

52

92
(5
(7

(5
(3

(1

(6
s
es

es
gs

m
gs

es
ur

ur
Da
in

in

ur
ct

ct
ild

ld

ct
ru

tr u
ui
bu

tr u
st
lb

ls
ad
e

er
ria
ris

Al
th
Ro
st
h-

O
du
g
Hi

In

Figure 1.2 Failures and errors discovered in different types of structures, adapted from Matousek and
Schneider (1976).

Safety, Risk and Reliability, Heriot-Watt University 26


1. Introduction to the Concept of Risk and Safety
Poor construction procedures 54.3
Inadequate connection of elements 47
Inadequate load behaviour 42.2
Unclear contract information 23.5
Contravention of instructions 21.9
Unforeseeable events 7.1
Errors in design calculations 2.5
Reliance on construction accuracy 1.8
Complexity of project system 1.2

0 10 20 30 40 50 60
Frequency (%)

Figure 1.3 Causes of structural failures, adapted from Stewart and Melchers (1997).

1.6 Examples of structural failures


There are a number of ways to classify structural failures. For example, they may be classified by the
behaviour of a structure at failure - brittle or ductile, or by physical mechanism of failure such as yielding
of the structure material, buckling, fracture (including fatigue), wear-out, etc. In the following examples
structural failures are classified in accordance to their causes. Note that the examples do not cover all
possible causes of failure such as, for example, accidental overload or under-strength, foundation
movements, etc. It is also important to understand that very often structural failures occur due to
simultaneous occurrences of a number of different causes, e.g., human error and natural hazard, or
deterioration and overload.

1.6.1 Failures due to human error

Human error is one way or the other the cause of most failures. These errors can arise from simple
slips, or ignorance and lack of understanding. Several failures have occurred because a designer forgot
to include an essential detail or simply made mistakes in calculations. Others occurred when the
behaviour of a material or the loading to which the structure would be subjected was not well
understood. Even if the designer gets it right the builder/constructor may get it wrong. Many failures
have occurred because someone forgot to put all reinforcing bars in a reinforced concrete member or
put them in wrong places; poor casting has led to boiler explosions and poor welding to keels falling off
yachts. Failures can also occur in service because structures or components are overloaded or not used
or maintained properly.

Safety, Risk and Reliability, Heriot-Watt University 27


1. Introduction to the Concept of Risk and Safety
Failure of the 1,907-foot (570-m) long I-35W Bridge over the Mississippi River in Minneapolis,
Minnesota occurred on August 1, 2007 when all three deck truss spans and several approach spans
suddenly collapsed (see Figure 1.5). Bridge deck overlay work was underway at the time of the collapse
with four of the eight traffic lanes closed to traffic. Approximately 110 vehicles including construction
equipment and materials were on the structure at the time of the collapse. Thirteen people were killed
and 145 people were injured. The collapse initiated in the main span at the south end of the bridge, at a
location referred to as Node U10. The investigation established that the cause of the failure was a
design error - the as-built capacity of the U10 gusset plates (see Figure 1.4) was much less than the
capacity required by the governing design code (Hill et al. 2008).

Figure 1.4 Node U10E before and after the collapse (from Hill et al. 2008).

Safety, Risk and Reliability, Heriot-Watt University 28


1. Introduction to the Concept of Risk and Safety
Figure 1.4 Overview of I-35W Bridge collapse (from Hill et al. 2008).

1.6.2 Failures due to natural hazards

Many failures have occurred due to (or been triggered by) natural hazards such as earthquakes,
storms, or floods. Often the structures that failed were not designed to withstand such events (this
especially concerns old structures) or the magnitude of a hazard was much higher than expected based
on data available at the time of design. In such cases, the natural hazard can be considered as a primary
cause of failure. Structures may also fail due to inadequate design caused by either inability to correctly
predict their behaviour under hazard-induced loads, or underestimation of the possible magnitude of a

Safety, Risk and Reliability, Heriot-Watt University 29


1. Introduction to the Concept of Risk and Safety
hazard (i.e., incorrect use of available data), or simply design error. In cases like this, the natural hazard
is just one of the causes (or trigger) of failure.

Of course, the distinction between the cases is not always clear cut. For example, somebody can say
that an old structure was not designed for a particular hazard due to lack of knowledge and the
structure failure when the hazard occurs is due to the lack of knowledge and not the hazard itself (i.e.,
the lack of knowledge is a primary cause of failure). Another example, a structure was properly designed
against a particular type of hazard but still failed when this type of hazard occurred. This happened
because the magnitude of the hazard was very high, the probability of such an event was assessed as
very low, and it was considered uneconomical to design for it. How should we classify this failure –
caused by a natural hazard or overload?

Earthquakes

Figure 1.6 Failure of a bridge column (Northridge, California 17/01/1994, magnitude 6.8).

Safety, Risk and Reliability, Heriot-Watt University 30


1. Introduction to the Concept of Risk and Safety
Figure 1.7 Collapse of a parking garage at California State University in Northridge (Northridge, California
17/01/1994, magnitude 6.8).

Figure 1.8 Leaning residential buildings in Niigata, Japan – foundation failure due to liquefaction
(Niigata, Japan 16/06/1964, magnitude 7.4).

Storms/strong winds

Three out of a group of eight cooling towers at Ferrybridge 'C' Power Station (West Yorkshire,
England) collapsed on 01/11/1965 (Figures 1.9 and 1.10), with the remaining towers sustaining severe
structural damage. The towers, each 375 feet (113 m) high, had been constructed closer together than

Safety, Risk and Reliability, Heriot-Watt University 31


1. Introduction to the Concept of Risk and Safety
was usual and had greater shell diameters and shell surface. High winds were considered to be the
trigger for the collapse, but an inquiry found that the cause of failure was a serious underestimation of
the wind loading in the initial design, in particular safety margins did not really cover any uncertainties
in the wind loadings.

Figure 1.9 A cooling tower comes crashing to the Figure 1.10 Aerial view of the towers after the
ground. collapse.

Both the east and westbound lanes of the section of Interstate 10 (I-10) that crosses Lake
Pontchartrain (near Slidell, Orleans Parrish, Louisiana, USA) suffered numerous failures during Hurricane
Katrina (August 2005, a Category 3 hurricane with sustained winds of 125 mph (205 km/h)). The
concrete bridge was built as a series of simply supported pre-stressed spans, prefabricated off-site for
ease of construction. The failure mode is similar to what has been seen after earthquakes, with loss of
support at the piers after dynamic loading. In this case, the non-continuous spans were unseated by
repeated uplifting and dropping during the storm (Figures 1.11 – 1.13, photos: J. O'Connor).

Figure 1.11 Close up view of the Figure 1.12 This prefabricated Figure 1.13 Front view of
eastbound spans collapsed into superstructure unit shifted off its westbound spans collapsed into
the water. bearings, but did not fall. the water.

Figure 1.14 shows failure of the Arcadia Community Centre (Arcadia, Florida) caused by Hurricane
Charley (August 2004, a Category 4 hurricane with sustained winds of 150 mph (240 km/h)). It was a
hybrid building construction with the lower 2/3 constructed from reinforced concrete block and light
metal framing on the upper 1/3. The wall failure was likely the result of the roof framing lifting from the
top wall plate, which caused the wall to lose its lateral support. This resulted in the wall framing being

Safety, Risk and Reliability, Heriot-Watt University 32


1. Introduction to the Concept of Risk and Safety
unable to sustain the lateral wind loads and progressive collapse began. The building was 1.5 years old
and used as a hurricane shelter during the storm. Some 1300 persons were inside the building as it
began to collapse at the height of the storm.

Figure 1.14 Partial collapse of the Arcadia Community Centre caused by Hurricane Charley.

1.6.3 Failures due man-made hazards

Explosions of domestic gas

Figure 1.15 Damage due to gas explosion. Figure 1.16 Partial collapse due to gas
explosion.

Safety, Risk and Reliability, Heriot-Watt University 33


1. Introduction to the Concept of Risk and Safety
Figure 1.17 shows partial collapse of a 22-storey
residential building in Ronan Point, UK, in 1968, when
a gas explosion in one of flats on the 18-th floor
caused the failure of an entire section of the building
(Griffiths et al. 1968). The Ronan Point collapse is an
example of progressive collapse, i.e., the type of
structural failure in which some local damage triggers
a chain reaction of failures causing collapse of the
whole structure or of a major part of it. In this
particular case, local damage was caused by gas
explosion; however, in general local damage may be
caused by different accidental events such as gas or
bomb explosion, vehicle impact, gross errors in
design, construction or utilization. The common
feature of accidental events is that they have very
low probability of occurrence (around 10-7-10-5 per
housing unit per year) and as such are usually not
considered explicitly in structural design. As a result,
when they occur they can cause a local damage to a
structure and if the structure is not sufficiently robust
Figure 1.17 Partial collapse of a 22-storey
residential building due to gas explosion in the latter may spread through it leading to its
Ronan Point, UK.
disproportionate failure (another definition of progressive collapse). The Ronan Point collapse was not
the first case of progressive collapse but due to its wide exposure attracted substantial attention to this
problem. Returning to Figures 1.15 and 1.16, the first one shows an example of a robust structure
(damage is limited to the immediate area of gas explosion) while Figure 1.16 shows another example of
progressive collapse. There are two other previously presented structural failures which can be
characterised as progressive collapse: the collapse of the bridge (Figures 1.4-1.5) triggered by failure of a
single node and the partial collapse of the Arcadia Community Centre (Figure 1.14) initiated by failure of
the roof frame.

Safety, Risk and Reliability, Heriot-Watt University 34


1. Introduction to the Concept of Risk and Safety
Bomb explosions

Figure 1.18 Bombing of the Murrah Federal Building in Oklahoma City, USA, 1995.

Aircraft collision

Figure 1.19 Partial collapse of the Pentagon, Washington, USA, 11/09/2001.

Vehicle impact

Figure 1.20 Private house damaged by a truck.

Safety, Risk and Reliability, Heriot-Watt University 35


1. Introduction to the Concept of Risk and Safety
1.6.4 Failures due to deterioration

Corrosion

Figure 1.21 Patrol station canopy failed due corrosion of supporting columns.

Figure 1.22 Collapse of a concrete pedestrian bridge over the Lowe’s Motor Speedway, Concord, NC due
to corrosion of prestressing strands, 20/05/2000.

Concrete deterioration

Figure 1.23 Collapse of a portion of the de la Concorde overpass, Laval, Montreal, Canada, 30/09/2006
(from Johnson et al. 2007).
Figure 1.23 shows the collapse of a portion of the de la Concorde overpass, Laval, Montreal, Canada,
which happened on 30/09/2006 and caused death of five people and seriously injured another six. The
overpass collapsed due to a shear failure in the southeast cantilever. One of the primary causes of the
Safety, Risk and Reliability, Heriot-Watt University 36
1. Introduction to the Concept of Risk and Safety
collapse was deterioration of concrete due freeze-thaw cycles in the presence of de-icing salts. Other
primary causes included poor design and misplacement of a reinforcing bar in the upper part of the
cantilever, which failed (Johnson et al. 2007).

Figure 1.24 Collapse of a portion of the roof of Terminal 2E in Paris-Charles de Gaulle Airport,
23/05/2004.
Figure 1.24 shows the collapse of a section of Terminal 2E in Paris-Charles de Gaulle Airport, which
occurred on 23/05/2004 killing four people. There were a number of causes for the collapse, one of
which was concrete cracking during construction and cycles of stress from differential thermal and
moisture movements between the concrete shell and the curved steel member and struts that led to an
increase in flexibility of the structure.

Revision Questions
1.1 What is the definition of risk in engineering?

1.2 What are the main stages of the risk management process?

1.3 When may the risk assessment be required in structural engineering?

1.4 What does the risk assessment include?

1.5 How can we categorise the types of risk analysis? When do we need to estimate the probability of
failure and what approaches can be used for it?

1.6 When is the risk treatment required? What approaches can be used for it?

1.7 How can we classify structural failures? What are possible causes of structural failures?

1.8 What is progressive collapse? Give examples of this type of structural failure.

Safety, Risk and Reliability, Heriot-Watt University 37


1. Introduction to the Concept of Risk and Safety
1.9 What is an accidental event? Give examples of accidental events.

SUMMARY

 The definition of risk used in engineering has been given. The risk management process has been
described.

 Possible causes of structural failures have been described and illustrated by examples.

 The terms “progressive collapse” and “accidental event” have been defined.

References
AS/NZS 4360 (1999). Australian/New Zealand Standard: Risk Management. Standards Australia.

Griffiths, H., Pugsley, A., and Saunders, O. (1968). Report of the Inquiry into the Collapse of Flats at
Ronan Point, Canning Town. Her Majesty's Stationery Office, London.

Hill, H.J. et al. (2008). I-35W Bridge over the Mississippi River Collapse Investigation Final Report WJE No.
2007.3702. Wiss, Janney, Elstner Associates, Inc., Northbrook, IL.

Johnson, P.M., Couture, A., and Nicolet, R. (2007). Report of the Commission of inquiry into the collapse
of a portion of the de la Concorde overpass. Gouvernement du Québec, Canada.

Matousek, M. and Schneider, J. (1976). Untersuchungen zur Struktur des Sicherheitsproblem bei
Bauwerken, Institut für Statik und Baukonstruktion, ETH Zürich, Bericht No. 59, ETH Zürich.

Melchers, R.E. (1999). Structural Reliability: Analysis and Prediction. 2nd Ed. John Wiley & Sons, Ltd,
Chichester, England.

Otway, H.J. et al. (1970). A Risk Analysis of the Omega West Reactor. Report No. LA 4449, Los Alamos
Scientific Laboratory, University of California, Los Alamos, CA.

Stewart, M. and Melchers, R. E. (1997). Probabilistic Risk Assessment of Engineering Systems, Chapman
& Hall.

Safety, Risk and Reliability, Heriot-Watt University 38


1. Introduction to the Concept of Risk and Safety
2 BASICS OF PROBABILITY THEORY

2.1 Introduction
Only when a solid basis is established for the treatment of uncertainties affecting the probability of
occurrence of a certain event, e.g., failure of an engineering system, it is possible to assess risks
associated with construction, operation and decommission of this system in a consistent and rational
way. The probability theory provides such a basis. In this lecture we consider some key elements of the
probability theory.

2.2 Elements of Set Theory


The modern theory of probability is based on the language of set theory. A set, X say, is a collection
of ‘things’ with a common property. The ‘things’ (or sample points) may be objects or actions or
numerical values and are generally referred to as members, x  X. We are concerned with measuring
the possibility that something will happen (or has happened), and subsets of our sets will be called
events. The set of all members of a common property is called the universal set I. The universal set is
also called the sample space S. The set that contains nothing at all is the empty set or null set .

As defined above, an event is a subset of a sample space and thus is also a set of sample points. If the
subset is empty, i.e., contains no sampling points, it is said that this event is impossible. If the subset
contains all sample points of a sample space, i.e., is identical to the sample space, the event is certain.

The algebra of sets, better known as Boolean algebra, is carried out with the following operators

 Union, the OR operator, (also denoted +),

 Intersection, the AND operator, (also denoted  or ),

\ Difference.

The last operation is related to taking the complement of a set,

X I\X

which is the NOT operator. Therefore I can be partitioned into X and its complement X , as illustrated in
the Venn diagram below (Figure 2.1).

Safety, Risk and Reliability, Heriot-Watt University 39


2. Basics of Probability Theory
I
X

Figure 2.1 A Venn diagram to illustrate X and its complement X .

Table 2.1 A table of the key results of Boolean algebra.

Mathematical symbolism Designation

(1a) X Y  Y  X Commutative Law


(1b) X Y  Y  X
(2a) X  (Y  Z )  (Y  X )  Z Associative Law

(2b) X  (Y  Z )  (Y  X )  Z
(3a) X  (Y  Z )  ( X  Y )  ( X  Z ) Distributive Law

(3b) X  (Y  Z )  ( X  Y )  ( X  Z )
(4a) X X  X Idempotent Law
(4b) X X  X
(5a) X  ( X Y )  X Law of Absorption

(5b) X  ( X Y )  X

(6a) (X Y)  X Y De Morgan’s Theorems

(6b) (X Y)  X Y

Safety, Risk and Reliability, Heriot-Watt University 40


2. Basics of Probability Theory
2.3 Definition of Probability

2.3.1 Classical definition

The classical definition goes back to Pascal and Fermat and up until 20-th century the probability
theory was based on this definition. The inspiration for this definition comes from the games of cards
and dice. According to this definition the probability of an event X, P(X), is determined without any
experiment as the ratio of the number of favourable outcomes, NX (i.e., the outcomes at which the
event X occurs) to the number N of all possible outcomes, provided that all the outcomes are equally
likely

NX
P X   (2.1)
N

For example, according to this definition the probability of getting “head” as a result of coin flipping is
0.5 since there are two equally likely outcomes of this experiment (“head” and “tail”), one of which is
favourable.

There are a number of problems with the classical definition

- it can only be applicable to experiments, whose outcomes are equally likely, e.g., it cannot be
used for the dice experiment when the dice is not fair;

- it requires that all possible outcomes be identified;

- it may cause difficulties when the number of outcomes is infinite.

2.3.2 Relative-frequency definition

The relative-frequency definition was developed by von Mises about seventy years ago. It is based on
experiments, easy to understand intuitively, and as a result of it popular among engineers. According to
this definition the probability of an event X, P(X), is simply the relative frequency of occurrence of X in
the experiment, i.e., if the experiment under consideration is carried out n times and X occurs in nX
outcomes then

nX
P X   lim (2.2)
n  n

In order to determine the probability of getting “head” as a result of coin flipping using this definition it
is necessary to flip a coin a number of times and as the number of coin flipping increases the probability
converges to 0.5.

Safety, Risk and Reliability, Heriot-Watt University 41


2. Basics of Probability Theory
Problems associated with the definition are:

- from a theoretical point of view, the number of experiments will be always finite, thus the
existence of the limit is just a hypothesis which cannot be validated experimentally;

- from practical point of view, the estimation of P(X) requires carrying out a sufficiently large
number of more or less identical experiments that very often is impossible, e.g., many structural
components are available in small numbers or unique and subject to different loads and operate
in different conditions.

2.3.3 Bayesian definition – probability as a measure of belief

According to this definition the probability of an event X, P(X), is simply a measure of belief that the
event X will occur expressed in quantitative terms as a real number between 0 and 1

P(X) = measure of belief that X will occur (2.3)

Of course, the measure of belief depends on the state of mind of a particular person and as such is
subjective, i.e., person dependent. However, in the Bayesian statistical analysis the probability includes
both the classical and relative-frequency definition. For example, if to return to the example with coin
flipping, a reasonable person using this definition would determine the probability of getting “head” as
0.5 by arguing that there are only two possibilities – “head” and “tail”, and since there is no reason to
give preference to any of these two outcomes the probability should be 0.5.

The main argument in favour of this approach in engineering risk assessment (decision making under
uncertainty) is that a subjective estimate of the probability of an event X (a prior probability) is better
than no estimate. However, as relevant data are collected and processed using the Bayesian statistical
analysis new estimates of P(X) (posterior probabilities) become less and less dependent on the prior
estimate and closer and closer to that based on the relative-frequency definition.

2.3.4 Axiomatic definition

In mathematics the definition of probability is based on the following three axioms and no matter
what of the above interpretations of probability is chosen as long as it satisfies these three axioms any
results derived through the correct use of the probability theory will be mathematically valid.

Axiom 1

The probability of an event X is

Safety, Risk and Reliability, Heriot-Watt University 42


2. Basics of Probability Theory
0  P X   1 (2.4)

Axiom 2

The probability of a certain event or the sample space S is unity

PS   1 (2.5)

Axiom 3

The probability of an event which is the union of two mutually exclusive events is the sum of the
probability of these two events

P X  Y   P X   PY  (2.6)

Axiom 3 can be extended to any number N of mutually exclusive events

P  N
j 1

X j   j 1 PX j 
N
(2.7)

From Axiom 3 also follows that Axiom 2 can be written as

 P X   1
all j
j (2.8)

where Xj are the simple events (sample points) constituting the sample space S.

2.4 Basic rules and theorems

2.4.1 Union of not mutually exclusive events

If two events X and Y are not mutually exclusive using Axiom 3 and the Venn diagram (Figure 2.2) it is
easy to show that the probability of their union is

P X  Y   P( X )  P(Y )  P( X  Y ) (2.9)

where P(X∩Y) is the probability the intersection of the events X and Y. The union can be seen to be the
area of X plus the area of Y minus the intersection of the two sets to avoid counting the intersecting area
twice.

Safety, Risk and Reliability, Heriot-Watt University 43


2. Basics of Probability Theory
Y Y
X
X

a) b)

Figure 2.2 a) the union of two sets; b) the intersection of two sets.

When X and Y are mutually exclusive their intersection is the empty set, i.e., (X∩Y)=, and the
corresponding probability equals zero so that Eq. (2.9) reduces to Eq. (2.6).

Eq. (2.9) can be generalised for any number N of non-exclusive events

P     P X
j 1
    P X  X k 
N N
N
j 1
Xj j j
j 1 j  2 k 1
(2.10)
 
j 1 k 1
    PX j  X k  X      (1)
N
N 1 N
P j 1
Xj
j 3 k  2  1

2.4.2 Multiplication rule and conditional probability

As can be seen from the previous section, in order to calculate the probability of the union of non-
exclusive events the probability of their intersection needs to be known (see Eqs. (2.9)-(2.10)). The latter
can be calculated using the multiplication rule, which for two non-exclusive events X and Y can be
written as

P X  Y   P X | Y PY   PY | X P X  (2.11)

where P(X|Y) and P(Y|X) are conditional probabilities, i.e., the probabilities of occurrence of one event
given that the other event has occurred. For example, P(X|Y) denotes the probability of occurrence of
the event X given that the event Y has occurred. The multiplication rule can be generalised for N events

P 
N
j 1
 
X j  P X 1   P X 2 | X 1   P X 3 | X 1  X 2    P X N |  j 1 X j
N 1
 (2.12)

If the occurrence of one event does not depend on the occurrence of another event such two events
are called statistically independent events. If X and Y are statistically independent events then

P X | Y   P X  (2.13)

Safety, Risk and Reliability, Heriot-Watt University 44


2. Basics of Probability Theory
Thus, for statistically independent events X and Y the multiplication rule (Eq. (2.11)) simplifies

P X  Y   P X PY  (2.14)

For N statistically independent events Eq. (2.12) becomes

P 
N
j 1

X j   j 1 PX j 
N
(2.15)

When events X and Y are non-exclusive but statistically independent the probability of their union can
be calculated as (see Eq. (2.9))

P X  Y   P( X )  P(Y )  P( X ) PY  (2.16)

When events are statistically independent Eq. (2.10) can also be simplified and becomes

P 
N
j 1

X j  1   j 1 PX j   1   j 1 1  PX j 
N N
(2.17)

Worked Example 2.1

The probabilities of occurrence of the three events A, B, and C shown in the Venn diagram (Figure
2.3) are P(A)=0.5, P(B)=0.4, P(C)=0.2 and P(A∩B)=0.15. Determine the probabilities of the following
events:

(a) A  C ; (b) B  C ; (c) A  B  C ; (d) A  B ; (e) A  B  C

A B
C

Figure 2.3 Venn diagram for Example 2.1.

Solution

(a) P A  C   P A  PC   P A  C   P A  PC   PC   P A  0.5

(b) PB  C   PB  PC   PB  C   0.4  0.2  0  0.6

Safety, Risk and Reliability, Heriot-Watt University 45


2. Basics of Probability Theory
(c) P A  B  C   P  0

   
(d) P A  B  P A  B  1  P A  B   1  0.15  0.85

(e)

 
P A  B  C  1  P A  B  C 
 1  P A  PB   PC   P A  B   PB  C   PC  A  P A  B  C 
 1  0.5  0.4  0.2  0.15  0  0.2  0  0.25

Worked Example 2.2

A bridge can be damaged either due to failure in its foundation (event F) or superstructure (event S).
The probabilities of these failures for the bridge are P(F)=0.008 and P(S)=0.005. It is also found that if
there is failure in the foundation, the probability of failure in the superstructure is 0.50.

(a) What is the probability of damage of the bridge?

(b) What is the probability of damage of the bridge if F and S are statistically independent?

Solution

(a) It is given that P(S|F)=0.5.

Pdamage  PF  S   PF   PS   PF  S   PF   PS   PS | F PF 


 0.008  0.005  0.5  0.008  0.009

(b) When F and S are statically independent

Pdamage  PF  S   PF   PS   PF  S   PF   PS   PF PS 


 0.008  0.005  0.008  0.005  0.01296

The example shows that the probability of damage to the bridge is higher when the two event are
statistically independent than when they are dependent.

Worked Example 2.3

Calculate the probability of failure of the nine-member truss shown in Figure 2.4 under design load.
The truss designed in such a way that the probability of failure of each of its members is 5×10 -5. It can be
assumed that failures of the individual truss members are statistically independent events.

Safety, Risk and Reliability, Heriot-Watt University 46


2. Basics of Probability Theory
X8 X9

X4
X3 X5 X6 X7

X1 X2
Figure 2.4 Truss from Worked Example 2.3.

Solution

Denote failures of the truss members as Xj (j=1,…,9). The truss is statically determinate; hence, it fails
if any of its members fails. The probability of the truss failure can then be expressed as

Ptruss failure   P X 1  X 2  X 3  X 4  X 5  X 6  X 7  X 8  X 9 

The probability of the union of the 9 independent events can be calculated by Eq. (2.17), thus

Ptruss failure   1   1  PX j   1  1  5  10 5  


9
9
 4.499  10 4 .
j 1

2.4.3 Theorem of total probability

The mutually exclusive and collectively exhaustive events X1, X2, …, XN are called partitions of the
sample space S (see the Venn diagram in Figure 2.5). The probability any event Y of S can then be
expressed as

PY    PX j  Y   PX j PY | X j 


N N
(2.18)
j 1 j 1

The last equation represents the theorem of total probability.

S
X2 X3 X4
X1

Xj XN

Figure 2.5 Venn diagram for the theorem of total probability.

Safety, Risk and Reliability, Heriot-Watt University 47


2. Basics of Probability Theory
2.4.4 Bayes’ theorem

The theorem of total probability allows to calculate the probability of an event Y, when it depends on
mutually exclusive and collectively exhaustive events X1, X2, …, XN with known probabilities. Often it may
be useful to know the probability of one of mutually exclusive and collectively exhaustive events Xj given
that the event Y has occurred, i.e., P(Xj|Y). From Eq. (2.11) follows that

PY | X j PX j 
P X j | Y   (2.19)
PY 

If to substitute Eq. (2.18) instead of P(Y) the following formula is obtained

PY | X j PX j 
P X j | Y   (2.20)
 PY | X PX 
N

j j
j 1

The formula is called Bayes’ theorem. The theorem is very important since it provides a tool for updating
probabilities of events based on new observations. It is widely used in the reliability assessment of
existing structures.

The conditional probability P(Y|Xj) is often referred as the likelihood, i.e., the probability of observing
an event Y given that Xj is the true state. The probability P(Xj) is referred as the prior probability of Xj,
i.e., the probability of Xj prior to observing the event Y (new observation), while P(Xj|Y) is the posterior
probability of Xj, i.e., the updated probability after observing Y.

Worked Example 2.4

Assume that the primary causes of serious damage to a building (event D) during its service life are
fire (event F), strong winds (event W) and earthquakes (event E). The probabilities of serious damage to
the building in the case of occurrence of one of these events are: 0.004 for F, 0.002 for W and 0.007 for
E. If none of these three hazards occurs (event O), the probability or serious damage to the building due
to other minor hazards is 0.001. It can be assumed that hazards, which can cause serious damage to the
building, cannot occur simultaneously. The probabilities of F, W and E during the building service life are
estimated as 0.1, 0.5 and 0.2, respectively.

(a) What is the probability of serious damage to the building during its service life?

(b) If the building has been seriously damaged, what is the probability that it was caused by fire? By
strong wind? By an earthquake? By none of these hazards?

Safety, Risk and Reliability, Heriot-Watt University 48


2. Basics of Probability Theory
Solution

(a) The events F, W, E and O are mutually exclusive and collectively exhausted. Thus, the probability of
serious damage can be calculated as

PD   PD | F PF   PD | W PW   PD | E PE   PD | O PO 


 0.004  0.1  0.002  0.5  0.007  0.2  0.001 0.2  0.003

(b) The probability that the damage was caused by fire

PD | F PF  0.004  0.1


P F | D     0.133
P D  0.003

By strong wind

PD | W PW  0.002  0.5


PW | D     0.333
P D  0.003

By an earthquake

PD | E PE  0.007  0.2


P E | D     0.467
P D  0.003

By other hazards

PD | O PO  0.001  0.2


PO | D     0.067 .
P D  0.003

Worked Example 2.5

An old concrete bridge needs to be assessed because of increasing traffic loads. The compressive
strength of the concrete, fc, used for the bridge is unknown and in order to estimate it concrete
cylindrical cores are drilled out of the bridge and tested. The following classification is used for the
concrete:

C1: 0 < fc ≤ 25 N/mm2

C2: 25 < fc ≤ 40 N/mm2

C3: fc > 40 N/mm2

Based on the experience with similar bridges the following probabilities that the concrete belongs to
a particular class have been assigned: C1 – 0.45, C2 – 0.40, and C3 – 0.15 (see Table 2.2).

The test method is not very accurate and when the test shows that the concrete belongs to a certain
class there is a probability that it can belong to another class. These probabilities have also been
estimated and shown in Table 2.2.

Safety, Risk and Reliability, Heriot-Watt University 49


2. Basics of Probability Theory
One test (T) has been carried out and the compressive strength obtained from it is 23.5 N/mm 2.
Using this new information, update the probabilities of the concrete belonging to one of the three
classes.

Table 2.2 Data for Worked Example 2.5.

Concrete class Prior Likelihoods, P(T|Ci)


probabilities T=C1 T=C2 T=C3
C1 0.45 0.75 0.23 0.02
C2 0.40 0.18 0.65 0.17
C3 0.15 0.01 0.29 0.70

Solution

The posteriors probabilities given T=C1(23.5 N/mm2)

PT  C1 | C1PC1
PC1 | T  C1 
PT  C1 | C1PC1  PT  C1 | C 2PC 2  PT  C1 | C 3PC 3
0.75  0.45 0.75  0.45
   0.821
0.75  45  0.18  0.40  0.01  0.15 0.4110

PT  C1 | C 2PC 2
PC 2 | T  C1 
PT  C1 | C1PC1  PT  C1 | C 2PC 2  PT  C1 | C 3PC 3
0.18  0.40 0.18  0.40
   0.175
0.75  45  0.18  0.40  0.01  0.15 0.4110

PT  C1 | C 3PC 3
PC 3 | T  C1 
PT  C1 | C1PC1  PT  C1 | C 2PC 2  PT  C1 | C 3PC 3
0.01  0.15 0.01  0.15
   0.004
0.75  45  0.18  0.40  0.01  0.15 0.4110

Exercises
Exercise 2.1

A building can be damaged either by fire or by an earthquake. Based on previous observations it is


estimated that the probability of fire is 0.003 and the probability of an earthquake which can damage
the building is 0.01. Calculate the probability of damage of the building. Assume that fire and
earthquakes are statistically independent events.

Ans. 0.01297

Safety, Risk and Reliability, Heriot-Watt University 50


2. Basics of Probability Theory
Exercise 2.2

There are three sources of power supply to a village – electricity, gas and oil. It may be shortage of
these sources during the coming winter with the following probabilities: of electricity (event E) 0.10, of
gas (event G) 0.05, and of oil (event O) 0.15. Moreover, if there is a shortage of oil, the probability of an
electrical power shortage increases by 70%, i.e., becomes 0.17. A shortage of gas is statistically
independent of shortages of oil and electricity.

(a) What is the probability of shortage of the power supply to the village during the coming winter
(i.e., shortage of the three sources at the same time)?

Ans. 1.275×10-3

(b) What is the probability of a shortage of at least one of the three sources?

Ans. 0.263275

(c) If there is a shortage of electricity, what is the probability that there are also shortages of gas and
oil?

Ans. 0.01275

Exercise 2.3

There is a concern that reinforcing steel in a concrete beam is subject to corrosion. The severity of
corrosion depends on the corrosion rate expressed in terms of the corrosion current density, icorr, and
classified as follows:

Low (L): icorr < 1 μA/cm2


Medium (M): 1 μA/cm2 ≤ icorr ≤ 3 μA/cm2
High (H): icorr > 3 μA/cm2
Based on the experience with similar beams in similar conditions and visual observations the following
probabilities that corrosion in the beam belongs to one of the above three classes have been assigned:
low – 0.35, medium – 0.60, and high – 0.05.

In order to obtain more information about the corrosion rate it has been decided to perform
corrosion measurements (CM) on-site. However, the device used for this is not very accurate and when
it shows that the corrosion rate belongs to a certain class there is a probability that it can belong to
another class. These probabilities are shown in the table below.

Corrosion Prior probabilities Likelihoods, P(CM|Class)


rate CM=Low CM=Medium CM=High
Low 0.35 0.80 0.19 0.01
Medium 0.60 0.20 0.70 0.10
High 0.05 0.02 0.23 0.75

Safety, Risk and Reliability, Heriot-Watt University 51


2. Basics of Probability Theory
The corrosion rate in the beam has been measured and the result was icorr = 2 μA/cm2. Using this new
information, update the probabilities of the corrosion rate belonging to one of the three classes.

Ans. Updated probabilities: corrosion rate belongs to L = 0.1335, to M = 0.8434, to H = 0.0231.

Exercise 2.4

The annual probabilities of occurrence of a natural fire and a strong earthquake in a town have been
estimated as 0.01 and 0.005, respectively. Moreover, a strong earthquake by itself can cause a fire with
the probability of 0.4. It can be assumed that natural fires and earthquakes are statistically independent
and mutually exclusive events.

(a) What is the annual probability that the town will have a fire due to a strong earthquake?

Ans. 0.002

(b) What is the annual probability that the town will have a fire?

Ans. 0.012

(c) What is the probability that there will be no fires in the town in the next 10 years?

Ans. 0.8863

Exercise 2.5

A concentrated load on a cantilever beam may be placed at either location A or B, with probabilities
P(A)=0.3 and P(B)=0.7. If the load is placed at A, the probability of bending failure of the beam is 0.01
and the probability of shear failure is 0.001. If the load is placed at B, the probability of bending failure is
0.02 and the probability of shear failure is still 0.001. What is the overall probability of failure of the
beam?

Ans. 0.018

Revision Questions
2.1 What are the definitions of set, universal set (or sample space) and empty (or null) set?

2.2 What definitions of probability do you know?

2.3 How is the probability of union of two events calculated?

2.4 How is the probability of intersection of two events calculated?

2.5 What are statistically independent events?

Safety, Risk and Reliability, Heriot-Watt University 52


2. Basics of Probability Theory
2.6 What is the conditional probability?

2.7 Formulate the theorem of total probability.

2.8 Formulate Bayes’ theorem.

2.9 What are likelihood, prior and posterior probabilities?

SUMMARY

 The definition of set and the key formulae of Boolean algebra have been given.

 Existing definitions of probability have been described and their shortcomings have been discussed.

 Basic rules and theorems of the probability theory such as the probabilities of union and intersection
of events, theorem of total probability and Bayes’ theorem have been formulated and their
applications have been illustrated by examples.

Safety, Risk and Reliability, Heriot-Watt University 53


2. Basics of Probability Theory
3 UNCERTAINTY MODELLING

3.1 Types of uncertainty in structural engineering


In risk and reliability analysis it is common to distinguish the following types of uncertainty:

- inherent variability,

- model uncertainty, and

- statistical uncertainty.

The first type (inherent variability) is also often referred to as aleatory uncertainty (i.e., uncertainty
that arises due to natural, unpredictable variation in the performance of the structural system under
consideration), while the other two (model and statistical uncertainties) as epistemic uncertainty (i.e.,
uncertainty due to a lack of knowledge about the behaviour of the structural system that in principle can
be resolved).

The inherent variability may be split into two subtypes: one is influenced by human activities and the
other is not. To the first subtype belong, for example, uncertainties associated with the strength of
materials (i.e., concrete, steel), which can be reduced by improving production technology and more
strict quality control measures. To the second belong uncertainties associated with natural loads (e.g.,
wind, earthquake), environmental conditions (e.g. temperature, humidity), or properties of natural
materials (e.g., soil).

The model uncertainty can also be split into two subtypes. Uncertainties associated with a lack of
knowledge about structural behaviour or intentional simplifications of models used for its description
belong to the first subtype. These uncertainties may be reduced by further research or the use of more
sophisticated models if such are available. The second subtype is related to uncertainties associated
with the prediction of future developments (e.g., the prediction of future traffic loads), which cannot be
reduced significantly by further research.

The statistical uncertainty is usually caused by a limited number of observations or tests. Collection
of more relevant data via observations or tests usually enables to decrease this type of uncertainty.

3.2 Random variables


The behaviour of structures/engineering systems is described by models (analytical, empirical or
numerical). In order to take into account various uncertainties associated with structural properties,
loads and models themselves parameters of the models are represented by random variables. A random

Safety, Risk and Reliability, Heriot-Watt University 54


3. Uncertainty Modelling
variable is a numerical variable whose specific value cannot be predicted with certainty before an
experiment has been carried out and its outcome has been observed/measured.

In mathematical terms, a random variable X can be defined as a number assigned to an outcome of


an experiment specified in the sample space S, i.e., X gives the rule of correspondence between any
member of S and the number assigned to it. In the following a random variable will be denoted by a
capital letter, while a particular realisations of the random variable by the corresponding lowercase
letter.

A random variable can be either a discrete random variable or a continuous random variable. A
discrete random variable has a finite or infinite countable number of values, which are often positive
integers. The number of earthquakes within a certain period of time, the number of cars crossing a
bridge, and the duration in days of a construction activity are examples of discrete random variables. A
continuous random variable has always an infinite number of values and is free to take on any value on
the real axis. Note that the latter does not mean that a continuous random variable should take on
values over the entire axis, e.g., it can be defined over a certain interval or can be only positive. Strength
of materials, dimensions of structural elements, and magnitude of loads are examples of continuous
random variables.

The relationship between values of a random variable and the corresponding probabilities is
described by the probability distribution of the random variable.

3.3 Probability distributions

3.3.1 Discrete random variables

The probability distribution of a discrete random variable is usually described by a probability mass
function (PMF). The PMF of a discrete random variable X, pX(xi), is the probability of the event that the
random variable equals to a particular value xi

p X xi   P X  xi  (3.1)

Since the PMF represents probability, it should satisfy the three axiom of probability theory, i.e.

0  p X xi   1 for all xi

 p x   1
all xi
X i (3.2)

Safety, Risk and Reliability, Heriot-Watt University 55


3. Uncertainty Modelling
xi b
Pa  X  b    p x  X i
xi  a

The cumulative distribution function (CDF) of a random variable X, FX(x), which is defined for both
discrete and continuous variables, is the probability of the event that X does not exceed x.

FX x   P X  x  (3.3)

For a discrete random variable this function can be expressed as

FX x    p x  X i (3.4)
all xi  x

3.3.2 Continuous random variables

The probability distribution of a continuous random variable X can be described by its CDF, FX(x),
which is defined by Eq. (3.3). It can also be described by the probability density function (PDF), fX(x),
which is the derivative of the CDF

dFX x 
f X x   (3.5)
dx

The probability that X is within an infinitesimal interval between x and x+dx is then equal to fX(x)dx, i.e.,

Px  X  x  dx   f X x dx (3.6)

For an interval of finite length [x1,x2] the probability that X takes on a values in this interval is

x2

Px1  X  x 2    f x dx
X (3.7)
x1

If the PDF of X is known, its CDF can be found as

x
FX x   P   X  x    f  y dy
X (3.8)


Note that a value of fX(x) is not itself a probability and, therefore, is not necessarily less or equal to 1.
The PDF should satisfy the following conditions

f X x   0 (3.9)

 f x dx  1

X (3.10)

Safety, Risk and Reliability, Heriot-Watt University 56


3. Uncertainty Modelling
The CDF of both discrete and continuous random variables has the following properties

0  FX x   1 (3.11)

FX    0 (3.12)

FX   1 (3.13)

FX x     FX x  for any positive ε (3.14)

FX x2   FX x1   Px1  X  x2  (3.15)

3.4 Expected value

3.4.1 Expected value of a function

The expected value (or expectation) of a function g(X) of a continuous random variable X is denoted
as E(g) and defined as


E g    g x  f x dxX (3.16)


The expected value exists if

 g x  f x dx  

X (3.17)

For a discrete random variable

E g    g x  p x 
i X i (3.18)
all xi

The expected value can be interpreted as a weighting of g(X) by fX(x).

3.4.2 Moments of a random variable

If g(X)=Xn, where n is a positive integer, the expected values of g(X) are called moments of X


EX    x
n n
f X x dx (3.19)


Safety, Risk and Reliability, Heriot-Watt University 57


3. Uncertainty Modelling
where E(Xn) is the n-th moment of X. Note that this moment corresponds to the n-th moment of the
area of the PDF with respect to the origin.

The most important moment is for n=1, which is called the mean (or mean value) of a random
variable X and further denoted as μX


 X  E  X    xf X x dx (3.20)


The mean represents the first moment of the area of the PDF about the origin and also the centroid of
the PDF area, since the PDF area should be equal to unity.

Eq. (3.19) gives the moments of the area of the PDF with respect to the origin but, of course, it is
possible to estimate moments of the area about any point. Moments with respect to the mean are
called central moments

   x  

E X   X   n f X x dx
n
X (3.21)


Central moments correspond to the moments of areas with respect to their centroids.

The most important central moment is for n=2, which is called the variance of a random variable and
further denoted as Var(X)

   x  

Var  X   E  X   X   2 f X x dx
2
X (3.22)


The square root from the variance is called the standard deviation of a random variable and will be
denoted as σX

 X  Var  X  (3.23)

The ratio of the standard deviation of a random variable to its mean is called the coefficient of
variation (COV)

X
COV  X   (3.24)
X

Another central moment for n=3 is called the skewness and it characterises the asymmetry of a
probability distribution

   x  

skewness  E  X   X   3 f X x dx
3
X (3.25)


Safety, Risk and Reliability, Heriot-Watt University 58


3. Uncertainty Modelling
If a distribution is symmetrical, the skewness is zero. Positive values of the skewness usually correspond
to PDFs with dominant tails on the right; negative values to long tails on the left.

The above formulas are for continuous random variables. If X is a discrete random variable, its mean,
variance and skewness can be calculated using the following formulas

X   x p x 
all xi
i X i (3.26)

Var  X    x   X  p X  xi 
2
i (3.27)
all xi

skewness   x   X  p X  xi 
3
i (3.28)
all xi

3.4.3 Properties of the expectation operation

It can easily be shown that the expectation operation possesses the following properties

E a   a (3.29)

E aX   aE  X  (3.30)

Ea  bX   a  bE X  (3.31)

Eg1  X   g 2  X   Eg1   Eg 2  (3.32)

where a and b are constants. Note that in general the expectation of a function of X cannot be found by
substituting the mean of X into the function, i.e.,

Eg  X   g E  X  (3.33)

Several useful properties of variances can also be derived

Var a   0 (3.34)

Var aX   a 2Var  X  (3.35)

Var a  bX   b 2Var  X  (3.36)

Safety, Risk and Reliability, Heriot-Watt University 59


3. Uncertainty Modelling
3.5 Commonly used probability distributions

3.5.1 Discrete random variables

3.5.1.1 Binomial distribution

Let us consider a situation when a structural component is tested N times. Each test has two
outcomes: the component survives the test or it fails. These outcomes are collectively exhaustive (there
is no other possible outcome) and mutually exclusive (they cannot both be true at the same time). The
tests are also statistically independent (the result of any one test has no influence on the others). We
also assume that the probability of success, (1-p), or failure, p, is the same for all tests. Such tests are
called Bernoulli trials.

What is the probability that there are three failures in twenty-five trials? We are not interested in
which trials the component fails. This problem can be tackled by using Boolean algebra and the axioms
of probability theory. However, we can find an answer to this question more easily by using the Binomial
probability distribution, which is designed to answer the question we have just posed.

The Binomial distribution depends on two parameters: N (the number of trials) and p (the probability
of failure in any one trial). The properties of this distribution are

Mean   Np (3.37)

Variance  2  Np(1  p) (3.38)

The probability mass function is

p X ( xn )  CnN p n (1  p) N n (3.39)

The binomial coefficient is defined by means of factorials

N!
C nN  0n N (3.40)
n!( N  n)!

It is more conveniently defined using Pascal’s triangle:


N C nN
0 1
1 1 1
2 1 2 1
3 1 3 3 1
4 1 4 6 4 1
5 1 5 10 10 5 1
6 1 6 15 20 15 6 1

Safety, Risk and Reliability, Heriot-Watt University 60


3. Uncertainty Modelling
In each row n is labelled from 0 to N. The first and last entry on each row equals 1, and the interior
entries are the sum of the pair of values on the line immediately above it, according to the formula

CnN  CnN11  CnN 1

The cumulative distribution function CDF is

n
FX ( xn )   C jN p j (1  p) N  j (3.41)
j 0

What is the probability of 3 failures out of 25? The answer is obtained from the probability mass
function

p X ( x3 )  C325 p 3 (1  p) 22  2300 p 3 (1  p) 22

Worked Example 3.1

The annual probability of failure of a bridge due to an earthquake is estimated as 5×10-5. The design
life of the bridge is 100 years. Assume that the annual probability of failure remains constant over the
design life and failures of the bridges in different years are statistically independent events. What is the
probability of failure of the bridge due to an earthquake during its design life?

Solution

The probability of no failure of the bridge during 100 years can be estimated using the binomial
distribution as

P(no failure in 100 years)  p X x0   C0100 p 0 1  p 


100 0

100!
0!100  0!
5  10 5  1  5  10 5 
0 100

 0.9950
Pfailure in 100 years  1  Pno failure in 100 years  1  0.9950  0.0050

3.5.1.2 Poisson distribution

The mean  is a single parameter of the Poisson distribution. The PMF is given by

 n
p N ( n)  e (3.42a)
n!

This distribution may be used to approximate the binomial distribution when the probability p is very
small and N large. It is obtained by taking the limits

p 0 N   Np    O(1)

Safety, Risk and Reliability, Heriot-Watt University 61


3. Uncertainty Modelling
where O(1) reads “of order one”, meaning that the result is a finite value little different in magnitude
from unity.

The PMF of the Poisson distribution can also be written in a slightly different form. Let λ denotes the
mean annual occurrence rate of an event under consideration. Then, over a period of t years, the event
will occur on average (λt) times. If to divide the time period t into N intervals (N is very large so the
intervals are very short), when it can be assumed that the event may occur only once in each interval
and the probability of this occurrence p=(λt)/N, i.e., (λt)=μ and the PMF expressed as

p N ( n )  e  t
t n (3.42b)
n!

gives the probability on n occurrences of the event over t years.

The CDF of the distribution is

n
j
FN (n)  e    (3.43)
j 0 j!

Worked Example 3.2

The safety of a building in a seismic region is examined. According to available data there have been
three strong earthquakes in the region during the last 150 years. Assessment of the building also shows
that the probability of damage to the building during a strong earthquake is 0.15. Assume that damages
to the building during different earthquakes are statistically independent events. (a) What is the
probability of no strong earthquake in the region over next 100 years? (b) What is the probability that
there will be only two strong earthquakes over the next 100 years? (c) What is the probability that the
building will be damaged due to a strong earthquake over the next 100 years?

Solution

(a) The mean rate of occurrence of strong earthquakes, λ, can be estimated as 3/150=0.02 per year.
Thus, μ=λt=0.02×100=2.

20
Pno strong earthquake in 100 years  p0  e 2  0.1353
0!

22
(b) P2 strong earthquakes in 100 years  p2  e 2
 0.2707
2!

(c) Denote the event of the building damage due to a strong earthquake as D.

Safety, Risk and Reliability, Heriot-Watt University 62


3. Uncertainty Modelling
 
PD   1  PD   1   PD | N  n  pn   1   1  0.15 e  2
n 2n
n 0 n 0 n!

 1  e 2 

0.85  2 n
 1  e  2 e1.7  0.2592
n 0 n!


xn
Note that e   x

n  0 n!

3.5.2 Continuous random variables

3.5.2.1 Normal or Gaussian distribution

The PDF of a normal distribution, which has the mean  and standard deviation  as parameters, is

1  1 x   2 
f X ( x)  exp    
 2    
(3.44)
2   

The random variable x is defined in the range (-,). The standard form of this distribution is obtained
by replacing the random variable x by an equivalent normalised value, u, called the standard form,
which is defined as

x
u (3.45)

The PDF becomes

1  u2 
f U (u )  exp    (3.46)
2  2 

and the CDF is

x
FX x   P( X  x)     (u ) (3.47)
  

The mean of the standard form of the distribution is 0, the variance is 1 and the skewness is 0. The
normalised form of the normal distribution is illustrated in Figure 3.1.

Safety, Risk and Reliability, Heriot-Watt University 63


3. Uncertainty Modelling
1
PDF
CDF 0.8

0.6

0.4

0.2

0
-6 -4 -2 0 2 4 6
u

Figure 3.1 The standard form of the normal distribution.

A table with numerical values of the standard normal CDF, (u), is given in Appendix 2. The table can
be used in two ways: (i) find (u) given u, (ii) find u, given (u). In the first case, find (-1.14) and
(0.78). For u = -1.14, in the first column (headed u) find the row –1.1, then find the column with
heading value 0.04. The table entry for this row and column gives the required result, (-1.14) = 0.1271.
For u = 0.78, find the row 0.7, and the column 0.08, then (0.78) = 0.7823. When u is negative, the
column value is negative, u = -1.1-0.04 = -1.14, whereas when u is positive, the row and the column u
values are added, u = 0.7+0.08 = 0.78.

In the second case, let us find u when (u) = 0.9996023. Inspecting the second column of the table,
we find that the values of (u) change from 0.9995 to 0.9996 between u = 3.3 and u = 3.4. Looking along
the row u=3.3, we find that (u) changes from 0.9995959 to 0.9996103 between columns 0.05 and
0.06. If we required an accurate value of u we should interpolate linearly between these column values.
However, u estimated to two decimal places is sufficiently accurate for most purposes, and we should
choose the entry closest to the required value, which is u = 3.35 in this case.

Useful property of a normal distribution: the sum of independent normally distributed random
variables Xi (i=1,…,n) is also normally distributed with a mean, μΣ, equal to the sum of their means and a
variance, σΣ2, equal to the sum of their variances, i.e.,

n n
    X i
and  2    X2 i
i 1 i 1

Safety, Risk and Reliability, Heriot-Watt University 64


3. Uncertainty Modelling
Worked Example 3.3

Characteristic value of a material strength is often defined as a 5% fractile of the statistical


distribution of the strength, i.e., there is a 0.05 probability that the actual strength of the material in a
sample is less than the characteristic value.

Compressive strength of concrete is described by a normal distribution with mean 48 MPa and
standard deviation 6 MPa. Find the characteristic compressive strength of the concrete.

Solution

 f c ,k  48 
According to the definition of characteristic strength    0.05 , where fc,k denotes the
 6 
characteristic compressive strength of concrete. From the table in Appendix 2, Φ(-1.645) = 0.05. Thus,
(fc,k – 48)/6 = -1.645 so that fc,k = 48 – 1.645×6 = 38.13 MPa.

Worked Example 3.4

This example illustrates the approximation of the binomial distribution by the normal distribution.
Consider n failures in a batch of 75 components when the probability of failure of a component is p =
0.36. (a) Determine the probability that there are exactly 27 failures. (b) Determine the probability that
there are between 25 and 30 failures.

Solution

(a) The binomial distribution gives the result

0.36 27 1  0.36
75! 75 27
P(n  27)   0.09562
(75  27)!27!

Approximating the binomial distribution by the normal distribution, we have the parameters of the
normal distribution

  Np  0.36  75  27  2  N p1  p   17.28   4.157

The probability density function evaluated at n = 27 gives a reasonable estimate of the corresponding
probability mass function of the Binomial distribution:

1  1 n   2  1  1  27  27  2  1
f (n  27)  exp     exp    
2   2     2 4.157  2  4.157   2 4.157
   
 0.0960

It seems surprising that a probability can be estimated from a probability density. In fact the
approximate value is f(n)n, where the change in random variable n=1. A more precise statement of

Safety, Risk and Reliability, Heriot-Watt University 65


3. Uncertainty Modelling
the probability mass function is obtained by integrating the probability density function over the range
(n-0.5, n+0.5):

 x 
n  0.5  n  0.5     n  0.5     0.5    0.5 
P(n  27)   f  dx            
n 0.5
          4.157   4.157 
 0.12   0.12  0.5478  0.4522
 0.0956

The probability is read from the table given in Appendix 2, and this result is closer to the true value.

(b) From the binomial distribution the probability that the number of failures lies in the stated range is

30

 (75  n)!n! 0.36 1  0.36


75! 75 n
CDF (n  30)  CDF (n  25)  n
 0.5246
n  25

The above calculation is easily done with a computer, but is rather inconvenient if it must be done by
hand.

In estimating the probability over the range of random values we calculate the difference of the CDFs
of the normal distribution determined at the upper and lower limits, n = 30 and 25 respectively. The
upper limit is modified by adding one half, n+0.5; and the lower limit by subtracting one half, n-0.5. The
limits are u1 = (25-0.5-)/ = (25-0.5-27)/4.157= -0.60, and u2 = (30+0.5-)/ = 0.84. Φ(0.84)-Φ(-
0.60)=0.7995-0.2743=0.5252. The obtained solution compares well with the true value.

3.5.2.2 Lognormal distribution

A random variable has a lognormal distribution when its natural logarithm has a normal distribution.
The random variable x lies in the range [0,).The PDF is

1  1  ln x    2 
f X ( x)  exp      (3.47)
 x 2  2    
 

The CDF is

 ln x   
FX ( x)    (3.48)
  

The lognormal distribution depends on two parameters ξ and λ. They are related to the mean  and
variance 2 according to

 2 
  exp      2   2 exp( 2 )  1 (3.49)
 2 

Safety, Risk and Reliability, Heriot-Watt University 66


3. Uncertainty Modelling
1    2 
  ln    2   ln 1     (3.50)
2   
 

Note that when COV = (σ/μ) < 0.3 it can be assumed that ξ ≈ COV=σ/μ. The PDF is illustrated in Figure
3.2 below.

Useful property of a lognormal distribution: the product of independent lognormally distributed


random variables Xi (i=1,…,n) is also lognormally distributed with the following parameters:

n n
    i
and  2    2 i
i 1 i 1

4
0.1

3
PDF

2
0.5
1 1

0
0 0.5 1 1.5 2 2.5 3
x

Figure 3.2 The probability density function of the lognormal distribution; the numbers attached to each
curve are the values of ξ, λ=0 for all the curves.

Worked Example 3.5

Compressive strength of concrete is described by a lognormal distribution with mean 48 MPa and
standard deviation 6 MPa. Find the characteristic compressive strength of the concrete.

Solution

The parameters of the lognormal distribution: COV = 6/48 = 0.125 < 0.3, hence ξ=0.125 and λ=ln(48)-
0.5×0.1252=3.8634. According to the definition of characteristic strength

 ln  f c ,k   3.8634 
   0.05
 0.125 

where fc,k denotes the characteristic compressive strength of concrete. From the table in Appendix 2, Φ(-
1.645) = 0.05. Thus, [ln(fc,k) – 3.8634]/0.125 = -1.645 so that

Safety, Risk and Reliability, Heriot-Watt University 67


3. Uncertainty Modelling
fc,k = exp(3.8634 – 1.645×0.125) = 38.77 MPa.

3.5.2.3 Uniform distribution

The uniform distribution has PDF:

 1
 , a xb
f X x    b  a (3.51)

 0, elsewhere

The CDF is a piecewise straight line

 0, xa
x  a
FX  x    , a xb (3.52)
b  a
 1 xb

The mean and standard deviation are:

ab ba
 ;  (3.53)
2 12

3.5.2.4 Exponential distribution

If events occur according to a Poisson distribution, then the time T before the first occurrence of the
event, that means no occurrence (n=0) in time t ≤ T, can be represented by an exponential distribution.
It can easily be shown

PT  t   p N n  0  e t
t 0  e t (3.54)
0!

Thus, the CDF of T is

FT (t )  PT  t   1  e t

The PDF is given by

dFT
f T (t )    e  t , t ≥ 0 (3.55)
dt

The mean and variance are given, respectively, by

1 1
 and  2  (3.56)
 2

Safety, Risk and Reliability, Heriot-Watt University 68


3. Uncertainty Modelling
3.5.2.5 Other probability distributions

The Beta, Gamma, and Pareto distributions are often applied to reliability problems, usually in
simplified form. The probability density functions for these distributions are given in this section for
completeness.

The Beta distribution is defined on a finite range a  x  b ; outside this range it is zero. The PDF is:

( p  q) x  a  b  x 
p 1 q 1
f X ( x)  a  x  b, p  0, q  0 (3.57)
 ( p ) ( q ) b  a  pq1

where (.) is the Gamma function. The CDF is given by the incomplete beta function (not considered in
this module). The mean and variance are:

aq  bp
 (3.58)
pq

 2

b  a  pq
2
(3.59)
 p  q 2  p  q  1

A common choice of parameters is a = 0, p = 1, q is a positive integer, and, if x is the time to failure, b is


the time at which an item is sure to have failed. In this case the CDF can be calculated more easily:

q
 x
FX ( x )  1  1   (3.60)
 b

The Gamma distribution is defined on the range 0  x   , and has the following PDF:

 p x p 1 exp( x)
f X ( x)  x  0, p0 (3.61)
( p )

The CDF depends upon the incomplete gamma function (not considered in this module). However, if p is
an integer, the CDF can be determined by integration by parts.

The CDF of the Pareto distribution can be written as

p
 a 
FX ( x)  1    x  0, a  0, p0 (3.62)
 xa

Safety, Risk and Reliability, Heriot-Watt University 69


3. Uncertainty Modelling
3.6 Extreme value distributions

3.6.1 Modelling extremes

In many engineering applications the largest or smallest values of random variables are of special
interest. When loads are considered, their largest values are usually the most important. The resistance
of a system may also depend on extremes – the strength of its weakest component.

When the variable Y, representing the maximum of n random variables Xi (i=1,...,n) is of interest

Y  max X 1 , X 2, , X n  (3.63)

its CDF can be expressed as

FY  y   PY  y   Pall n of X i  y  (3.64)

If the Xi’s are independent then

FY  y   P X 1  y P X 2  y  P X n  y   FX1  y FX 2  y  FX n  y  (3.65)

and in the case when all the Xi’s have identical distributions with the CDF FX(x)

FY  y   FX  y 
n
(3.66)

If the variable Z, representing the minimum of n mutually independent and identically distributed
random variables Xi (i=1,...,n) is of interest

Z  min X 1 , X 2, , X n  (3.67)

in a similar way it can be shown that its CDF can be expressed as

FZ z   1  1  FX z 
n
(3.68)

When it can be assumed that the Xi’s are mutually independent and identically distributed, in a
number of important cases the shape of the distribution of their extreme value (maximum or minimum)
is relatively insensitive to the exact shape of the distribution of the Xi’s. In these cases, limiting
(asymptotic, as n grows) forms of the distribution of the extreme value can be found. The asymptotic
distributions depend on whether maximum or minimum values are of interest, and also on the
behaviour of the appropriate tail of the underlying distribution of the Xi’s. These asymptotic
distributions form a group of the so-called extreme value distributions.

Safety, Risk and Reliability, Heriot-Watt University 70


3. Uncertainty Modelling
Worked Example 3.6

Ten cracks have been detected in a steel beam during its inspection. Assume that the crack size can
be modelled by a normal distribution with mean 12.5 mm and COV = 0.10. What is the probability that
the maximum crack size is less than 15 mm?

Solution

Let X be the random variable representing the crack size. X is a normal random variable with a mean
of 12.5 mm and COV=0.10, i.e., σX=0.10×12.5=1.25 mm.

We are interested in the maximum value out of 10 cracks, which we denote as Y10 – the maximum
crack size of 10 cracks.

FY10  y   PY10  y   P X  y 


10

 15  12.5 
P X  15     (2)  0.9772
 1.25 

PY10  15  0.9772  0.794


10

3.6.2 Gumbel distribution

This maximum value distribution is known by names: Gumbel or Extreme Value Type 1 (EV 1)
distribution. The CDF of the Gumbel distribution is

  x  x0 
FX ( x)  exp   exp        x  ,   0 (3.69)
   

It is often used in structural reliability to describe extreme loads due to repeated application of n
statistically varying loads, and with this notation it is expressed in the form:

FY  y   exp  exp   n  y  u n  (3.70)

where y has replaced x as the random variable, αn is the reciprocal of θ, and un replaces the
threshold x0. The PDF, fY(y) is

f Y  y    n exp   n  y  u n   exp   n  y  u n  (3.71)

The mean and variance are given by:

Safety, Risk and Reliability, Heriot-Watt University 71


3. Uncertainty Modelling
0.5772157 2
  un  and 2  (3.72)
n 6 n2

Defining the standard variable w = αn (y-un), the Gumbel distribution and its PDF are plotted in Figure
3.3.

A related distribution is the Extreme minimum value distribution, type 1. This is defined by the CDF:

FZ z   1  exp  exp 1 z  u1   z   (3.73)

The PDF is:

f Z z   1 exp 1 z  u1   exp 1 z  u1  (3.74)

These are plotted in standard form below, in Figure 3.4.

1
PDF
CDF 0.8

0.6

0.4

0.2

0
-6 -4 -2 0 2 4 6
w

Figure 3.3 Gumbel distribution (αn=1).

1
PDF
CDF 0.8

0.6

0.4

0.2

0
-6 -4 -2 0 2 4 6
w

Figure 3.4 Extreme minimum value distribution, type 1 (α1=1).

Safety, Risk and Reliability, Heriot-Watt University 72


3. Uncertainty Modelling
3.6.3 Weibull distribution

This distribution is also called Extreme Value Type III (EV III) distribution of the smallest values.
Weibull developed this distribution while he was studying failures of materials due to fatigue and
fracture. This distribution depends on three parameters m, , and x0, where the last is an offset giving
the random variable in the range [x0, ).

The PDF for x  x0 is

m  x  x0 
m 1
  x  x0  m 
f X ( x)    exp      (3.75)
       
 

The CDF is

  x  x0  m 
FX ( x)  1  exp      (3.76)
    
 

The mean is

 1
    1    x0 (3.77)
 m

and the variance is

  2 2 1 
 2   2  1     1    (3.78)
  m  m 

where (.) is the Gamma function.

If we are given the mean and variance (and x0) then we have to solve equations for m and . The
parameter θ can be determined if the relationship between the m parameter and the coefficient of
variation (the ratio of the standard deviation to the mean) is known.

The PDF is illustrated in Figure 3.5 below, including the special case of the exponential distribution, m
= 1.

In some applications it is useful to approximate the normal distribution by a three-parameter Weibull


distribution using the relationships

m  3.44 (3.79a)

  3.46 (3.79b)

t0    3.11 (3.79c)

Safety, Risk and Reliability, Heriot-Watt University 73


3. Uncertainty Modelling
This transformation gives good results provided the CDF takes values in the interval (0.1,0.9). It has the
advantage that the probability requires the evaluation of an exponential in place of the probability
function of the normal distribution.

2
0.5
4
1.5
PDF

1 2
1

0.5

0
0 0.5 1 1.5 2 2.5 3
t/ 

Figure 3.5 The probability density function for the Weibull distribution; the numbers attached to each
curve are the values of m.

3.7 Multiple random variables


Up till now modelling of uncertainty by a single random variable has been considered. However, in
many problems uncertainties arise from a number of sources and in order to model them several
random variables need to be considered simultaneously. Joint behaviour of two or more random
variables is characterised by a joint probability distribution. In the following, joint probability
distributions of two random variables will be discussed; however, presented results can easily be
extended to any number of random variables.

3.7.1 Joint probability distributions

Consider two random variables X and Y. Their joint cumulative distribution function (CDF), FXY(x,y), is
defined as

FXY x, y   P X  x   Y  y   P X  x, Y  y  (3.80)

If X and Y are discrete random variables then their joint probability mass function (PMF), pXY(x,y), is
defined as

Safety, Risk and Reliability, Heriot-Watt University 74


3. Uncertainty Modelling
p XY x, y   P X  x  Y  y   P X  x, Y  y  (3.81)

and their joint CDF is

FXY x, y   
xi  x
 p x , y 
yj y
XY i j (3.82)

If X and Y are continuous random variables, the joint probability density (PDF), fXY(x,y), replaces the
PMF

2
f XY x, y   FXY x, y  (3.83)
xy

The joint PDF can also be interpreted the probability of the intersection of two events

Px  X  x  dx   y  Y  y  dy   f XY x, y dxdy

The joint CDF of two continuous random variables is then

x y

FXY x, y     f x, y dxdyXY (3.84)


 

and the probability of the joint occurrence of X and Y in some region in the sample space is determined
by integration of their joint PDF over this region

x2 y2

Px1  X  x2    y1  Y  y 2     f x, y dxdy


XY (3.85)
x1 y1

3.7.2 Marginal distributions

Sometimes it may be necessary to eliminate consideration of Y and to study only the behaviour of X.
In such cases the marginal PDF (when X is continuous) or PMF (when X is discrete) needs to be
calculated


f X x    f x, y dy
XY (3.86)


or

p X x    p x, y  XY i (3.87)
all yi

Safety, Risk and Reliability, Heriot-Watt University 75


3. Uncertainty Modelling
3.7.3 Conditional distributions

In some cases, values taken on by one random variable may be statistically dependent on values of
another random variable. For example, if X and Y are discrete random variables what is the probability
of X=y given that Y=y? The distribution describing this is called the conditional PMF, pX|Y(x|y), and can be
found using the multiplication rule given by Eq. (2.11)

P X  x   Y  y  p XY x, y 
p X |Y x | y   P X  x | Y  y   
PY  y   p XY xi , y 
all xi (3.88)
p  x, y 
 XY
pY  y 

In a similar way, for continuous random variables X and Y the conditional PDF of X given Y is defined
as

f XY x, y 
f X |Y x | y   (3.89)
fY  y

If X and Y are statistically independent, then pX|Y(x|y)=pX(x) or fX|Y(x|y)=fX(x) and

p XY x, y   p X x  pY  y  (3.90)

or

f XY x, y   f X x  f Y  y  (3.91)

3.7.4 Covariance and correlation

As it has been noted previously, the most basic information about the distribution of a single random
variable X is provided by its mean μX and variance Var(X)=σX2 (or coefficient of variation COV(X)= σX/μX).
In a similar way, the interdependence between random variables X and Y in their joint probability
distribution can be represented by the covariance, Cov(X,Y), which is the second moment of their joint
PDF about their means

 
Cov X , Y   E X   X Y  Y     x   y    f x, y dxdy
X Y XY (3.92)
 

It can easily be shown that for statistically independent X and Y Cov(X,Y)=0, otherwise, it can be positive
or negative. Cov(X,Y) indicates the degree of linear dependence between the two random variables.

A normalised version of the covariance, called the correlation coefficient, ρXY, is found by dividing the
covariance of X and Y by the product of their standard deviations

Safety, Risk and Reliability, Heriot-Watt University 76


3. Uncertainty Modelling
Cov X , Y 
 XY  (3.93)
 XY

It can be shown that

 1   XY  1 (3.94)

y
y

x
x (b) 0 < ρXY < 1
(a) ρXY ≈ 0
y y

x x
(c) ρXY ≈ 1 (d) -1 < ρXY < 0
y y

x x
(e) ρXY ≈ -1 (f) ρXY ≈ 0

Figure 3.6 Correlation coefficient of two random variables.

Like the covariance, the correlation coefficient shows the degree of linear dependence between two
random variables. When ρXY is close to zero, it does not mean that there is no dependence at all
between two random variables; there may be some nonlinear relationship between these variables.
Figure 3.6 illustrates the concept of correlation. It is important to note that the terms “statistically
independent” and “uncorrelated” are not synonymous. Statistically independent is a stronger
characteristic than uncorrelated. If two random variables are statistically independent, they must be
uncorrelated. However, uncorrelated random variables are not necessarily statistically independent.

Safety, Risk and Reliability, Heriot-Watt University 77


3. Uncertainty Modelling
When the correlation coefficient is estimated using observed sample values (e.g., experimental data)
it is rare to obtain values of precisely 0, 1, -1. The two random variables can be assumed to be
statistically independent if the absolute value of their correlation coefficient is less than 0.3; two
random variables can be assumed perfectly correlated if | ρXY| > 0.9.

Worked Example 3.7

The joint PDF of two random variables X and Y is defined as

 
f XY x, y   a x 2  9 y 2  4 , 0 ≤ x ≤ 3 and 0 ≤ y ≤ 2
= 0, elsewhere

(a) Determine the constant a

(b) Determine the marginal density function for X.

(c) Determine the marginal density function for Y.

(d) Are X and Y statistically independent?

(e) Determine the probability of the following events:

P(X>2| Y=1) and FXY(2, 2).

Solution

  ax  
2 3
(a) 2
 9 y 2  4 dxdy  1.0 or
0 0

3 3
 x3   y3 
0 ay  4 3  9 x  dy  0  18ay  4dy  18a 3  4 y   18  3 a  1.0
 16 
2 2
2 2

0 0

1
a
96

 96 x    
2
(b) f X x  
1 1 2
2
 9 y 2  4 dy   x 9
0
18

 96 x    
3
(c) f Y  y  
1 3 2
2
 9 y 2  4 dx   y 4
0
16

(d) f X x  f Y  y   
 1 2
  3 2
x  9    
 1 2
y 4      
x  4 y 2  9  f XY x, y 
 18   16  96

Safety, Risk and Reliability, Heriot-Watt University 78


3. Uncertainty Modelling
Thus, X and Y are statistically independent random variables.

(e) Since X and Y are statistically independent random variables f X |Y x | y   f X x 

Thus

3
1  x3 
P X  2 | Y  1    x 2  9dx     9 x   0.1481
3
1
2
18 18  3 2

   
2 2
FXY 2,2 
1

96 0
x 2  9 dx  y 2  4 dy  0.8518
0

3.7.5 Common multivariate distribution

Obviously, multivariate distributions are more complex than distributions of a single random variable
and not many of them are commonly used. One of the common multivariate distributions is a joint
normal distribution. The joint PDF of two normally distributed random variables X and Y depends on five
parameters: the means μX and μY, the standard deviations σX and σY, and the correlation coefficient ρXY

  x    2 x   X  y  Y  
  X
  2  XY
   X   XY 
f XY x, y  
1 1
exp  
2 X  Y 1   XY
2 
 2 1   XY
2
 
  y  Y 
2

     
   Y  
(3.95)

3.8 Central limit theorem


The central limit theorem says that under certain general conditions, as the number of random
variables in the sum becomes large, the distribution of the sum of random variables will approach the
normal distribution.

The statement “under certain general conditions” requires additional explanation. Basically, this
means that the random variables in the sum are mainly independent, not necessarily identically
distributed but none of these variables dominates the sum.

The important fact is that even if the number of random variables in the sum is only moderately
large, as long as no one variable dominates and the variables are not highly dependent, the distribution

Safety, Risk and Reliability, Heriot-Watt University 79


3. Uncertainty Modelling
of their sum is very close to a normal distribution. The practical importance of the theorem is that this
statement about the normal distribution can be made without any exact knowledge about the
distributions of the contributing random variables and their number. Since the random variation in
many phenomena arises from a number of additive variations, the normal distribution is often the right
choice to model uncertainty.

The central limit theorem can be extended to a product of a number of random variables. Let Y be a
product of a number of random variables Xi (i=1,…,n), which satisfy “certain general conditions” and can
only take on values greater than zero

n
Y   Xi (3.96)
i 1

If to take the natural logarithm of both sides of Eq. (3.96)

n
ln Y   ln X i (3.97)
i 1

This sum can be interpreted as the sum of random variables lnXi, thus, according the central limit
theorem the distribution of lnY approaches a normal distribution, or the distribution of Y approaches a
lognormal distribution. Thus, under certain general condition as the number of random variables in the
product becomes large, the distribution of the product of random variables will approach the lognormal
distribution. From practical point of view this means that when it can be justified that a random variable
is a product of a number of uncertain parameters, this random variable can be modelled by a lognormal
distribution.

3.9 Estimation of distribution parameters


In Section 3.4.2 the parameters of probability distributions such as a mean and a standard have been
defined theoretically, based on knowledge of the PDF (of PMF) of the distribution. In practice, the true
probability distribution is usually unknown and the parameters can only be estimated using
observations (test data). The moments of the collected numerical data (i.e., sample) provide obvious
estimates of the corresponding moments of the probability distribution. This intuitive approach to the
estimation of the parameters of a probability distribution is called the method of moments. If a sample
of n observations xi (i=1,…,n) is obtained for a random variable X then, according to this method, the
estimator mX of the true mean, μX, is the sample mean, and the estimator sX of the true standard
deviation, σX, is the sample standard deviation

Safety, Risk and Reliability, Heriot-Watt University 80


3. Uncertainty Modelling
1 n
mX   xi
n i 1
(3.98)

1 n
sX   xi  m X 2
n  1 i 1
(3.99)

Note that the denominator (n-1) appears in Eq. (3.99) instead of n; it can be shown that this is required
in order to obtain the unbiased estimate of the standard deviation.

In order to obtain the estimate rXY of the correlation coefficient ρXY of two random variables X and Y
observed data for both of them are needed. If there are n observations of X, xi (i=1,…,n) and n
observations of Y, yi (i=1,…,n) then after calculating the sample means mX and mY and standard
deviations sX and sY, the estimate of the correlation coefficient can be calculated as

1
 x i  m X  yi  mY 
rXY  i 1
(3.100)
n 1 s X sY

Exercises
Exercise 3.1

Show that Var(X) = E(X2) – μX2.

Exercise 3.2

The drainage system in a city has been designed for one in 50 years rainfall (i.e., the design rainfall
intensity can be exceeded on average once in 50 years). (a) What is the probability that the city will be
flooded in two out of 15 years? (b) What is the probability that the city will be flooded at most two out
of 15 years? (c) What is the probability of no flood in 50 years?
Ans. (a) 0.0323; (b) 0.9970; (c) 0.3642

Exercise 3.3

A steel cable has to carry a weight of 10 kN. According to available information, the strength of the
cable, R, can be modelled by a normal random variable with a mean of 25 kN and a standard deviation
of 5 kN. What is the probability that the cable will break under the weight?

Ans. 0.0013

Exercise 3.4

Safety, Risk and Reliability, Heriot-Watt University 81


3. Uncertainty Modelling
A prestressing strand consists of seven cold drawn wires spun together in helical configuration.
Assume that tensile strength of a single wire can be described by a normal distribution with mean 2019
MPa and COV = 0.04. What is the probability that the weakest wire in the strand has the tensile strength
less than 1860 MPa?
Ans. 0.1589

Exercise 3.5

The annual probability of damage to a building due to fire, p, is estimated as 0.05. Assume that the
design life of the building is 50 years.

(a) What is the probability that the building will not be damaged by fire during its design life?
Ans. 0.077

(b) What is the probability that the building will be damaged by fire for the first time exactly in the 10-
th year of its service life?
Ans. 0.0315

(c) If the insurance company requires that the maximum probability of damage to the building by fire
be limited to 0.10 during its design life, what is the maximum acceptable value of p?
Ans. 0.0021

Exercise 3.6

The compressive strength of concrete delivered by a supplier can be modelled by a lognormal


random variable with a mean of 47 N/mm2 and a COV of 0.21.

(a) Calculate the characteristic strength of the concrete, which is the lower 5% fractile, i.e., if fc denotes
the compressive strength of concrete and fck its characteristic strength P(fc ≤ fck)=0.05.
Ans. 32.7 MPa

(b) Suppose the COV of the compressive strength is reduced to 0.10 along with the reduction of the
mean value to 41 N/mm2 by introducing new quality control procedures. Calculate the
characteristic compressive strength of the concrete in this case.
Ans. 34.7 MPa

(c) By comparing the results obtained in (a) and (b), discuss whether the introduction of the new
quality control procedure is beneficial.

Exercise 3.7

For a large construction project, the contractor estimates the average rate of on-the-job accidents as
three per year. From the past experience, the contractor also estimates that the cost of an accident can

Safety, Risk and Reliability, Heriot-Watt University 82


3. Uncertainty Modelling
be modelled as a lognormal random variable with a mean of £6,000 and COV of 0.20. The costs of
accidents can be assumed to be statistically independent.
(a) What is the probability that there will be no accident in the first month of construction?
Ans. 0.78
(b) What is the probability that only one out of the first three months of construction will be without
accidents?
Ans. 0.113
(c) What is the probability that an accident will incur a loss exceeding £7,000?
Ans. 0.19
(d) What is the probability that none of the accidents in a month will cost more than £7,000?
Ans. 0.9536

Exercise 3.8

A steel cable consists of eight high-strength steel strands. The strength of each strand can be
modelled by a lognormal random variable with a mean of 50 kN and COV of 0.10. What is the probability
that the weakest strand will have strength less than 40 kN?
Ans. 0.11

Exercise 3.9

The annual maximum stage height in a river channel is modelled using a Gumbel distribution with a
mean of 9 m and COV of 0.10. The stage height at which flooding will occur is 12 m. What is the
probability that the annual maximum stage height will exceed this level?
Ans. 0.0077

Exercise 3.10

To estimate the settlement due to consolidation of a homogeneous saturated soil deposit, the
coefficient of consolidation, cv, needs to be determined. It can be calculated as

k
cv 
mv  w

where k is the hydraulic conductivity (or coefficient of permeability), mv the coefficient of volume
compressibility, and γw the unit weight of water which has a constant value of 9.81 kN/m 3. Assume that
k and mv are statistically independent lognormal random variables with means of 1.3×10 -7 m/min and
1.1×10-3 m2/kN, respectively. Both have COV of 0.10.

(a) Determine the distribution of cv and the parameters of the distribution.

Safety, Risk and Reliability, Heriot-Watt University 83


3. Uncertainty Modelling
Ans. Lognormal distribution with λ=-11.33 and ξ=0.1411

(b) What is the probability that cv is greater than 1.2×10-5 m2/min?


Ans. 0.5017

Revision Questions
3.1 What are the types of uncertainty distinguished in engineering applications?

3.2 What is random variable? What types of random variables do you know?

3.3 What functions are used to describe a probability distribution?

3.4 How are the mean, variance, standard deviation and coefficient of variation of a random variable
defined?

3.5 What types of discrete and continuous probability distributions do you know?

3.6 What distributions are used for modelling extremes?

3.7 What distributions are used to describe the joint behaviour of two or more random variables?

3.8 What parameters do represent the interdependence between random variables?

3.9 Formulate the central limit theorem.

3.10 How can the distribution parameters be estimated?

SUMMARY

 Types of uncertainty arising in structural engineering have been described.

 The notions of a random variable and a probability distribution have been explained; functions and
parameters used to characterise them have been defined.

 Commonly used types of probability distributions, including extreme value distributions, have been
described.

 Functions and parameters used to describe the joint behaviour of two or more random variables
have been presented.

 The central limit theorem has been explained.

 The method of moments for estimation of the parameters of a probability distribution has been
briefly described.

Safety, Risk and Reliability, Heriot-Watt University 84


3. Uncertainty Modelling
4 FUNDAMENTALS OF STRUCTURAL RELIABILITY

4.1 Definition of failure and limit states


The main aim of structural reliability analysis is to evaluate the probability of failure of a structural
system or its member. In order to do this, the event “failure” needs to be defined. In structural
engineering failure is usually defined based on the concept of a limit state, which represents a boundary
between desired and undesired performance of a structural system or its member. Three types of limit
states are usually considered:

1. Ultimate limit states (ULSs) are mainly associated with the loss of load-bearing capacity. Examples of
failures of this type include:

- exceeding the resistance of a structural component in bending, shear, compression or tension;

- loss of the overall stability;

- bucking;

- weld rupture.

2. Serviceability limit states (SLSs) correspond to conditions beyond which specified service
requirements for a structural system or its member are no longer met, i.e., the structural system
cannot perform properly its required functions. Examples of failures of this type include:

- excessive deflections;

- excessive vibrations;

- permanent deformations, which accumulate when load-induced stresses in structural members


exceed an elastic limit;

- excessive cracking (usually in reinforced concrete structures).

3. Fatigue limit state (FLS) is associated with the loss of strength under repeated loads. Sometimes this
limit state is included in ULSs.

Thus, failure can be defined as violation of one of these limit states. As should be clear from the
above discussion, failure defined in such a way does not necessarily mean collapse of a structural
system. In order to carry out an analysis, a limit state needs to be described quantitatively, which is
done with the help of the so-called limit state (or performance) function, G. The limit state function is
defined in the following way:

G < 0 – undesired performance, the structure is unsafe

Safety, Risk and Reliability, Heriot-Watt University 85


4. Fundamentals of Structural Reliability
G = 0 – a limit state, i.e., the boundary between safe and unsafe performance (4.1)

G > 0 – desired performance, the structure is safe

In the simplest form the limit state function can expressed as

GCD (4.2)

where C denotes the capacity of a structure (or its member) (e.g., resistance) and D is demand (e.g., load
effect).

4.2 Deterministic and semi-probabilistic approaches to structural


safety
In the deterministic approach to safety the capacity or capability (C) of a structure to withstand a
load always exceeds the load effect or demand (D) placed upon it by a significant margin usually
expressed via the so-called safety factor

Capacity(C ) Strength
Safety factor =  (4.3)
Demand( D) Load

This is illustrated in Figure 4.1. Here the line at position ‘C’ indicates the magnitude of the strength or
‘capacity’. This can be measured, for example, in the force required to cause collapse of the structure, or
the stress in the structure that would cause failure of the structure material. The line at ‘D’ represents
the expected lifetime maximum ‘demand’ or load effect on the structure; again this can be measured as
a force or a stress (the ‘maximum working stress’). As long as the magnitude of ‘C’ is greater than that of
‘D’ then there is a margin (or factor) of safety.

0 D C force
Figure 4.1 Schematic diagram illustrating the deterministic approach to structural safety.

The factor of safety or safety factor is usually expressed as a stress factor or a load factor in codes of
practice where

Safety, Risk and Reliability, Heriot-Watt University 86


4. Fundamentals of Structural Reliability
Yield stress Load to cause collapse
Stress factor = , and Load factor =
Max. working stress Max. working load

However, the factor of safety can also be based on any measure of structural performance. So for
fatigue and structures limited by deflection we use respectively

Time to failure Allowable deflection


Safety factor = and Safety factor =
Design life time Max. working deflection

Early safety factors for cast iron (based on stress) were typically around 4. For steel structures factors of
safety may range from around 1.1 up to 4 or more depending on the understanding of material, the
understanding of the maximum loading involved and the consequences of any failure.

The principal limitation of this deterministic approach to structural safety is that it does not admit
the possibility of failure, but failures do occur. If the factor of safety is reduced by 30% by how much has
the risk of failure increased? On the other hand if the factor of safety is increased by 30% by how much
has the risk reduced? The model has no answer other than the risk of failure will go up in the first case
and down in the second.

At the design stage there is usually uncertainty about the ultimate strength a structure will actually
have. For example, suppose we get ten fabricators each to build a small wooden bridge all to the same
design and then take each bridge in turn to a structures laboratory and load it up until it collapses. We
would find the strength in each case to be slightly different and if we plotted the strengths we would get
a histogram. For these measurements we could estimate the mean strength μC and its standard
deviation σC. When such bridges are put into service the biggest loading to which each will be subjected
during its lifetime will be different. Again if we knew these maximum lifetime loads we could estimate
the mean maximum load μD for all these bridges and its standard deviation σD, and plot the loads in the
form of a histogram or represent them by some probability density function.

This uncertainty in the strength (capacity) and loading (demand) is acknowledged in the partial safety
factor approach. This approach associates characteristic values DK and CK with Demand and Capacity -
usually representing the estimated 95% fractiles - upper and lower respectively as depicted in Figure 4.2.
Here, rather than just taking the mean or most likely values, the uncertainties in both demand and
capacity are recognised.

Safety, Risk and Reliability, Heriot-Watt University 87


4. Fundamentals of Structural Reliability
Capacity
Demand

DK CK
Figure 4.2 Illustration of the characteristic demand (load) and characteristic capacity (strength)
used in the partial safety factor approach.

If the distributions for the demand and capacity are normal distributions then

DK = µD + 1.645D and CK = µC - 1.645C

where 1.645 is the factor for 95% of the area under normal distribution curve.

Now instead of using a single safety factor of safety in this approaches a number of partial safety
factors are used. These partial safety factors are determined by experts on Code of Practice committees
who consider all the sources of uncertainty associated with the structure and its loading and the
consequences of any failures.

The characteristic load DK is then multiplied by a series of partial safety factors (usually  1.0) to
account for uncertainty in the loading estimation. For example, a structure may be subject to ice loading
but the thickness and weight of the ice may be difficult to predict at the design stage, so a safety margin
needs to be introduced via a partial safety factor to allow for this. Similar there are partial safety factors
for the strength uncertainties (usually  1.0). For example, if the quality control that can be exercised
during construction is limited then the structure may have lower strength than anticipated by the
designer. Dividing the characteristic strength by a partial safety > 1.0 duly allows for this. If, for example,
the material behaviour was not well understood then another partial safety factor would allow for this.
All aspects of uncertainty in the strength and loading are considered and a partial factor assigned to
each. To ensure safety the approach then requires that

CK/(  m1 m 2 ...... )  DK  f 1 f 2 ...... (4.4)

Safety, Risk and Reliability, Heriot-Watt University 88


4. Fundamentals of Structural Reliability
where m and f are the partial safety factors for strength and loading variables, respectively. If this
inequality is not satisfied then the characteristic design strength CK must be increased. This is known as a
semi-probabilistic approach and it is the basis of many design codes.

4.3 Probabilistic approach to structural safety


The probabilistic approach to structural safety assumes that all aspects of uncertainty concerning the
demand and capacity can be assessed explicitly. In the simplest case D and C can be modelled as two
random variables represented by probability distributions fD(D) and fC(C). The limit state function G is
given by Eq. (4.2). The probability of failure, Pf, can then be expressed as

Pf  PG  0  PC  D  0 (4.5)

Eq. (4.2) can be rewritten in a different form. A structure fails when the demand exceeds its capacity
(or load exceeds its resistance). If the capacity of a structure C equals a specific value c, the structure
fails when D ≥ c. However, D is also a random variable, thus the probability of failure can be expressed
as the sum of all possible intersections of two events: (C=c) and (D ≥ c)

Pf   PC  c   D  c    PD  C | C  c PC  c  (4.6)

If C and D are continuous random variables, the summation is replaced by integration. The probability

PD  C | C  c   1  PD  C | C  c   1  FD c  (4.7)

In the limit

PC  c   f C c dc (4.8)

Substituting Eqs. (4.7) and (4.8) into Eq. (4.6) and replacing the summation by integration lead to

 
Pf   1  F c  f c dc  1   F c  f c dc

D C

D C (4.9)

In a similar way it can also be shown that the probability of failure can be expressed as


Pf   F d  f d dd

C D (4.10)

Generally, the capacity C may be a function of material properties and structural dimensions, while
the demand D a function of applied loads, materials densities and also structural dimensions, and each
of these parameters may be a random variable by itself. Let the vector X ={X1,X2,…} represent all the
random variables affecting C and D and, subsequently, the limit state function G in a particular problem;

Safety, Risk and Reliability, Heriot-Watt University 89


4. Fundamentals of Structural Reliability
such random variables are usually called the basic random variables for this problem. The joint
probability density function of the vector X of the basic random variables can be denoted as fX(x). The
probability of failure can then be expressed as

Pf  PGX  0     f xdx
X (4.11)
G  X 0

Note that the capacity C and demand D do not longer appear in the formulation of Pf, generally they
may be implicit in X. If the basic random variables are statistically independent then their joint
probability density function can be found as

f X x   f X i xi   f X1
x1  f X x2  f X x3 
2 3
(4.12)
i

where fXi(xi) is the marginal PDF of the i-th basic random variable. The multidimensional integral over the
failure domain G(X≤0) in Eq. (4.11) cannot be found analytically except of a very few special cases, some
of which will be considered further. Generally, the integral is evaluated numerically using Monte Carlo
simulation or specially developed approximate methods.

4.4 Special case: C and D are normal random variables


One of the special cases, when the probability of failure can easily be evaluated, is when C and D are
both treated as independent normal random variables. The limit state function G=C-D is then also a
normal random variable with the following mean and variance (see the property of the sum of
independent normal random variables in Section 3.5.2.1)

G  C   D (4.13a)

 G2   C2   D2 (4.13b)

where μC and μD are means and σC and σD standard deviations of C and D, respectively. The probability of
failure expressed by Eq. (4.5) is then becomes

 0  G    
Pf  PG  0  FG 0       G      (4.14)
 G   G 

where β=μG/σG is called the safety (or reliability) index and Φ(.) is the CDF of the standard normal
distribution. The safety index can also be expressed via the means and variances of the capacity and
demand using Eq. (4.13)

C   D
 (4.15)
 C2   D2

Safety, Risk and Reliability, Heriot-Watt University 90


4. Fundamentals of Structural Reliability
The safety (or reliability) index is a very important parameter in structural reliability, which is very often
used as a measure of structural reliability instead of the probability of failure. As can be seen from Eq.
(4.14) there is a direct relationship between β and Pf , which is also illustrated in Figure 4.3

Pf     (4.16)
5
Pf β

0.5 0 4

Reliability index, 
0.1 1.28
3
0.05 1.64

0.01 2.33 2

1×10-3 3.09
1
-4
1×10 3.72

1×10-5 4.26 0
-6 -5 -4 -3 -2 -1
10 10 10 10 10 10 0.5
-6
1×10 4.75
Probability of failure, P
f

Figure 4.3 The relationship between the probability of failure and the safety index.

In the deterministic approach safety of a structure is provided using the safety factor, which is the
ratio of the capacity to the demand (see Eq. (4.3)). It has not been specified there what values of C and D
are used. If C and D are represented by their mean values then their ratio is referred to as the central
safety factor, λ0

C
0  (4.17)
D

When C and D are independent normal random variables a relationship between λ0 and Pf (or β) can
easily be established. From Eq. (4.14)

        C   D 
Pf    G     C D      (4.18)
 G    C   D2
2   COV (C )  C2  COV 2 D  D2
2 
   

and dividing through by μD

 0  1 
Pf         (4.19)
 COV (C )0  COV D  
2 2 2

Safety, Risk and Reliability, Heriot-Watt University 91


4. Fundamentals of Structural Reliability
From the last equation it can also be shown that

0 

1   COV 2 C   COV 2 D    2 COV 2 C COV 2 D  
12

(4.20)
1   2 COV 2 C 

Eq. (4.20) shows that the central safety factor depends on the variability of the capacity and demand
(i.e., COV(C) and COV(D)) and in order to ensure the same level of safety (i.e., the same target values of
Pf or β) different values of λ0 should be used depending on COV(C) and COV(D). This demonstrates the
deficiency of the deterministic approach, which is unable to take this into account.

Eq. (4.15) for the safety index can be expended for a more general case when the limit state function
is a linear function of n independent normal random variables, Xi (i=1,…,n)

n
GX  a0   ai X i (4.21)
i 1

where ai (i=0,…,n) are constants. The limit state function is then also a normal random variable with
mean

 n
 n
 G  EGX  E a0   ai X i   a0   ai  X (4.22)
 
i
i 1 i 1

and variance

 G2  Var GX  Var a0   ai X i    ai X 


 n
 n
2
(4.23)
 
i
i 1 i 1

The safety index is then equal to

n
a0   ai  X i
G
  i 1
(4.24)
G
 a  
n
2
i Xi
i 1

Worked Example 4.1

Some nominally identical steel cables have been tested and found to have mean tensile strength of
8kN at failure with a standard deviation of 0.5 kN. A similar cable is used in rigging of a boat where it will
be subjected to an estimated maximum load of 4 kN with an estimated standard deviation of 1.0 kN.
What is the probability it will fail when it comes under load? Assume that the cable tensile strength and
the load on it can be modelled as independent normal random variables.

Safety, Risk and Reliability, Heriot-Watt University 92


4. Fundamentals of Structural Reliability
Solution

Both strength and loading are normally distributed. In this case the limit state function is simply

GC, D  C  D

where C is the strength of the cable and D is the load on it.

Hence μG  μC  μD i.e.  G  8  4  4kN ,

and  G2   C2   D2 i.e.  G2  0.52  1.02  1.25kN 2 .

Thus  G  1.25  1.118kN

Pf   3.578  0.000174


4
and   3.578 so
1.118

The cable would have a 0.0174% chance of failing when loaded.

In this example the central safety factor λ0 = 8/4 = 2.

Worked Example 4.2

A simply supported timber beam of length 5 m is loaded with a central load Q having mean of 3 kN
and standard deviation of 1 kN. The bending strength of similar beams has been found to have a mean
strength of 10 kNm with a coefficient of variation (COV) of 0.15. Estimate the probability of failure of the
beam. Assume that the load and the beam bending strength can be modelled as independent normal
random variables.

Solution

Assume that the beam self-weight and any variation in the beam length can be neglected. From basic
structural theory, the maximum applied bending moment (the demand D) at midspan of the beam is

QL
D
4

Since L=5 m

2
5
 D   Q   3  3.75 kNm and  D2     Q2   1  1.56 kNm2
5 5 25
4 4 4 16

The mean and variance of the beam bending strength (capacity C) are

 C  10.0 kNm and  C2  COV (C )C 2  0.15  102  2.25 kNm2

Safety, Risk and Reliability, Heriot-Watt University 93


4. Fundamentals of Structural Reliability
The mean and variance of the limit state function G are then

G  C   D  10  3.75  6.25 kNm

 G2   C2   D2  2.25  1.56  3.81 kNm2

Thus

G
 3.20 and Pf       3.20  7  10 4 .
6.25
 
G 3.81

4. Return period
The design wind speed, rainfall or flood level at a particular location is usually expressed in terms of
return period. The return period may be defined as follows. For independent samples from a population
(i.e., Bernoulli trials), the probability of the first occurrence of an event at the trial t is given by the
geometric distribution

PT  t   p1  p 
t 1
(4.25)

where p is the probability of occurrence of the event at each trial. The recurrence time, the time
between two consecutive occurrences of the same event, should follow the same geometric distribution
given by Eq. (4.25). The mean recurrence time, also known as the return period, can then be calculated
using Eq. (3.26) as


return period T  E T    tp 1  p 
t 1
(4.26)
t 1

It can be shown that the infinite sum in Eq. (4.47) equals 1/p, i.e.,

1
return period T  (4.27)
p

Eq. (4.27) shows that if the design wind speed corresponds to a 50-year return period, then the
annual probability that the design wind speed will be exceeded is 1/50=0.02. Note that the probability
the design wind speed will not be exceeded in 50 years is (1-0.02)50=0.364.

Exercises
Exercise 4.1

A simply supported beam of length 20 m is loaded by uniformly distributed load of intensity w and a
central concentrated load Q. Assume that the loads w and Q, and the beam bending strength (capacity

Safety, Risk and Reliability, Heriot-Watt University 94


4. Fundamentals of Structural Reliability
C) can be modelled as independent normal random variables. Calculate the safety index and probability
of the beam failure for following distribution parameters of w, Q and C:

μw = 1 kN/m; COV(w) = 0.10

μQ = 12 kN; COV(Q) = 0.15

μC = 200 kNm; COV(C) = 0.12

Ans. β = 3.45; Pf = 2.8×10-4

Exercise 4.2

If the capacity C and demand D are independent lognormal random variables show that

  C 1 2 
 ln 

 
1  COV 2 D  1  COV 2 C    
 
Pf         D
   
ln 1  COV C  1  COV D 
2 2 12


 
 
 

Show that for COV(C) < 0.3 and COV(D) < 0.3 this simplifies to

   
 ln  C  
  D  
Pf        
2

 COV C   COV ( D) 
2 1 2

 
 

Exercise 4.3

There is a cable supporting a weight. The tensile strength of the cable has a mean and standard
deviation of 120 kN and 18 kN, respectively. The weight has a mean of 50 kN and standard deviation of
12 kN. Calculate the probability of failure of the cable assuming:

(a) The cable strength and weight can be modelled as independent normal random variables.
Ans. 6×10-4

(b) The cable strength and weight can be modelled as independent lognormal random variables.
Ans. 7.9×10-4

Exercise 4.4

Consider a simply supported beam which is 12 m long. The beam is subjected to uniformly
distributed dead, live and wind loads. The distribution parameters of the loads are shown in the table
below. The bending strength of the beam has a mean of 100 kNm and COV of 0.13.

Safety, Risk and Reliability, Heriot-Watt University 95


4. Fundamentals of Structural Reliability
Calculate the probability of failure for the beam. Assume that all the random variables are
independent and normally distributed.

Table: Distribution parameters of loads


Load Mean, kN/m Standard deviation, kN/m
Dead 0.95 0.1
Live 1.5 0.2
Wind 0.6 0.12
Ans. β = 3.27; Pf = 5.38×10-4

Revision Questions
4.1 Explain the notion of a limit state. What types of limit states are usually considered?

4.2 Describe the deterministic approach to structural safety. What are its limitations?

4.3 What are characteristic values and how are they used in the partial safety factor approach? What
is a partial safety factor?

4.4 Explain the main ideas of a probabilistic approach to structural safety. What is a limit state
function?

4.5 What is the safety or reliability index and how is it linked to the safety factor?

4.6 Explain the notion of a return period.

SUMMARY

 The traditional deterministic approach to structural safety is described and its limitations are
discussed

 The partial safety factor approach is described

 The concepts of structural reliability theory are explained and the technical terms demand,
capacity, limit state function and safety index are defined; illustrating examples are provided

 The relationship between the safety index and probability of failure is described

 The concept of a return period is explained

Safety, Risk and Reliability, Heriot-Watt University 96


4. Fundamentals of Structural Reliability
5 FIRST ORDER METHODS OF RELIABILITY ANALYSIS

5.1 First-Order Second-Moment (FOSM) method

5.1.1 Introduction

This section describes the First Order Second Moment (FOSM) method for estimating structural
reliability. The FOSM equation for the safety index is derived using the Taylor series. Examples are then
given of the use of the FOSM method. It is shown that the output from the FOSM method depends on
the way the limit state function is expressed and the problem of the ‘lack of invariance’ is discussed.
Alternative methods, which do not suffer this problem, are then presented.

The First Order Second Moment (FOSM) method was the first structural reliability method to be used
and its extension, the First Order Reliability Method (FORM), is one of the most widely used methods in
structural reliability today. The FOSM method serves to illustrate many of the principal features of
structural reliability calculations and is also relatively simple. The name ‘first order’ comes from the fact
that the theory is based on a linear expansion using just the first derivative terms in a Taylor series. All
the higher order terms are neglected. The ‘second moment’ comes from the fact that each variable is
described only by its mean value and variance. The mean is the first moment of the area under the
probability density curve about the ordinate, and the variance is the second moment of area about the
mean. As will be explained, the FOSM method can give exact answers to certain types of structural
reliability problems but for most problems it does not provide an exact answer. However, even in these
cases it can usually give a useful approximate answer which may be all that is necessary in a preliminary
quantitative risk assessment.

5.1.2 The FOSM method

In the previous chapter it has been described how the safety (reliability) index β can be calculated
(see Eq. (4.24)), when the limit state function is a linear function of a number of independent normal
random variables

GX  a0  a1 X 1  a2 X 2  a3 X 3 .......  an X n (5.1)

The FOSM method extends this result for the case when the limit state function G(X) is a nonlinear
function of normal random variables. This is achieved by linearization of the limit state function using a
Taylor series expansion and neglecting the 2 nd and higher order derivatives

Safety, Risk and Reliability, Heriot-Watt University 97


5. First Order Methods of Reliability Analysis
 G 
   
n
GX  G x1* , x2* ,, xn*     X i  xi* (5.2)
i 1  X i  x*

where x*={x1*, x2*,..., xn*} is the point about which the expansion is performed. (∂G/∂Xi) denotes a partial
derivative of the limit state function G with respect to the random variable Xi, which in general may also
be a function of the random variables; the subscript x* means the random variables in this function
should be replaced by their values at the expansion point. In principle, x* can be any point, but in the
FOSM method the linearization is carried out about the mean values of the random variables, μX. Thus,
Eq. (5.2) becomes

 G 
   
n
GX  G  X1 ,  X 2 ,,  X n     X i   X i (5.3)
i 1  X i μX

The mean value of this linear approximation of the limit state function is

  G  
G  EGX  E G  X ,  X ,,  X      
n
 X i   X i (5.4)
 i 1  X i μX 
1 2 n

Since E(xi-μXi)=0, as by definition E(xi)=μXi, the second term on the right hand side of Eq. (5.4) disappears
so that

 G  G X ,  X ,,  X
1 2 n
 (5.5)

The variance of G(X) given be Eq. (5.3) is

 n 
2


  E GX    G 
2 2
 
 E  
 G 

 X i  X i 

 (5.6)
 i 1  X i
G
 μX  
 

Consider now the expansion of the right hand side (r.h.s) of Eq. (5.6). There will be two types of
terms: the first involving squares and the second involving cross-products

 n 
2
 n 
 XG  X 
2
 G    E   G  X    G 
    
n
E     Xi  Xi 
     X i   X i  X j 
2

 i 1  X i     i 1  X i  μ X
i Xi
i , j 1,i  j  X i  μ
j

 μX    X  j μX 
(5.7)

Eq. (5.7) can be rewritten as

     G   G 
  
2
n
 G  n
r.h.s     E X i   X i     E Xi  X X j  X
2

i 1       (5.8)
X i μX i , j 1,i  j  X i μX  X i j
j μ
X

Safety, Risk and Reliability, Heriot-Watt University 98


5. First Order Methods of Reliability Analysis
By definition E[(Xi-μXi)]=σXi2 and E[(Xi-μXi)(Xj-μXj)]=Cov(Xi,Xj).

If the random variables are independent the covariance Cov(Xi,Xj)=0, so that the variance becomes

2
n G  
   
2
  X i  (5.9)
i 1  X i
G
 μX 

The safety index can then be estimated as

 G G  X ,  X ,,  X 
  1 2 n
(5.10)
G
 ai X 
n
2
i
i 1

where

 G 
ai    (5.11)
 X i μX

If the random variables are correlated then Cov(Xi,Xj)≠0 and can be expressed as (see Eq.
(3.93))

CovX i , X j    X i X j  X i  X j (5.12)

The variance of the limit state function is then

2
n n  G   G  
 
2
 
  

 X X  X  X 

(5.13)
j 1  X i  μ X  X j 
G i j i j
i 1
 μX 

The safety index can now be calculated as

G 
G  X 1 ,  X 2 , ,  X n 
  (5.14)
G
  a a 
n n

i j X X  X  X
i j i j
2

i 1 j 1

where

 G   G 
ai    and aj    (5.15)
 X 
 X i μX  j μX

After the safety index has been found, the probability of failure is estimated from Pf=Φ(-β).
Note that Eqs. (5.10) and (5.14) are exactly true only when all the random variables are normally
distributed and the limit state function is linear; when the limit state function is nonlinear the

Safety, Risk and Reliability, Heriot-Watt University 99


5. First Order Methods of Reliability Analysis
FOSM method provides just approximate estimates of the safety index and the probability of
failure.

Worked Example 5.1

A gantry crane shown in Figure 5.1 has a lifting beam spanning 4 m, with a mean plastic section
modulus of 110–4 m3 and a standard deviation of 1.010–5 m3. The beam is made of steel with a
mean yield stress 270 N/mm2 and a standard deviation of 27 N/mm2. The maximum load is 22 kN and
the standard deviation is 2.5 kN. Calculate the safety index and the probability of failure.

kg

WC

Figure 5.1 Load WC causes a freestanding gantry crane to collapse when a plastic hinge develops at the
loading point in midspan.
Solution

The capacity (bending strength) of the lifting beam is C=Z×fy, where Z denotes the plastic section
moment and fy the yield stress of steel. The demand (maximum bending moment) is D=Wc×l/4. The limit
state function can then be written as

Wc l
GX  Zf y 
4.0
 Zf y  Wc  Zf y  1  Wc (kNm)
4 4

Using Eq. (5.5) and substituting the mean values of Z, fy and Wc we find

G   Z  f  1 W  1.5  10 4  270  103  1 22  18.5 kNm


y c

The variance of the limit state function is found using Eq. (5.9)

2
 G 
2
 
 G    2f   G
2

    Z     W2 c   2f y  Z2   Z2  2f y  12  W2 c
2 2
  W
 Z   X  f y   X
G
 c  X
y


 270  10 3  1.0  10 5   1.5  10
2 4
 27  10 3   1 2.5
2 2
 29.9425 kNm
2

Safety, Risk and Reliability, Heriot-Watt University 100


5. First Order Methods of Reliability Analysis
Let’s consider in more detail how the above result has been obtained. For example, the partial
derivative of G with respect to Z, (∂G/∂Z) = fy , which is a very simple function of the random variable fy;
substituting this random variable by its mean value (i.e., its value at the expansion point) gives μfy. In a
similar way

 G   G 
   Z μ   Z ;    1μ X  1
 f 
 Wc μX
X
 y μX

Hence  G  29.9425  5.472 kNm

 G 18.5
The safety index is then     3.38
 G 5.472

And the corresponding probability of failure is found using Appendix 2

Pf       3.38  3.62  10 4 .

5.1.3 The use of FOSM and the ‘lack of invariance’ problem

The FOSM method is easy to use and in order to use it we do not actually need any knowledge about
the distribution of the random variables, apart from their mean values and standard deviations. Using it
we assume that the random variables are normally distributed. However, there is an invariance
problem: the value of the safety index obtained with the help of the FOSM method depends on a
particular formulation of the limit state function. The invariance problem is best illustrated by examples.

Worked Example 5.2

A light hoist is designed with a cable that has a mean strength of 12 kN with a standard deviation of
2.4 kN. The hoists are all marked "maximum load – 500 kg" but experience shows this is frequently
ignored and the estimated maximum load is 6 kN (about 600kg) with a standard deviation of 0.6 kN.
Estimate the reliability of the hoists in terms of the safety index β.

Solution

1) The limit state function can be written as

GX  C  D

The mean of the limit state function is

G  C   D  12  6  6 kN

Safety, Risk and Reliability, Heriot-Watt University 101


5. First Order Methods of Reliability Analysis
and the variance

 G2   C2   D2  2.4 2  0.6 2  6.12 (kN)2

Therefore  G  6.12  2.47kN

6
Hence the safety index    2. 429
2. 47

which gives the estimated probability of failure as

Pf   2.429  7.55  10 3

2) The limit state function could also have been written as G X  


C
1
D

The mean value of G is then

C 12
G  1   1  1. 0
D 6

Using the FOSM method

2 2
 G   G   1    
2 2 2 2
1   12 
     C2     D2     C2    C2   D2    2.4     2  0.6 
2

 C  X  D  X  D   D 
G
6   6 
 0.20
 G  0.2  0.447

1.0
Thus    2.236
0.447

Pf   2.236  1.27  10 2 , which is 68% higher than the value above.

This example demonstrates the problem of "lack of invariance", which arises because the limit state
function G(X) in the second case is no longer linear. The first answer is the correct one. However, in
many cases the limit state function cannot be expressed as a linear function and there is no indication
which of the possible formulations of the limit state functions provides a more accurate result. This is
illustrated by the following example.

Worked Example 5.3

In Worked Example 5.1 the probability of failure of the gantry crane was calculated by the FOSM
method using the strength formulation, i.e., the limit state function was expressed in terms of the

Safety, Risk and Reliability, Heriot-Watt University 102


5. First Order Methods of Reliability Analysis
bending strength of the lifting beam. Solve the problem again by expressing the limit state function in
terms of stress.

Solution

The limit state function written in terms of stress is

Wc l 4.0 Wc W
GX  f y   fy   f y  1  c (kN/m2)
4Z 4 Z Z

From Eq. (5.5)

W 22
G   f  1 c
 270  10 3  1   123.3  10 3 kN/m 2
y
Z 1.5  10 4

and using Eq. (5.9)

2
 G  
2 2 2
   2  1  2
 G    2f   G
2

    Z     W2 c   W2c   Z  12   2f y      Wc
2 2
  W 
 Z   X  f y   X  Z  z 
G
 c  X 
y

2
 
2

 

22
4

 10 5   1  27  10 3
 2 

1
4


 2.5   1102.38  10 6 kN/m 2 2

 1.5  10   1.5  10 

Thus

 G  1102.38  10 6  33.20  10 3 kN/m 2

 G 123.3  10 3
The safety index is then     3.71
 G 33.20  10 3

The corresponding probability of failure is Pf       3.71  1.04  10 4 , which is more than

three times lower than that in Worked Example 5.1. The FOSM method does not provide any indication
which of the two results is more correct.

Obviously, changing the formulation of the limit state function does not change the actual border
between the safe and failure domains and if to calculate the probability of failure by direct integration of
Eq. (4.11) the same probability of failure should be obtained for both formulations. Unfortunately, the
calculation of the probability of failure by direct integration of Eq. (4.11) may be very complicated even
in simple cases and in many cases even impossible. The problem of "lack of invariance" is resolved using
another definition of the safety (reliability) index proposed by Hasofer and Lind (1974). This definition of
the safety index and the corresponding method for its calculation will be discussed further in the lecture
notes.

Safety, Risk and Reliability, Heriot-Watt University 103


5. First Order Methods of Reliability Analysis
5.1.4 Designing to achieve a given reliability using FOSM

It may be necessary to design a structural element to achieve a given reliability index or probability of
failure. This can be achieved by using either Eq. (5.10) or Eq. (5.14) with required reliability index and
solving it to find the mean value of the parameter that is to be selected. The following example
illustrates the procedure.

Worked Example 5.4

A solid circular cross section tie-bar is to be designed to carry a mean maximum load of 5 kN with a
standard deviation of 1 kN without yielding. The tie bar is be made of steel with a mean yield stress of
270 N/mm2 with COV of 0.10. The diameter of steel rod to be used has a standard deviation of 0.25 mm.
The target safety index βT = 3.5. Find the required mean diameter of the rod.

Solution

The limit state function can be written as GX  A  f y  W

D 2
where A is the cross section area of the rod A  .
4

Hence we can write GX  0.25D 2  f y  W

But this is not a convenient form for the limit state function in this case. For ease of calculation we need
the limit state function to be linear in D, and so we manipulate as follows.

GX   D 2 
4W
f y

This separates D from the other terms. We can now rewrite this as

GX  D 
4W
 D  1.13W 0.5 f y0.5
f y

which still satisfies the rules for a limit state function. Hence

 G   D  1.13W 0.5  fy0.5



  D  1.13  (5) 0.5  270  10 3 
0.5
  D  4.863  10 3 (m)

The standard deviation of fy: σfy=0.1×270×103=27×103 kN/m2.

The variance of the limit state function

Safety, Risk and Reliability, Heriot-Watt University 104


5. First Order Methods of Reliability Analysis
2
    
2
 
  2f  12   D2    1.13  0.5    W2
2 2
 G   G  G
 G2    D    W  
2 2

 W   X   W  f y 
 D   X  f y   X
y

 
2
 1.13  0.5    
2
1.13  0.5

 W  2

  f y  1  2.5  10
4

2
    1
  fy
3
  5  270  10 3 
 
2
 
 1.13  0.5  5


 27  10 3   35.81  10 8 m 2  
 
 270  10 3 3
 

and its standard deviation  G  35.81 10 8  5.984  10 4 (m) .

 D  4.863  10 3
Hence  T  3.5  and so μD= 6.957×10-3 m ≈ 7 mm.
5.984  10 4

Note that this result will only be approximate as the limit state function was nonlinear and the FOSM
method in its current formulation suffers from “the lack of invariance. More accurate analysis can be
done using the definition of the safety index proposed by Hasofer and Lind (1974), which will be
explained in the next section.

5.2 Hasofer-Lind safety index and AFOSM method

5.2.1 Hasofer-Lind safety index

As has been shown in the previous section, the original definition of the safety index (see Eq. (4.14))
and the corresponding FOSM method lead to the ‘lack of invariance’ problem, when the limit state
function is nonlinear. Hasofer and Lind (1974) proposed a new definition of the safety (reliability) index,
which did not exhibit the invariance problem. In order to define the new index, the basic random
variables X={X1,X2,…, Xn} are transformed into the standard (or reduced) random variables Y={Y1,Y2,…,
Yn}. If X are independent normal random variables, the transformation is simple

X i   Xi
Yi  (5.16)
X i

i.e., the standard (or reduced) variables Y are merely the standard normal variables with zero mean and
unit standard deviation (see Eq. (3.45)). The original limit state function, G(X), is also transformed
accordingly to the reduced limit state function g(Y). The coordinate system x corresponding to X is called
the original coordinate system, while the coordinate system y is called the reduced or transformed
coordinate system.

Safety, Risk and Reliability, Heriot-Watt University 105


5. First Order Methods of Reliability Analysis
Hasofer and Lind (1974) then defined the safety (reliability) index β as the minimum distance from
the origin of the transformed coordinate system y to the limit state surface g(y)=0. Thus, a value of β can
be found by solving the following constrained optimisation problem

12
 n 
minimise     yi2  
 yT  y 12
subject to g y   0 (5.17)
 i 1 

The minimum distance point on the limit state surface, y*, is called the design (or checking) point. The
vector y* represents the values of the reduced random variables at the design point. The values of the
original random variables at the design point are denoted as x*.

Let’s consider a simple case, when the limit state function is a linear function of two independent
normal random variables X1(≡C) and X2(≡D):

GX  X 1  X 2 (5.18)

The boundary between the failure and safe regions, i.e., G(X)=0, in the original coordinate system is
shown in Figure 5.2a. The random variables X1 and X2 can be expressed via the reduced random
variables Y1 and Y2 using Eq. (5.16) as

X 1   X1   X1 Y1 and X 2   X 2   X 2 Y2 (5.19)

To obtain a formula for the reduced limit state function g(Y) substitute Eq. (5.19) into Eq. (5.18) so that

g Y   X1   X1 Y1   X 2   X 2 Y2 (5.20)

The boundary between the failure and safe regions, i.e., the limit state surface (line) g(Y)=0, is shown in
the reduced coordinate system in Figure 5.2b. As can be seen from this figure, if the limit state surface is
closer to the origin of the reduced coordinate system, the failure region is larger and vice versa. Thus,
the location of the limit state surface in the reduced coordinate system with regard to the system origin,
which is characterised by the minimum distance (i.e., the safety index β), is directly related to the
probability of failure.

Safety, Risk and Reliability, Heriot-Watt University 106


5. First Order Methods of Reliability Analysis
X2 Y2
Failure region
G(X)=X1-X2=0
g(Y)<0   X 1   x2 
Design point, x*  0, 
x2 *   X2 
 
Failure region Design point, y*
Mean, μx Safe region
G(X)<0 g(Y)=0
μx2 g(Y)>0
Safe region
β
G(X)>0
X1   X 1   x2 
x1 * μx1  ,0  Y1
  
 X1 
(a) (b)

Figure 5.2 Hasofer-Lind safety index for linear limit state function: (a) original coordinate system; (b)
reduced coordinate system.

The coordinates of the intercepts of the limit state line with the reduced coordinate axes Y1 and Y2
are (-( μX1-μX2)/σX1,0) and (0,(μX1-μX2)/σX2), respectively. From simple geometric considerations it can be
shown that the minimum distance from the origin to the limit state line β is

X  X    
 1 2  C D  (5.21)
 X2   X2   C   D2
2 
1 2  

As can be seen, it is the same as the previous safety index defined by Eq. (4.15). However, while Eq.
(4.15) was derived using the properties of the independent normal random variables C and D, Eq. (5.21)
was obtained in a different way based on geometry. This shows that when the limit state function is a
linear function of two independent normal random variables C and D, both definitions give the same
expression for the safety (reliability) index.

In the case when the limit state function G(X) is a linear function of n independent normal random
variables, Xi (i=1,…,n) given by Eq. (4.21) the corresponding reduced limit state function g(Y) is

 
n
g Y   a0   ai  X i   X i Yi (5.22)
i 1

The limit state surface g(Y)=0 in the reduced coordinate system is a hyperplane described by the
following equation

n n
a0   ai  X i   ai X i Yi  0 (5.23)
i 1 i 1

Safety, Risk and Reliability, Heriot-Watt University 107


5. First Order Methods of Reliability Analysis
Its intercepts with the coordinate axes are

n
a 0   ai  X i
intercept with Yi   i 1
(5.24)
ai X i

and as known from geometry, it lies at the following distance from the origin, which by definition is the
safety index β

n
a 0   ai  X i
distance from the origin  i 1
 (5.25)
 a  
n
2
i Xi
i 1

The formula is exactly the same as Eq. (4.24) derived previously for the safety index using the properties
of independent normal random variables. Once again, both definitions of the safety index: via the mean
and standard deviation of the limit state function β=μG/σG proposed by Cornell (1969) and the geometric
definition suggested by Hasofer and Lind (1974), lead to the same result.

This changes when the limit state function is a nonlinear function of random variables. Figure 5.3
illustrates the case. The circles in the figure show contours (i.e., equal value lines) of the standard
normal probability density function of Y1 and Y2 – the function is axisymmetric, has the highest value at
the origin (the mean point) and decreases continuously with increasing distance from the origin. Thus,
the design point y* (i.e., the closest to the origin point on the limit state surface) is the most probable
failure point.
Y2
g(Y)=0
Failure region
g(Y)<0

Design point, y* α1
y 2*
α2 1 Safe region
β
g(Y)>0

y 1* Y1

Figure 5.3 Hasofer-Lind safety index for nonlinear limit state function.

Safety, Risk and Reliability, Heriot-Watt University 108


5. First Order Methods of Reliability Analysis
As can be seen, the Hasofer-Lind safety index  is also the shortest distance from the origin to the
tangent line to the limit state surface g(Y)=0 at the design point. The equation of the tangent line can be
found by linearization of the limit state surface using a Taylor series expansion about the design point

g g
g Y   g y *  Y1  y1 *  Y2  y 2 *  0 (5.26)
Y1 y*
Y2 y*

Since the design point lies on the limit state surface g(y*)=0, Eq. (5.26) can be written as

g g  g g 
Y1  Y2   y1 *  y 2 *  0 (5.27)
Y1 Y2  Y1 Y2 
y* y*  y* y* 

The second term in Eq. (5.27) is just a constant and the shortest distance from the coordinate origin to
this line, i.e., β, is

g g
y1 *  y2 *
Y1 Y2
 
y* y*
(5.28)
2 2
 g   g 
    
 Y1  y*  Y2  y*

The result can be extended for n independent normal random variables X={X1,X2,…, Xn}. First, they are
transformed into the standard (or reduced) random variables Y={Y1,Y2,…,Yn} using Eq. (5.16). The
equation of the tangent hyperplane to the limit state surface in the reduced coordinate system is then
found using a Taylor series expansion about the design point

n
g
g Y   g y *   Yi  y i *  0 (5.29)
i 1 Yi y*

Since g(y*)=0, Eq. (5.29) becomes

n
g n
g

i 1 Yi
Yi  
i 1 Yi
yi *  0 (5.30)
y* y*

From geometry, the shortest distance from the coordinate origin to the hyperplane, i.e.,  , can be
found as

n
g
 Y
i 1
yi *
i y*
  (5.31)
2
n
 g 
  Y
i 1 

i  y*

Safety, Risk and Reliability, Heriot-Watt University 109


5. First Order Methods of Reliability Analysis
It should be clear that the safety index defined this way is invariant, because regardless of the
formulation of the limit state function, the geometric shape and location of the boundary between the
failure and safe regions, i.e., the limit state surface, remain the same and so the shortest distance from
this boundary to the coordinate origin. The probability of failure is estimated as before as Pf = (-).

The approach to the evaluation of the Hasofer-Lind safety index may be interpreted in a different
way, similar to the FOSM method considered in the previous lecture. The differences are: (i) the
problem is solved in the reduced coordinate system, i.e., original random variables X are transformed to
reduced random variables Y; (ii) the linearization of the limit state function is carried out about the
design point y* and not about mean values. The ratio of the mean to the standard deviation of the
linear approximation of the limit state function obtained this way should be equal to  given by Eq.
(5.31).

There are the following relationships between the design point y* and the safety index in the
reduced coordinates system

n
   y *
2
i (5.32)
i 1

and

yi *   i  (i=1,2,…,n) (5.33)

where

g
Yi
i 
y*
(i=1,2,…,n) (5.34)
2
n
 g 
  Y
i 1 

i  y*

are the direction cosines of the unit outward normal to the limit state surface at the design point (see
Figure 5.3). The negative sign in Eq. (5.33) arises because αi the components of the outward normal as
defined in conventional mathematical notation. The design point in the original coordinate system, x*,
can then be found as

xi *   X i   X i y*   X i   i X i  (i=1,2,…,n) (5.35)

Worked Example 5.5

In Worked Example 5.2 the same simple problem was solved twice: 1) with the linear limit state
function G(X) = C – D so that the correct solution was obtained; 2) with the nonlinear limit state function
Safety, Risk and Reliability, Heriot-Watt University 110
5. First Order Methods of Reliability Analysis
G(X) = C/D – 1 using the FOSM method that yielded an incorrect solution. Solve the problem again with
the nonlinear limit state function and the Hasofer-Lind definition of the safety index. In order to solve
the problem find the design point from the correct solution 1) and use Eq. (5.28) to calculate  .

Solution

In the problem μC = 12 kN, σC = 2.4 kN, μD = 6 kN, σD = 0.6 kN. The correct solution

 C  D 12  6 6
    2.4254
 
2
C
2
D 2.4  0.62 2
6.12

The linear limit state function in the reduced coordinate system

C  C D  D
Y1  and Y2  is g Y   C   C Y1   D   DY2 .
C D

Since the limit state surface is a linear function (i.e., a straight line), the direction cosines of the outward
normal are the same at any point along the line and equal

g
Y1 C 2.4
1     0.9701
2
 g   g 
2
 2
C
2
D 2.4 2  0.6 2
    
 Y1   Y2 

g
Y2  D  0.6
2     0.2425
2
 g   g 
2
 2
C
2
D 2.4 2  0.6 2
    
 Y1   Y2 

The design point in the reduced coordinate system is then

y1 *  1   0.9701 2.4254  2.3528 y2 *   2   (0.2425)  2.4254  0.5882

The nonlinear limit state function in the reduced coordinate system

 C   C Y1 12  2.4Y1
g Y   1  1
 D   DY2 6  0.6Y2

The partial derivatives of the reduced limit state function at the design point are

g 2.4 2.4
   0.3778
Y1 y*
6  0.6Y2 y*
6  0.6  0.5882

Safety, Risk and Reliability, Heriot-Watt University 111


5. First Order Methods of Reliability Analysis
g 0.612  2.4Y1  0.612  2.4 2.3528
   0.0944
Y2 y*
6  0.6Y2 2
y*
6  0.6  0.58822

Substituting these results in Eq. (5.28) we obtain

0.3778   2.3528   0.0944  0.5882


   2.4254
0.3778 2   0.0944
2

Thus, using the Hasofer-Lind definition of the safety index the same result has been obtained with
two different formulations of the limit state function. This illustrates that the Hasofer-Lind safety index
does not depend on a particular formulation of the limits state function.

In Worked Example 5.5 the safety index was calculated when the design point was known. It was
done only to demonstrate the invariance of the Hasofer-Lind safety index, since when the design point is
known the safety index can be calculated much easier using Eq. (5.32). Methods for finding the design
point (that is equivalent to finding the safety index) are considered in the next section.

5.2.2 The AFOSM method

The reliability evaluation using the Hasofer-Lind definition of the safety index is based on
linearization of the limit state function (but in contrast to the FOSM method about the design point and
not about mean values of random variables) and description of the random variables only by their
means and variances (i.e., it is assumed that random variables are independent and normally
distributed). Thus, this is still a first-order second-moment method but in order to distinguish it from the
FOSM considered in the previous section it will be referred as the Advanced FOSM (AFOSM) method.

To find the design point and along with it the Hasofer-Lind safety index the constrained optimisation
problem given by Eq. (5.17) needs to be solved. Using the method of Lagrangian multipliers the problem
can be converted to the following unconstrained optimisation problem

12
 n 
minimise y,      yi2    g y  (5.36)
 i 1 

where λ is a Lagrangian multiplier.

For a stationary point of B(y,λ) (it can be a minimum, maximum or a ‘saddle point’):

1 2
  n  g
 yi   yi2   0 (i=1,2,…,n) (5.37a)
yi  i 1  y

Safety, Risk and Reliability, Heriot-Watt University 112


5. First Order Methods of Reliability Analysis

 g y   0 (5.37b)


Simultaneous solution of the system of (n+1) equations may produce a number of points. Which of this
points represents a minimum needs to be checked (actually, in general there is no guarantee that a
minimum exists). If g(y)=0 is linear or regular and convex towards the coordinate origin, the stationary
point should be a minimum. Since in structural reliability many of the limit state functions depart only
slightly from linearity, it can be expected that the minimum point will be found.

Obviously, as the number of basic random variables increases, the solution of the system of
simultaneous nonlinear equations to find the design point becomes more and more complex. Thus, a
more straightforward method of solving Eq. (5.17) is needed.

The following iterative scheme to solve Eq. (5.17) and find the design point and β has been proposed
(e.g., Melchers 1999):

1. Transform basic random variables X into independent standard normal variables Y

X i   Xi
Yi  (i=1,2,…,n)
X i

2. Transform the original limit state function G(x) into the reduced limit state function g(y)

3. Select starting values for y: y(1) =(y1(1), y2(1),….), which is often taken as the coordinate origin, but
not necessarily always.

4. Calculate the partial derivatives of g(y) with respect to yi (i=1,2,…,n) at the current value of y(m) (m
is the number of the iteration step).

2
n
 g 
5. Compute l: l ( m)
   
i 1  y i  y(m)

g
y i y(m)
6. Compute the direction cosines αi (i=1,2,…,n):  i( m ) 
l

 y 
n
7. Compute β:  ( m )  (m) 2
i
i 1

8. Compute g(y(m)).

9. Compute new values of y(m+1) using the following equation

Safety, Risk and Reliability, Heriot-Watt University 113


5. First Order Methods of Reliability Analysis
y ( m 1)
  ( m)  
 ( m) g y ( m)
   ( m)
i i  (i=1,2,…,n)
 l 
10. Iterate on steps 4 to 9 until convergence is obtained on β and g(y)=0 or close to 0.

This procedure converges reasonably rapidly in many cases, but not always. If the procedure does
not converge, the problem can be overcome by selecting different starting point and common sense
appraisal of results. It is easy to set up an Excel spreadsheet to do the calculations.

Worked Example 5.6

Solve Exercise 5.1:

A tie-bar holding up one corner of a suspended platform has a cross section area A, is made of steel with
a yield stress fy and has to support a load of W. The corresponding limit state function can be expressed
simply as GX  A  f y  W .

The mean values and standard deviations for these random variables are respectively:  A  90 mm2,

 f  320 N/mm2,  w  15 kN and  A  7 mm2,  f  25 N/mm2 and  W = 1.5 kN. Estimate the
y y

safety index.

using the AFOSM method with the above iterative procedure

Solution

The transformation of basic random variables:

A  90
y1  hence A  90  7 y1
7

Similarly,

f y  320  25 y 2 and W  15000  1500 y3

The transformed limit state function is then

g (y)  90  7 y1 320  25 y2   15000  1500 y3 

and the partial derivatives are

g g g
 7320  25 y 2 ;  2590  7 y1 ;  1500
y1 y 2 y3

Safety, Risk and Reliability, Heriot-Watt University 114


5. First Order Methods of Reliability Analysis
Calculations are carried out using Excel and the resulting spreadsheet is shown below. An Excel file
with the spreadsheet is available from the course website.

The procedure basically converges after the 4th iteration, i.e., g(y) becomes very close to zero and β
does not change in further iterations. The resulting safety index β=4.288 and the corresponding
probability of failure Pf=9.02×10-6. The results obtained by the FOSM method are: strength formulation -
β=3.93 and Pf=4.25×10-5 (Exercise 5.1); stress formulation - β=4.68 and Pf=1.4×10-6 (Exercise 5.2).

Note that the AFOSM method is an approximate method (based on linearization of the limit state
surface) and it yields the exact value of the probability of failure only when the limit state function is a
linear function of independent normal random variables. If the limit state function is nonlinear, the
method overestimates the probability of failure when the limit state surface in the reduced coordinate
space is convex towards the coordinate origin and underestimates it when the limit state surface is
concave. However, in structural reliability problems the limit state functions depart usually slightly from
linearity, hence, the estimates of Pf obtained using the AFOSM method are usually sufficiently accurate.
More accurate estimates of Pf can be obtained using Monte Carlo simulation, which will be considered
further in the lecture notes.

5.2.3 Designing to achieve given reliability level using AFOSM method

As has been noted previously, it may be necessary to design a structural element to achieve a target
reliability defined by the target safety index, βT (or, the target probability of failure, PfT). In Section 5.1.4
it has been explained how this can be done using the FOSM method. However, the FOSM method may
lead to incorrect results when the limit state function is nonlinear due to the ‘lack of invariance’
problem. In order to use the AFOSM method for this purpose, the iterative procedure described in
Safety, Risk and Reliability, Heriot-Watt University 115
5. First Order Methods of Reliability Analysis
Section 5.2.2 needs to be slightly modified. The safety index is now given (it should be equal to βT),
moreover, values of the random variables should be on the limit state surface, i.e., at any iteration cycle
g(y(m))=0. Thus, new values of the standard random variables are now calculated as

yi( m1)   i( m)  T (5.38)

The mean value of the design variable is found from the condition that g(y(m))=0 (the way it is calculated
depends on available information on variability of the design variable, i.e., whether it is described by its
standard deviation or coefficient of variation (COV)).

The modified iterative scheme to find the mean value of the design variable as follows:

1. Arrange the limit state function G(x) in such a way that it is linear in terms of the design variable.

2. Transform basic random variables X into independent standard normal variables Y

X i   Xi
Yi  (i=1,2,…,n)
X i

3. Transform the limit state function G(x) into the reduced limit state function g(y)

4. Select starting values for y: y(1) =(y1(1), y2(1),….), which is often taken as the coordinate origin, but not
necessarily always.

5. Calculate the mean value of the design parameter from g(y(m))=0.

6. Calculate the partial derivatives of g(y) with respect to yi (i=1,2,…,n) at the current value of y(m) (m
is the number of the iteration step).

2
n
 g 
7. Compute l: l ( m)
   
i 1  y i  y(m)

g
y i y(m)
8. Compute the direction cosines αi (i=1,2,…,n):  i( m ) 
l

9. Compute new values of y(m+1) using the following equation

yi( m1)   i( m)  T (i=1,2,…,n)

10. Iterate on steps 5 to 9 until convergence is obtained on the mean value of the design parameter.

The procedure is illustrated by the following two examples: in the first example variability of the design
parameter is presented by its standard deviation, in the second – by its COV.

Safety, Risk and Reliability, Heriot-Watt University 116


5. First Order Methods of Reliability Analysis
Worked Example 5.7

A solid circular cross section tie-bar is to be designed to carry a mean maximum load of 5 kN with a
standard deviation of 1 kN without yielding. The tie bar is be made of steel with a mean yield stress of
270 N/mm2 with COV of 0.10. The diameter of steel rod to be used has a standard deviation of 0.25 mm.
The target safety index βT = 3.5. Find the required mean diameter of the rod using the AFOSM method.

Solution

The problem has been solved using the FOSM method (Worked Example 5.4). The limit state function

D 2
can be written as GX  A  f y  W , where A is the cross section area of the rod A  .
4

Hence, we can write GX  0.25D 2  f y  W .

The limit state function is then arranged that it becomes linear in terms of the diameter

GX  D 
4W
 D  1.13W 0.5 f y0.5
f y

The random variables are then expressed via the corresponding standard variables

D   D  0.25 y1 (mm); W  5000  1000 y 2 (N); f y  270  27 y3 (N/mm 2 )

The limit state function in terms of the standard random variables is

g y    D  0.25 y1   1.135000  1000 y 2  270  27 y3 0.5


0.5

To ensure that g(y)=0 the mean value of D should be equal to

 D  1.135000  1000 y 2 0.5 270  27 y3 0.5  0.25 y1

The partial derivatives of g(y) are

g
 0.25
y1

g
 5655000  1000 y 2  270  27 y3 
0.5 0.5

y 2

g
 15.2555000  1000 y 2 0.5 270  27 y3 
1.5

y3

Safety, Risk and Reliability, Heriot-Watt University 117


5. First Order Methods of Reliability Analysis
Calculations are carried out in Excel and the resulting spreadsheet is shown below. An Excel file with
the spreadsheet is available from the course website.

As can be seen, the process converges after the 5th iteration and the resulting mean value of D, which
ensures βT=3.5, is 7.0198 mm. In practice, the mean diameter can be set as 7 mm, which is just 0.3%
smaller than the calculated value.

Worked Example 5.8

The problem is very similar to Worked Example 5.7; the only difference is that variability of the
diameter of the tie-rod is given in terms of its COV, which is 0.05. Find the required mean diameter of
the rod using the AFOSM method.

Solution

The expression for the random variable D is now

D   D  0.05 D y1  μ D 1  0.05 y1  (mm)

The limit state function in terms of the standard random variables is

g y    D 1  0.05 y1   1.135000  1000 y 2  270  27 y3 0.5


0.5

To ensure that g(y)=0 the mean value of D should be equal to

 D  1.135000  1000 y2 0.5 270  27 y3 0.5 1  0.05 y1 

and the partial derivative of g(y) with respect to y1 is

Safety, Risk and Reliability, Heriot-Watt University 118


5. First Order Methods of Reliability Analysis
g
 0.05 D
y1

The rest is the same as in Example 5.7. The process converged after the 4 th iteration and the required
mean value of D this time is 7.1885 mm, i.e., can approximately be set as 7.2 mm.

The Excel spreadsheet with calculations is shown below. An Excel file with the spreadsheet is
available from the course website.

5.3 First-Order Reliability Method (FORM)


The previously considered methods (FOSM and AFOSM) are only applicable to the problems, in which
uncertainty is described by independent normal random variables. To use these methods only
information about the first two moments of random variables – mean and standard deviation, is
needed. However, if information about the type of probability distribution is available for at least some
of the basic random variables (and in engineering applications non-normal random variables are quite
common) it is desirable to take this information into account. Thus, in order to use the Hasofer-Lind
definition of the safety index, non-normal random variables need to be transformed to equivalent
normal random variables. The transformation approach can be considered as an extension of the
AFOSM method and is usually called the First-Order Reliability Method (FORM) – the limit state function
is still linearised, but random variables are no longer presented only by their first and second moments
(i.e., mean and standard deviation).

Safety, Risk and Reliability, Heriot-Watt University 119


5. First Order Methods of Reliability Analysis
5.3.1 Equivalent normal distribution transformation

This transformation is also known as the normal tail approximation. Consider a random variable X
with probability density function fX(x) and cumulative distribution function FX(x). The equivalent normal
distribution will be defined by a mean and standard deviation denoted here as μX(N) and σX(N). The
equivalent normal distribution should have similar properties to the original distribution at the design
point on the limit state surface, i.e., at x*. Two conditions are required to define μX(N) and σX(N) and these
are chosen as:

- Area under the tail of the cumulative distribution functions (CDFs) – original and normal, beyond x* is
the same, i.e.

 x *   X( N ) 
Px  x *  1  FX x *  1    (5.39)
 X
(N)

where Φ(.) denotes the CDF of a standard normal distribution.

- Probability density functions (PDFs) – original and normal, have the same value at x*, i.e.

  x   X( N ) 
 d  
 dFX x      X( N ) 
    
 dx  x*  dx 
 
  x*

or

 x *   X( N ) 
f X x *  ( N )  
1
 (5.40)
 X   X( N ) 

where ϕ(.) denotes the PDF of a standard normal distribution given by Eq. (3.46) (i.e., ϕ(.)≡fU(.)).

From Eq. (5.40)

 x *   X( N ) 
  
 X( N )
 X( N )    (5.41)
f X  x *

From Eq. (5.39)

 x *   X( N ) 
FX x *    (5.42)
 X
(N )

Safety, Risk and Reliability, Heriot-Watt University 120


5. First Order Methods of Reliability Analysis
therefore

x *   X( N )
  1 FX x * (5.43)
 (N)
X

Substituting Eq. (5.43) into Eq. (5.41) leads to

  1 FX x *
 X( N )  (5.44)
f X x *

and from Eq. (5.43)

 X( N )  x *  X( N )  1 FX x * (5.45)

Eqs. (5.44) and (5.45) define the equivalent normal distribution for the point x*. This procedure can
be incorporated into the iterative procedure for finding the safety index. Note that on each iteration the
estimated design point x* changes and so the parameters μX(N) and σX(N) of an equivalent normal
distribution will need to be redefined.

Worked Example 5.9

Show that the parameters of the equivalent normal distribution for a lognormal random variable X
can be calculated as

 X( N )  x *  ;  X( N )  x * 1  ln x *   

where ξ and λ are the parameters of the lognormal distribution.

Solution

From Section 3.5.2.2 we know that the PDF and the CDF of a lognormal distribution are

1  ln x *     ln x *   
f X x *   ; FX x *   
x *       

Substitute these relationships into Eqs. (5.44) and (5.45)

 ln x *   
  
 FX x *
1
1  1   ln x *       
 X( N )         
f X x * f X x *       f X  x *
x * f X x *
  x *
f X x *

Safety, Risk and Reliability, Heriot-Watt University 121


5. First Order Methods of Reliability Analysis
  ln x *    

 X( N )  x *  X( N )  1 FX x *  x *  X( N )  1    
     
 ln x *     ln x *   
 x *  X( N )    x *  x *     x * 1  ln x *   
     

5.3.2 Iterative procedure for FORM

An iterative procedure for calculation of the safety index, which incorporates the transformation
described above, was proposed by Rackwitz and Fiessler (1978). The Rackwitz-Fiessler procedure
consists of the following steps:

1. Select starting values for x: x(1) =(x1(1), x2(1),….), which are often taken as the mean values, but not
necessarily always.

 G 
2. Calculate G(x ),   ,  i 
  1 FX i xim     and    
 xi( m)   i  1 FX i xi( m) (i=1,2,…,n).
 
(m)

 xi  x( m )
i
f X i xi( m )

3. Calculate

n
 G  ( m )
 
x    xi
i 1  xi  x*

n
 G 
      i
i 1  xi  x*

2
 G  
 
n
     i 
2

 xi  x* 
i 1 

4. Calculate

 G 
   i
 xi  x*
i   (i=1,2,…,n)
   2

 
 x     Gx  ( m)

  2

5. Compute new values of x(m+1)

xi( m1)   i   i i  (i=1,2,…,n)

Safety, Risk and Reliability, Heriot-Watt University 122


5. First Order Methods of Reliability Analysis
6. Iterate on steps 2-5 until convergence is obtained on β and G(x)=0 or close to 0.

The iterative procedure can be incorporated in Excel spreadsheets for calculations.

Worked Example 5.10

The problem is similar to Exercise 5.1 and Worked Example 5.6 solved by the FOSM and AFOSM
methods; however, this time there are non-normal random variables.

A tie-bar holding up one corner of a suspended platform has a cross section area A, is made of steel
with a yield stress fy and has to support a load of W. The corresponding limit state function can be
expressed simply as GX  A  f y  W .

Find the safety index when the mean values, standard deviations and distribution types for these
random variables are:

A: μA=90 mm2, σA= 7 mm2, normal distribution;

fy: μfy=320 N/mm2, σfy= 25 N/mm2, lognormal distribution; and

W: μW=15 kN, σW= 1.5 kN, Gumbel distribution.

Solution

Parameters of the lognormal distribution of fy:

COV f y 
25
320
1
  1
 0.078 ,   0.078 ,   ln  f y  ln 1  COV f2y  ln 320   0.078 2  5.7653
2 2

Parameters of the Gumbel distribution of W:

1  1  0.5772157 0.5772157
n    8.550  10 4 , u n  W   15000   14325
6 w 6 1500 n 8.550  10 4

Denote x1  A , x2  f y , x3  W

Partial derivatives of G(X)=x1x2-x3:

G G G
 x2 ,  x1 ,  1
x1 x 2 x3

Parameters of equivalent normal distributions:

 1  7; 1  90

 2  0.078x2 ;  2  x2 1  lnx2   5.7653  x2 6.7653  lnx2 

Safety, Risk and Reliability, Heriot-Watt University 123


5. First Order Methods of Reliability Analysis
par3  8.550  10 4  x3  14325

  1 exp  exp  par3 


3 
8.550  10 4 exp  par3  exp  par3 

3  x 3  3  1 exp  exp  par3 

The Excel spreadsheet with calculations is shown below. An Excel file with the spreadsheet is
available from the course website.

The process converged after the 5th iteration and the safety index β=3.779 with the corresponding
probability of failure Pf =7.870×10-5.

In Worked Example 5.6, in which all the basic random variables were treated as normal, the safety
index was β=4.288 with the corresponding Pf =9.020×10-6. The difference between the results of the two
examples is only due to the change of distribution types for the yield stress – from normal to lognormal,
and for load – from normal to Gumbel. If to compare the PDFs of the normal and lognormal distributions
describing the yield stress, they look almost identical in the region of high probabilities, i.e., around the
mean value (=320 N/mm2) of fy (see Figure 5.4a), within which most of experimental data on values fy
are available. The difference between the distributions becomes noticeable in the region around the

Safety, Risk and Reliability, Heriot-Watt University 124


5. First Order Methods of Reliability Analysis
design point of the two solutions: 254.55 N/mm2 (=320-25×2.6177) in Worked Example 5.6 and 286.30
N/mm2 in Worked Example 5.10 (see Figure 5.4b).

0.018 0.001
0.016 Normal Normal
0.0008 Lognormal
0.014 Lognormal
0.012
0.0006
PDF

0.01

PDF
0.008
0.0004
0.006
0.004
0.0002
0.002
0
0
100 150 200 250 300 350 400 450 500
220 240 260 280
Yield stress of steel (N/mm2) 2
Yield stress of steel (N/mm )

(a) (b)
Figure 5.4 PDF of yield strength of steel: (a) whole distribution; (b) left tail near the design point
(Worked Examples 5.6 and 5.10).

However, since the probability of failure is very low the probability that values of fy will be observed
in this region should be very low as well – it can easily be estimated that in the examples it is of order of
10-3. This means that in order to determine experimentally which type of distribution should be used to
model variability of fy in the region of design point, i.e., the most important region for the calculation of
the probability of failure, tens of thousands of identical tests need to be carried out. From practical
point of view this is not feasible.

Thus, in structural reliability the types of distributions used to model uncertainty of various
parameters should be often assumed based on some general considerations. For example, a lognormal
distribution is considered to be more suitable for modelling strength parameters than a normal
distribution since it is defined only for positive values (and strength cannot be negative), while the use
of a normal distribution means that negative values are possible although their probability may be
negligibly low. The problem of inability to validate experimentally what type of distribution should be
used is often referred as the ‘tail sensitivity problem’, i.e., calculation of the probability of failure
depends on the tails of distributions (the left tail for resistance parameters and the right tail for load
parameters) but there are no statistical data to estimate actual probabilities associated with the tails
and validate the choice of distribution models. This is one of the reasons why the calculated probability
of failure should be treated as ‘nominal’ or ‘formal’ measure of the actual probability of structural
failure and not as an absolute value. Another important reason is that the influence of human error,
which is one of the main causes of structural failure, is not taken into account. However, it does not

Safety, Risk and Reliability, Heriot-Watt University 125


5. First Order Methods of Reliability Analysis
mean that calculated probability of failure cannot be used. It is very useful and important measure of
comparative performance of different structures.

5.4 Sensitivity factors


The direction cosine αi of the unit outward normal to the limit state surface at the design point,
which is defined by Eq. (5.34), can serve as a measure of the influence of the corresponding random
variable Xi on the safety index β and, consequently, on the probability of failure. This can be explained
by the fact that the absolute value of αi equals the derivative of β with respect to the standard random
variable yi at the design point

 
 y12  y 22    yi2   y n2 y*  2
yi y *
12

  i
yi y*
yi y1  y2    yi   yn  y*  (5.46)
2 2 2

while according to Eq. (5.33) αi=-yi*/β. Thus, αi is also called the sensitivity factor of the basic random
variable Xi.

It has been shown that β is approximately increased by a factor 1 1   i2 when the corresponding

random variable Xi is considered as deterministic parameter equal to its mean value (Madsen 1988).
Thus, neglecting uncertainty of a random variable with αi=0.1 results in an increase of β by about 0.5%
and with αi=0.3 by about 5%, respectively.

Worked Example 5.11

Examine the sensitivity factors obtained in the solution of Worked Example 5.10. Check what
happens with the value of β when the random variable with the lowest absolute value of the sensitivity
factor is treated as a deterministic parameter (i.e., represented in the analysis by its mean value).

Solution

The sensitivity factors at the end of iterations (i.e., at Iteration 6):


Random variable, xi Sensitivity factor, αi
Factor 1 1   i2
x1 – cross section area A -0.4173 1.100
x2 – yield stress fy -0.3670 1.075
x3 – load W 0.8314 1.800
The lowest absolute value of the sensitivity factor equals 0.3670 and belongs to x2 (fy). The

corresponding value of the factor 1 1   i2 = 1.075 means that if the yield stress fy is treated in a

reliability analysis as a deterministic parameter represented by its mean value (= 320 MPa) the resulting

Safety, Risk and Reliability, Heriot-Watt University 126


5. First Order Methods of Reliability Analysis
safety index will increase about 1.075 times (i.e., by 7.5%) compared to the one calculated in Worked
Example 5.10 and be equal to about 3.7791×1.075=4.0625. For the other two random variables A and
W, their replacement by the corresponding mean values should result in an increase of β by 1.100 (i.e.,
10%) and 1.800 (i.e., 80%), respectively. Thus, treating of any of these three random variables as a
deterministic parameter represented by the mean value of this random variable leads to a significant
increase in the resulting safety index and should not be done in actual reliability analysis. However, for
illustrative purposes the FORM analysis is carried out with only two random variables A and W, while fy
is treated as a deterministic parameter equal to 320 MPa. Results are shown in the table below. As can
be seen, the safety index obtained in the analysis equals 4.0673, i.e., very close to the value 4.0625
predicted above.

5.5 Correlated random variables


In many practical problems the basic random variables may be correlated. In order to use the AFOSM
method or FORM they should transformed into uncorrelated random variables. Possible procedures for
the transformation of correlated normal and non-normal random variables into uncorrelated standard
normal variables will be described in the following sections.

5.5.1 Correlated normal random variables

There are n correlated normal random variables X with the covariance matrix CX

Safety, Risk and Reliability, Heriot-Watt University 127


5. First Order Methods of Reliability Analysis
 Var  X 1  Cov X 1 , X 2   Cov X 1 , X n 
Cov X , X  Var  X 2   Cov X 2 , X n 
CX   2 1
(5.47)
     
 
Cov X n , X 1  Cov X n , X 2   Var  X n  

The transformation of the correlated random variables X into uncorrelated standard normal
variables Y can be expressed in the following form:

Y  Q 1 X  μ X  (5.48)

where μX is the vector of mean values and Q is a lower triangular matrix such that CX=QQT. In order to
find Q the so-called ‘square root’ method (Rubinstein 1981) can be used. The method consists of the
following recursive formula for computation of the elements of Q

j 1
cij   qik q jk
qij  k 1
12
(5.49)
 j 1

 c jj   q 2jk 
 k 1 

where cij are the elements of CX and

0 0

 qik q jk   q 2jk  0 and


k 1 k 1
1 j  i  n (5.50)

The inverse transformation is

X  QY  μ X (5.51)

Worked Example 5.12

The problem is similar to Worked Example 5.6 but this time there are correlated random variables.

A tie-bar holding up one corner of a suspended platform has a cross section area A, is made of steel
with a yield stress fy and has to support a load of W. The corresponding limit state function can be
expressed simply as GX  A  f y  W .

A, fy and W are normal random variables with the following means and standard deviations: μA=90
mm2, σA= 7 mm2; μfy=320 N/mm2, σfy= 25 N/mm2; μW=15 kN, σW= 1.5 kN. A and fy are correlated and with
correlation coefficient ρ=0.1. Find the safety index.

Solution

The covariance matrix for the random variables is:

Safety, Risk and Reliability, Heriot-Watt University 128


5. First Order Methods of Reliability Analysis
 72 0.1  7  25 0   49 17.5 0 
   
C X  0.1  25  7 25 2
0   17.5 625 0 
 2
  6

 0 0 1500   0 0 2.25 10

The elements of Q are calculated using Eq. (5.49)

c11
q11   49  7
c11

c21 17.5 c22  q 21


2
q 21    2.5 ; q 22   625  2.5 2  24.8747
c11 49 c22  q 2
21

c31 c32  q31q 21


q31   0 ; q32   0; q33  c33  q31
2
 q32
2
 2.25  10 6  1500
c11 c 22  q 2
21

The relationship between A, fy and W and the corresponding uncorrelated random variables Y can then
be found using Eq. (5.51)

 A  7 0 0   y1   90   7 y1  90 
         
 f y   2.5 24.8747 0   y 2    320   2.5 y1  24.8747 y 2  320
W   0 1500      1500 y3  15000 
   0  y3  15000  

The reduced limit state function is then

g (y)  7 y1  902.5 y1  24.8747 y 2  320  1500 y3  15000

Using this reduced limit state function the safety index can be calculated by the AFOSM method as in
Worked Example 5.6. Since the coefficient of correlation between A and fy is small (0.1), the resulting
safety index should be very close to that in Worked Example 5.6.

5.5.2 Correlated non-normal random variables

If there are n correlated non-normal random variables X and their joint probability distribution FX(x)
is available, an equivalent set of independent standard normal random variables Y can be obtained
using the Rosenblatt transformation. The latter represents successive inversion:

y1   1 F1 x1 

y 2   1 F2 x2 | x1 

Safety, Risk and Reliability, Heriot-Watt University 129


5. First Order Methods of Reliability Analysis
yi   1 Fi xi | x1 , x2, , xi 1  (5.52)

y n   1 Fn xn | x1 , x2, , xn1 

where Fi(xi|x1,x2,…,xi-1) is the conditional cumulative distribution function for the random variable Xi
given by

xi

f X 1 ,...,X i 1 , X i x1 ,..., xi 1 , z dz


Fi xi | x1 , x 2 ,..., xi 1   
(5.53)
f X1 ,...,X i 1 x1 ,..., xi 1 

In general, Eq. (5.52) needs to be calculated numerically.

The inverse transformation is

x1  F11  y1 

x2  F21  y 2  | x1  (5.54)

xn  Fn1  y n  | x1 ,, xn1 

In practice, the joint probability distribution is often unavailable. In such cases other transformations
may be used, e.g., the Nataf transformation (Melchers 1999).

Exercises
Exercise 5.1

A tie-bar holding up one corner of a suspended platform has a cross section area A, is made of steel
with a yield stress fy and has to support a load of W. The corresponding limit state function can be
expressed simply as GX  A  f y  W .

The mean values and standard deviations for these random variables are respectively:
 A  90 mm2,  f  320 N/mm2,  w  15 kN and  A  7 mm2,  f  25 N/mm2 and  W = 1.5 kN.
y y

Estimate the safety index and the probability of failure using the FOSM method.
Ans. β = 3.93; Pf = 4.25×10-5

Safety, Risk and Reliability, Heriot-Watt University 130


5. First Order Methods of Reliability Analysis
Exercise 5.2

Solve Exercise 5.1 using the stress formulation of the limit state function GX  f y  W A .

Ans. β = 4.68; Pf = 1.4×10-6

Exercise 5.3

A simply supported steel beam of span L=8 m is loaded by uniformly distributed load with intensity
w. The plastic section modulus of the beam is Z and the yield stress of steel is fy. Assume that w, Z and fy
are uncorrelated normal random variables with the following means and standard deviations:
Variable Mean Standard deviation
w 2.5 kN/m 0.3 kN/m
Z 1×10-4 m3 9×10-6 m3
fy 410 N/mm2 35 N/mm2
Calculate by the FOSM method the safety index and the probability of failure of the beam using

a) the strength formulation of the limit state function G(X) = Zfy - wL2/8;
Ans. β = 3.73; Pf = 9.57×10-5

b) the stress formulation of the limit state function G(X) = fy - wL2/(8Z).


Ans. β = 4.55; Pf = 2.68×10-6

Exercise 5.4

Solve Exercise 5.3 by the AFOSM method by modifying the Excel spreadsheet with solution of
Worked Example 5.6.
Ans. β = 4.10; Pf = 2.07×10-5

Exercise 5.5

A solid circular cross-section tie-bar has been designed to carry a mean load of 10 kN with a COV of
0.15. The tie-bar is made of steel with a mean yield stress of 290 N/mm 2 and a COV of 0.10. The mean
diameter of the bar is 10 mm and its COV is 0.05. Assume that all uncertain parameters can be modelled
as independent normal random variables.

Calculate in Excel by the AFOSM method the safety index and the probability of failure of the bar.
Ans. β = 4.07; Pf = 2.35×10-5

Revision Questions
5.1 Why are the FOSM and AFOSM methods called first-order second moment methods?

5.2 List the main assumptions of the FOSM method.

Safety, Risk and Reliability, Heriot-Watt University 131


5. First Order Methods of Reliability Analysis
5.3 Explain the meaning of ‘lack of invariance’ of the FOSM method.

5.4 Give the definition of the Hasofer-Lind safety index. What is the design point?

5.5 Explain the difference between the FOSM and AFOSM methods.

5.6 What is the difference between the AFOSM method and the FORM?

5.7 What is the normal tail approximation? Where is it used?

5.8 Explain the tail sensitivity problem.

5.9 What is the sensitivity factor of a random variable?

5.10 Why may it be necessary to transform correlated random variables into uncorrelated? What
methods can be used for this?

SUMMARY

 The main concepts and computation procedures of the first-order second-moments methods have
been described and illustrated by a number of examples.

 The ‘lack of invariance’ problem of the FOSM method has been explained and illustrated by
examples.

 The idea of transforming the original space of random variables to standard normal space has been
introduced. The Hasofer-Lind safety index has been defined and the concept of the design point has
been explained.

 The use of the FOSM and AFOSM methods for designing to achieve a given reliability level has been
demonstrated by examples.

 A method of transforming non-normal random variables into standard normal variables have been
described.

 The FORM has been explained and an iterative procedure implementing the method has been
presented and illustrated by an example.

 The tail sensitivity problem has been explained.

 It has been explained how the influence of different random variables on results of reliability analysis
can be estimated.

 Methods for transforming correlated random variables into uncorrelated ones have been described.

Safety, Risk and Reliability, Heriot-Watt University 132


5. First Order Methods of Reliability Analysis
References
Cornell, C.A. (1969). A probability based structural code. Journal of American Concrete Institute, 66(12),
974-985.

Hasofer, A.M. and Lind, N. (1974). An exact and invariant first-order reliability format. Journal of
Engineering Mechanics, 100 (EM1), 111-121.

Madsen, H.O. (1988). Omission sensitivity factors. Structural Safety, 5(1), 35-45.

Melchers, R.E. (1999). Structural Reliability: Analysis and Prediction. 2nd Ed., John Wiley & Sons.

Rackwitz, R. and Fiessler, B. (1978). Structural reliability under combined random load sequences.
Computers and Structures, 9(5), 484-494.
Rubinstein, R.Y. (1981). Simulation and the Monte Carlo Method. John Wiley & Sons, New York, NY.

Safety, Risk and Reliability, Heriot-Watt University 133


5. First Order Methods of Reliability Analysis
6 MONTE CARLO SIMULATION

6.1 Introduction
Monte Carlo simulation refers to a group of computational methods that rely on repeated random
sampling to compute their results. Monte Carlo methods are often used to simulate engineering,
physical and mathematical systems. Because of their reliance on repeated computation and random or
more correctly pseudo-random numbers, Monte Carlo methods are most suited to calculation by a
computer. Monte Carlo methods tend to be used when it is infeasible, or impossible, or simply difficult
to compute an exact result with a deterministic algorithm.

The name ‘Monte Carlo’ was introduced in the 1940s by physicists (Stanislaw Ulam, Enrico Fermi,
John von Neumann and others) working on nuclear weapon projects in the Los Alamos National
Laboratory. The name is a reference to the Monte Carlo Casino in Monaco since the use of randomness
and the repetitive nature of Monte Carlo simulation are analogous to the activities conducted at a
casino.

In structural reliability theory there are several ways of calculating the safety or reliability index and
the corresponding probability of failure. Apart from the simple FOSM method considered in the previous
chapter most of these methods involve iterative procedures which can become quite complex if the
strength and load variables are not normally distributed. In the FORM method transformations from the
original distributions to corresponding equivalent normal distributions are needed at each cycle of the
iteration. In addition the solutions obtained are not exact, albeit the approximations are usually
reasonable. The Monte Carlo simulation enables, in principle, to obtain very accurate solutions for the
probability of failure provided the simulation is run for long enough. With the rapid development of low
cost and fast computing Monte Carlo simulation is becoming increasingly attractive.

Monte Carlo simulation methods in structural reliability include the following essential elements: (1)
defining the basic random variables for the problem under consideration; (2) quantifying the
probabilistic properties of the basic random variables in terms of their PDFs (or PMFs) and the
corresponding parameters; (3) generating values of the basic random variables; (4) evaluating the limit
state function for each trial, i.e., generated set of values of the basic random variables; (5) aggregating
results of individual trials into an estimate of the probability of failure; (6) evaluating the accuracy of the
calculated estimate of the probability of failure. In the following the elements (3)-(6) will be considered
in more detail.

Safety, Risk and Reliability, Heriot-Watt University 134


6. Monte Carlo Simulation
6.2 Basic steps of Monte Carlo simulation
To find the probability of failure Pf possible values the basic random variables X are generated N
times and for each generated set (i.e., trial) x(j) (j=1,2,…,N) the limit state function G(x(j)) is evaluated.
The probability of failure is then estimated as the ratio of the number of trials nf, at which the limit state
function less or equal than zero, to the total number of trials N

Pf 
   
n f G x( j)  0
(6.1)
N

In more detail, the process usually represents the sequence of the following steps:

1. Generate a random number u uniformly distributed between 0 and 1, i.e., 0≤u≤1 (there are a large
number of programs including Excel, which have capability to generate such random numbers;
many calculators also have the Ran# button (or similar), which returns such a number).

2. Find a value of the basic random variable Xi with the CDF FXi(xi) using the generated number u.
Possible methods for doing this will be described further in the chapter (note that a number of
computer programs are able to generate different types of random variables and not only uniform
on [0,1)).

3. Repeat steps 1 and 2 for all the basic random variables appearing in the limit state function G(X).

4. Substitute the generated values x(j) of the basic random variables into G(X) and calculate its value,
i.e., G(x(j)).

5. Repeat steps 1, 2, 3 and 4 for N trials calculating a new value of G(X) at each cycle.

6. Count the number of trials nf for which G(x(j)) ≤ 0

 
N
n f   I x( j) (6.2)
j 1

where

  ( j)

 
1, G x ( j )  0
 
Ix (6.3)
0, G x  0
( j)

7. Calculate the estimate P̂f of the probability of failure using the expression below

 I x   
N
j
nf
Pˆ f 
j 1
 (6.4)
N N

Safety, Risk and Reliability, Heriot-Watt University 135


6. Monte Carlo Simulation
8. Calculate the variance of P̂f and its coefficient of variation as

  1 N
   
Pˆ f 1  Pˆ f 
Var Pˆ f   P2ˆ   I x ( j )  Pˆ f
2
 (6.5a)
f
 
N N  1 j 1 N 1

 Pˆ 1  Pˆ f 1
COVPˆ  f
  (6.5b)
f
Pˆ f N  1Pˆ f nf

The resultant values of G(X) obtained at each trial, if plotted as a histogram, would appear as shown
below in Figure 6.1.

Frequency SAFE
of G(x)
G(X) REGION

FAILURE
REGION

G(x)G(X)

Figure 6.1 Distribution of the limit state function G(X) showing the safe and unsafe regions.

6.3 Advantages and disadvantages of Monte Carlo simulation


Advantages

1. There is no need to transfer variables into standard normal space.

2. Variables need not be normally distributed and can have any distribution function. Even empirical
distributions without explicit analytical forms can be used.

3. Complex limit state function G(X) can readily be handled.

4. Variables do not have to be independent. Correlated variables can be treated provided the
correlation is known.

5. As N   the estimate of the probability of failure approaches an exact value, i.e.,

Safety, Risk and Reliability, Heriot-Watt University 136


6. Monte Carlo Simulation
lim Pˆ f  Pf 
N   f xdx
G  X 0
X

Disadvantages

1. A computer and a computer program are needed for calculations.

2. A large number of trials, N, is necessary to obtain a sufficiently accurate estimate of the probability
of failure, when the latter is very low. This may require significant computational time.

6.4 Accuracy of simulation


It is obviously important for the simulation to run over a sufficient number of trials to get an accurate
estimate of the probability of failure. Eq. (6.5) provides information about the variance of the estimate
of the probability of failure during the simulation process and, as can be seen, the variance decreases
with the number of simulation trials N. However, it may be necessary to determine the number of trials
required to achieve certain accuracy in the estimation of Pf before starting the simulation process.

The error related to the number of trials can be estimated by approximating the binomial distribution
with a normal distribution as was suggested by Shooman (1968). According to this suggestion, each trial
in Monte Carlo simulation may be considered as a Bernoulli trial with the probability of failure (i.e.,
G(X)≤0) p=Pf and the probability of success q=(1-p). The number of trials in which failure occurs, nf,
then has a binomial distribution with total number of trials N, mean μ=Np and variance σ2=Npq (see Eqs.
(3.37)-(3.38)). Since N is large and p is small the binomial distribution can be approximated by a normal
distribution with the same mean and variance. In this case (μ±kσ) will be the 95% confidence interval for
nf (i.e., the interval in which nf will be observed with probability of 0.95) if


P  k n f   k  0.95  (6.6)

This means that Φ(k)=0.975 and from the table of normal distribution values it can be found that k=1.96.
If to substitute this result as well as formulae for μ and σ into Eq. (6.6) there is obtained


P  1.96 NPf 1  Pf  n f  NPf  1.96 NPf 1  Pf   0.95 (6.7)

At the same time the error between the estimate P̂f and the true probability of failure Pf is

nf
Pˆ f  Pf  Pf n f  NPf
  N  (6.8)
Pf Pf NPf

Safety, Risk and Reliability, Heriot-Watt University 137


6. Monte Carlo Simulation
From the last equation (nf - NPf) = εNPf and if to substitute this in Eq. (6.7) the absolute error with
probability of 0.95 should not exceed

1  Pf
  1.96 (6.9)
NPf

The relationship between the desired percentage error, ε%, and the total number of trials in Monte
Carlo simulation with 95% confidence can be expressed as

1  Pf 1
 %  1.96  100%  196 % (6.10)
NPf NPf

The last approximation is sufficiently accurate for Pf <<1, which almost always the case in structural
reliability.

Worked Example 6.1

If it is expected that the probability of failure is Pf= 1×10-4 estimate the required number of
simulation trails if the desired error is 20% with 95% confidence. What will be the required number of
trials if the desired error is 10%?

Solution

From Eq. (6.10)

2
1 1  196 
 %  196 4
 20% → N  4    960400
N x 10 10  20 

When ε%=10%

2
1  196 
N  4    3841600
10  10 

As can be seen, the required number of trials is very large and increases inversely proportional to the
probability of failure. The number of trials in Monte Carlo simulation required to provide a certain level
of accuracy can be reduced significantly if to use the so-called variance reduction techniques. These
techniques will be discussed further in the chapter.

6.5 Generation of random numbers uniform distributed on [0,1)


A computer or calculator usually generates random numbers from a uniform distribution over the
range 0 to 1 using a recursive algorithm called a linear congruential generator

Safety, Risk and Reliability, Heriot-Watt University 138


6. Monte Carlo Simulation
 ar  c 
ri 1  ari  c  mod m  ari  c   mINT i 
 m 

where a, c, and m are integers and INT means only the integer part of a real number is used; 0 < m – the
‘modulus’, 0 < a < m – the ‘multiplier’, and 0 ≤ c < m – the ‘increment’. In order to start the generation a
start value of r - an integer 0 ≤ r0 < m, which is called the ‘seed’, needs to be set. Then ri+1 lies in the
range between 0 and m-1 and hence

ri
u i 1 
m

gives a number on [0,1).

Consider an example of the use of this recursive algorithm when the following values are used: a=3,
c=1 and m=5 with the seed r0=1.

Then

 3 1  1 
r1  3  1  1  5 INT   4  5 0  4
 5 
4
u1   0.8
5

 3 4 1
r2  3  4  1  5 INT   13  5  2  3
 5 
3
u 2   0.6
5

This sequence repeats itself quite rapidly

i 0 1 2 3 4 5 6 7 8

ui - 0.8 0.6 0 0.2 0.8 0.6 0 0.2

This illustrates the importance of choosing the coefficients a, c, and m; the generator is very sensitive
to their values. All random numbers generated in this way are pseudo-random since a generated
sequence is completely determined by a small set of initial values a, c, m and r0. Note that the use of the
same seed r0 will always result in the same sequence. A sequence of pseudo-random numbers always
begins to repeat itself. The maximum length of such sequence before it starts to repeat itself called its
period is a very important parameter, which depends on the choice of a, c, and m. For example, in Excel
2002 and older versions, the standard RAND function used the generation algorithm which had a
relatively small period (less than 1 million numbers), so if you needed to run the simulation hundreds of
thousands of times, you could run out of random numbers. This problem has been fixed in Excel 2003
Safety, Risk and Reliability, Heriot-Watt University 139
6. Monte Carlo Simulation
and later versions. Typical values of the coefficients used in random number generators are: m=232,
a=1664525 and c=1013904223.

One of the most powerful and efficient pseudorandom number generators available today is called
the Mersenne twister. It was developed in 1998 (Matsumoto and Nishimura 1998) and has a period of
approximately 4.3 × 106001. The Mersenne twister uses a linear congruential generator similar to that
described above in order to generate its seed.

6.6 Generating other random numbers


In this section methods for generating random values from various distribution types using random
numbers ui uniformly distributed on [0,1) are described.

6.6.1 The inverse transform method

If the cumulative distribution function (CDF) can be written explicitly then a sample of the random
variable can be found using the inverse transform method. In this method, the CDF of the random
variable X is equated to the generated random number ui, which is uniformly distributed on [0,1), i.e.,

FX x i   ui (6.11)

Eq. (6.11) is then solved for xi, which is a random value distributed in accordance to FX(x)

x i  FX1 ui  (6.12)

Figure 6.2 illustrates the method.

1.0

f(x) F(x)

ui u

0
x xi x

Figure 6.2 Illustration of the inverse transform method.

Safety, Risk and Reliability, Heriot-Watt University 140


6. Monte Carlo Simulation
For example, consider how this method can be used in order to generate a random variable X
uniformly distributed on the interval [a,b). The CDF of X is
xa
FX  x  
ba
If to substitute this formula in Eq. (6.11)
xi  a
 ui
ba

so that

xi  ui b  a   a (6.13)

Consider an exponential distribution with the CDF

FX x   1  e x x  0

If to substitute this CDF into Eq. (6.11)

1  e xi  ui

Rearranging this expression gives

ln 1  u i   FX1 u i 
1
xi   (6.14)

where xi is a random value from the given exponential distribution.

As a further example consider the Gumbel (Extreme Value Type 1) distribution for the largest values

FX x   exp  exp   n x  u n 

Using the inverse transform method

ln  ln(u i ) 
1
xi  u n  (6.15)
n

6.6.2 Generating normal and lognormal random variables

The inverse function of the CDF of a normal distribution cannot be found analytically. Thus, the use
of the inverse transform method for this distribution is not efficient. Another procedure proposed by
Box and Muller (1958) can be employed. The procedure using a pair of random numbers u1 and u2
uniformly distributed on [0,1) produces a pair of statistically independent values y1 and y2 from the
standard normal distribution

Safety, Risk and Reliability, Heriot-Watt University 141


6. Monte Carlo Simulation
y1   2 ln u1  2 cos2u 2 
1

y 2   2 ln u1  2 sin2u 2 
1
(6.16)

Now, if X is normally distributed with mean μX and standard deviation σX a pair of its random values can
be obtained as

x1   X   X  2 ln u1 cos 2πu 2 
(6.17)
x2   X   X  2 ln u1 sin 2πu 2 

The procedure can easily be extended for a lognormal distribution. If X has a lognormal distribution with
the parameters λ and ξ then a pair of its random values can be obtained as


x1  exp     2 ln u1 cos2πu 2  
 exp    sin 2πu 
(6.18)
x2  2 ln u1 2

Worked Example 6.2

A tie-bar holding up one corner of a suspended platform has a cross section area A, is made of steel
with a yield stress fy and has to support a load of W. The corresponding limit state function can be
expressed as GX  A  f y  W .

The mean values, standard deviations and distribution types of the basic random variables are:

A: μA=90 mm2, σA= 7 mm2, normal distribution;

fy: μfy=320 N/mm2, σfy= 25 N/mm2, lognormal distribution; and

W: μW=15 kN, σW= 1.5 kN, Gumbel distribution.

The probability of failure of the tie-bar is calculated using Monte Carlo simulation. The problem has
been solved previously by the FORM (Worked Example 5.10).

(a) Perform two simulation trials using the following random numbers: u1=0.4753, u2=0.8741 and
u3=0.3046 and then u1=0.7153, u2=0.3271 and u3=0.1046.

Solution

From Worked Example 5.10:

- the parameters of the lognormal distribution of fy: λ=5.7653, ξ=0.078;

- the parameters of the Gumbel distribution of W: αn=8.55×10-4, un=14325.

Formulae for generating the basic random variables:

Safety, Risk and Reliability, Heriot-Watt University 142


6. Monte Carlo Simulation
  
A( j )  90  7  2 ln u1( j ) sin 2u 2( j ) 
   
f y( j )  exp 5.7653  0.078  2 ln u1( j ) cos 2u 2( j ) 

W ( j )  14325 
1
8.55  104

ln  ln u3( j ) 
Note that the superscripts (1) and (2) indicate the first and second simulation trials, respectively.

A(1)  90  7  2 ln 0.4753 sin2  0.8741  83.93 mm 2

 
f y(1)  exp 5.7653  0.078  2 ln 0.4753 cos2  0.8741  341.1 N/mm 2

ln ln 0.3046  14123 N


1
W (1)  14325 
8.55  104

GX   A1  f y1  W 1  83.93  341.1  14123  14506 N  0 - no failure


1

A( 2)  90  7  2 ln 0.7153 sin2  0.3271  95.07 mm 2

 
f y( 2)  exp 5.7653  0.078  2 ln 0.7153 cos2  0.3271  309.7 N/mm 2

ln ln 0.1046  13373 N


1
W ( 2 )  14325 
8.55  104

GX 
2 
 A2   f y2   W 2   95.07  309.7  13373  16070 N  0 - no failure
This process should be continued with millions more trials before sufficiently accurate estimate of the
probability of failure will be obtained. Of course, this cannot be performed manually.

The evaluation of structural reliability by Monte Carlo simulation is carried out using computers.
Nowadays a powerful PC is usually sufficient and a number of programs are commercially available that
can handle large problems and an extensive range of distribution functions. For simple problems that we
consider in this course a program can be written in a few minutes in Visual Basic, which runs from an
Excel spreadsheet, or another high-level language.

This worked example explains how a simple Visual Basic program can be written, or adapted, and run
from Excel. The coursework associated with this course requires an existing simulation program in Visual
Basic to be adapted to solve another problem. Just as the FOSM method can be used to design
structural components to achieve given probabilities (or target safety indices), so can Monte Carlo
simulation. This merely involves systematically varying the design variables until the calculated
probability of failure becomes less or equal to a target value.

Safety, Risk and Reliability, Heriot-Watt University 143


6. Monte Carlo Simulation
(b) Calculate the probability of failure of the tie-bar by Monte Carlo simulation from Excel with the help
of a Visual Basic program

Solution

The Excel spreadsheet with the Visual Basic coding for this worked example is available from the
course website. The Excel spreadsheet page is presented below. This shows the worksheet heading, the
Start Simulation button and the values of the variables returned at the end of the simulation. In this case
there are 10,000,000 trials resulting in 880 failures giving an estimated probability of failure of 8.8×10-5.
As can be seen, the coefficient of variation of the estimate of Pf calculated based on Eq. (6.5), COV_Pf =
0.0337. The value is sufficiently low (less than 0.05) so that the estimate of the probability of failure is
reasonably accurate. However, if to run the program again, a slightly different estimate of Pf will be
obtained. The accuracy will improve if to increase the number of simulation trials. The estimate of the
probability of failure is slightly higher than that obtained by the FORM in Worked Example 5.10
(7.87×10-5).

The following steps are involved in using a Visual Basic simulation program in Excel (in MS Excel
2010).

1. Start by loading the Excel simulation spreadsheet. Note that in MS Excel 2010 in order to work with
Macros (Visual Basic) you need to add the Developer Tab to the Ribbon (see Excel Help for more
detail).

2. You may get the message: - “The macros in this project are disabled. Please refer to the online help
or documentation of the host application to determine how to enable macros.” To enable the
macros the Security level must be set to Medium or Low to allow the Visual Basic program, which is
a Macro, to run. You may need to restart Excel for this change to take effect.

Safety, Risk and Reliability, Heriot-Watt University 144


6. Monte Carlo Simulation
3. In the spreadsheet click on ‘Developer’ and then ‘Visual Basic’ that brings up the Visual Basic code
sheet.

The coding for this Visual Basic program is given below. The various instructions in the program code
are described after the coding.

A copy of the Visual Basic subroutine used in Worked Example 6.2b

Private Sub CommandButton1_Click()


' Simulation solution to Example 6.2b
Numfails = 0
Ntrials = 10000000
Range("C8").Value = Ntrials
Pi_2 = 2 * 3.14159265358979
Randomize

For i = 1 To Ntrials
u1 = Rnd
u2 = Rnd
u3 = Rnd
If u1 = 0 Then
u1 = Rnd
End If
If u3 = 0 Then
u3 = Rnd
End If
A = 90 + 7# * Sqr(-2 * Log(u1)) * Sin(Pi_2 * u2)
f_y = Exp(5.7653 + 0.078 * Sqr(-2 * Log(u1)) * Cos(Pi_2 * u2))
W = 14325 - 1 / 0.000855 * (Log(-Log(u3)))
Gx = A * f_y - W
If Gx < 0! Then
Numfails = Numfails + 1
End If
Next i

Pf = Numfails / Ntrials
beta = -Application.WorksheetFunction.NormSInv(Pf)
COV_Pf = Sqr((1# - Pf) / (Ntrials - 1) / Pf)

Range("C10").Value = Numfails
Range("C12").Value = Pf
Range("C14").Value = COV_Pf
Range("C16").Value = beta

End Sub

The first instruction allows the program to be started from the Excel spreadsheet by clicking on the
button marked ‘Start Simulation’. The next instruction, starting with “'” is just a comment statement and
has no effect on the program. ‘Numfails’ is the counter for the number of times G(X) is less than zero. It
is set to zero at the outset. ‘Ntrials’ is the number of simulation trials; usually set low during program
development and then to a sufficiently large number to ensure at least 100 or so ‘failures’. ‘Randomize’
resets the random number generator. There follows a “For Next” loop that is usually indented as shown.

Safety, Risk and Reliability, Heriot-Watt University 145


6. Monte Carlo Simulation
This loop is executed ‘Ntrials’ times. Firstly, random numbers between 0 and 1 are generated. ‘Rnd’ is a
random number from a uniform distribution in the interval 0 to 1.0 (each time it is called a new random
number is generated). Some generated random numbers may be undistinguished from 0. If u1 or u3
equals 0 the program will stop working and error message will appear because the argument of Log
function cannot be 0. In order to avoid this, the ‘If’ statements check whether ‘u1’ and ‘u3’ equal 0; if
this is true a new random number is generated. After that the statements in the loop use the equations
discussed above to get random values for each of the variables in turn from their appropriate
distributions. ‘Gx’ is the limit state function that is evaluated on each cycle. The ‘If’ statement checks
whether ‘Gx’ is less than 0; if this is true the ‘Numfails’ is incremented by 1. The ‘Next’ statement returns
the execution up to the ‘For’ statement and this only stops when ‘Ntrials’ are completed.

The ‘Range’ statements return to the cells specified in the “ ” the current Values of the variables
specified. These are ‘Ntrials’, ‘Numfails’ (which is now the total number of failures in the ‘Ntrials’), ‘Pf’
(equals Numfails/Ntrials) which the estimate of the probability of the failure, and COV_Pf which is the
coefficient of variation of the estimate of Pf. The End Sub statement stops the programme.

This Visual Basic coding can easily be edited for use on other problems. The following notes explain
how to do this.

1 In the ‘design mode’ you can edit the Visual Basic code. You type in new lines and edit or remove
existing lines just as would edit any other document. After each change you make you can use the
‘debug’ facility to check the modified code works. Move the cursor to just below the point where
you have made the changes and use ‘run to cursor’. Errors are highlighted and moving the mouse
over the code produces a box giving current numerical values. For problems try ‘Help’ – it is quite
good.

2 Having made your changes under ‘Run’ menu click on ‘Reset’.

3 To return to the spreadsheet click on the icon at the bottom of the screen or just close the window.

4 In the spreadsheet click on the button ‘Start simulation’ to run the program.

You should be able to edit this code to solve exercises and coursework questions.

6.7 Variance reduction techniques - importance sampling


As can be seen from Eq. (6.5b), the coefficient of variation (COV) of the estimate of the probability of
failure obtained by Monte Carlo simulation decreases in proportion to nf-0.5, where nf is the number of
trials in which failure occurs (or the number of sample points in the failure region). Obviously, nf is
directly proportional to the probability of failure and if the latter is of order of 10 -6 – 10-5, which is often

Safety, Risk and Reliability, Heriot-Watt University 146


6. Monte Carlo Simulation
the case in structural reliability problem, on average the number of simulation trials N required to
obtain at least one point in the failure region should be of order of 10 5 - 106. The COV represents the
variance of the estimate of Pf and it is desirable to reduce it to at least 10% if we want to obtain a
sufficiently accurate estimate of Pf. This means that for Pf of order 10-6 – 10-5 the required number of
trials in crude Monte Carlo simulations should be of order 10 7 – 108. Depending on the complexity of the
limit state function, which should be evaluated at each simulation trial, the computational process may
become too time-consuming and not feasible to use. The efficiency of Monte Carlo simulation can be
improved if to find ways to generate more sample points in the ‘important’ failure region, which should
allow decreasing the variance of the estimate of Pf without using very large number of simulation trials.
A number of techniques for achieving this, known as variance reduction techniques, have been proposed
(e.g., Rubinstein 1981). In this course only one such technique – importance sampling, will be
considered.

The main idea of this technique consists of introducing an importance sampling PDF hX(x), which will
generate sample points in the parts of the failure region that are of most ‘importance’. The probability
of failure integral, Eq. (4.11), is then becomes

f X x  f x 
Pf   hX x dx   I x  X hX x dx (6.19)
  h x 
G X 0 X
hX x 

where fX(x) is the original PDF of the basic random variables and I(x) is the indicator function defined by
Eq. (6.3) (it is equal to 1 if G(x) ≤ 0 and 0, otherwise). The integral in Eq. (6.19) equals the mean of
I(x)fX(x)/hX(x); hence, an unbiased estimate of Pf can be obtained by Monte Carlo simulation with values
of the random variables generated according to hX(x) as

 
 I x   
1 N
f X x( j)
Pˆ f  ( j)
(6.20)
N j 1 hX x ( j )

Similar to Eq. (6.5a) the variance of this estimate can be expressed as

 ( j ) f X x ( j )  ˆ
2

    I x  h x ( j )   Pf

N
1
Var Pˆ f   (6.21)
N N  1 j 1  X 

With N→∞, Pˆ f  Pf and the variance becomes zero when the term in brackets is zero, i.e., the

sampling PDF is

I x  f X x 
hX x   (6.22)
Pf

Safety, Risk and Reliability, Heriot-Watt University 147


6. Monte Carlo Simulation
This ideal sampling PDF is non-zero only in the failure region and using it the exact failure probability can
be obtained with a single sample point. However, in order to obtain the ideal sampling PDF the
probability of failure needs already to be known, i.e., Eq. (6.22) is only of conceptual interest.

In any case, Eq. (6.21) shows that the variance can be reduced provided a good choice of the
sampling PDF. From (6.22) it can be inferred that a ‘good’ sampling PDF is the one that generates sample
points in the region of the failure domain, where the original PDF is the largest. To define such sampling
PDF prior information is needed. Such information can be obtained using other methods (e.g., FORM) or
gained step by step through the Monte Carlo simulation process.

The simplest way to construct a ‘good’ sampling PDF is to use a design point obtained by the FORM,
which is the point of the maximum value of the original PDF in the failure region, as mean of the
sampling PDF. Thus, the sampling PDF is the original PDF shifted to be centred at the design point. Since
this approach relies on information obtained by the FORM it can only be employed to check the
estimate of the probability of failure yielded by the latter.

Another approach, called adaptive importance sampling, is based on gaining information through the
simulation process, i.e., it starts as usual crude Monte Carlo simulation and then information obtained
from simulation trials is used for ‘adaptation’ of the original PDF. This approach is not considered herein
in more detail. Other commonly used variance reduction techniques are stratified sampling, Latin
hypercube sampling, conditional expectation method and directional simulation.

Worked Example 6.3

Estimate the probability of failure of the tie-bar from Worked Example 6.2 using importance
sampling with the sampling PDF centred around the design point found in Worked Example 5.10: A*=79
mm2, fy*=286 N/mm2 and W*=22595 N. Keep the same standard deviations as in the original PDF.

Solution

The sampling distribution function hX(x) centres around the design point. Thus, formulae for
generating the basic random variables:

      
- A( j )  A *  A  2 ln u1( j ) sin 2u 2( j )  79  7  2 ln u1( j ) sin 2u 2( j ) 
- parameters of the sampling lognormal distribution for fy:

f
*  ln  f y *  ln(286)  5.656;
25
*  y
  0.0874 so that
fy * 286

   
f y( j )  exp 5.656  0.0874  2 ln u1( j ) cos 2u 2( j ) 
Safety, Risk and Reliability, Heriot-Watt University 148
6. Monte Carlo Simulation
- parameters of the sampling Gumbel distribution for W:

0.5772157 0.5772157
 n *   n  8.55  10 4 ; u n *  W *   22595   21920
n * 8.55  10 4

W ( j )  21920 
1
8.55  10 4
  
ln  ln u3( j )

The COV of the estimate of Pf is calculated as

   
1 1
2
  Pˆ
   ( j) f X x( j)
N
Var Pˆ f   2
Pˆ f
 
N 1  N
  I x
j 1  hX x ( j )
 Pˆ f2  and COVPˆ  f
 Pˆ f

f

where hX(x) is the sampling PDF.

Calculations are carried out in Excel using a Visual Basic subroutine. An Excel file with the subroutine
is available from the course website. Results are shown below. Note that the estimate of the probability
of failure and COV_Pf obtained by Monte Carlo simulation are random numbers, which change slightly
for different runs.

The calculated estimate of Pf is 8.67×10-5 and its COV=0.0405. The result was obtained with just
10,000 simulation trials, i.e., 1000 times fewer trials than in Worked Example 6.2b using crude Monte
Carlo simulation. However, in order to achieve this information about the design point from the FORM
(Worked Example 5.10) was needed.

Safety, Risk and Reliability, Heriot-Watt University 149


6. Monte Carlo Simulation
6.8 Handling correlated random variables
It is easy to handle correlated variables in Monte Carlo simulation using the transformations
described in Section 5.5.

Exercises
Exercise 6.1

The two-parameter Weibull distribution is sometimes used to describe the distributions of initial
crack or defect sizes. Its CDF has the following form


FX ( x)  1  exp  x  
Write a formula for generation its random values by the inverse transform method.
1
 1 
Ans. xi   ln 1  u i 
  

Revision Questions
6.1 What are the essential elements of Monte Carlo simulation in structural reliability?

6.2 What are the advantages and disadvantages of using Monte Carlo simulation to estimate
probabilities of failure?

6.3 What does the accuracy of an estimate of the probability of failure obtained by Monte Carlo
simulation depend upon?

6.4 What steps are involved in generating a random value from a Gumbel distribution? From a normal
distribution?

6.5 What problem can occur with pseudo-random numbers?

6.6 What are variance reduction techniques? What are they used for?

6.7 Explain the main idea of the importance sampling.

SUMMARY

 The Monte-Carlo method for estimating probability of structural failure has been explained and the
advantages and disadvantages listed.

 The way to obtain random values for various distributions has been described.

Safety, Risk and Reliability, Heriot-Watt University 150


6. Monte Carlo Simulation
 A possible method of generating pseudo-random numbers has been explained.

 An example has been given showing one cycle of a Monte-Carlo simulation to obtain an estimate of
structural reliability.

 An example has been given of a Visual Basic program that can be used for Monte Carlo simulation
and estimate probabilities of failure.

 Importance sampling has been explained and an example illustrating the technique, including a
Visual Basic program has been given.

References
Box, G.E.P. and Muller, M.E. (1958). A note on the generation of random normal deviates. The Annals of
Mathematical Statistics, 29(2), 610-611.
Matsumoto, M. and Nishimura, T. (1998). Mersenne twister: a 623-dimensionally equidistributed
uniform pseudorandom number generator. ACM Transactions on Modeling and Computer Simulation,
8(3), 3-30.
Rubinstein, R.Y. (1981). Simulation and the Monte Carlo Method. John Wiley & Sons, New York, NY.
Shooman, M.L. (1968). Probabilistic Reliability: An Engineering Approach. McGraw-Hill, New York.

Safety, Risk and Reliability, Heriot-Watt University 151


6. Monte Carlo Simulation
7 LOAD AND RESISTANCE MODELLING

7.1 Introduction
To estimate the reliability of a structure or its components, the engineer must be able to describe in
probabilistic terms loads acting on the structure as well as relevant structural properties. A number of
probabilistic models, which are currently used to describe loads acting on structures and resistance
properties of structural components, will be considered in this chapter. Special attention will be paid to
the use of extreme value distributions for modelling maximum load effects and minimum values of
strength.

7.2 Load modelling

7.2.1 Introduction

The loads acting on structures can be broadly divided into two groups: (i) due to natural phenomena
such as wind, wave, earthquake and snow loads; and (ii) due to man-imposed effects such as dead (or
permanent) and live (sustained and transient) loads. Typical loads acting on a building are shown in
Figure 7.1.

Wind load Snow load

Sustained load

Dead load
Transient load

Figure 7.1 Loads on buildings.

Safety, Risk and Reliability, Heriot-Watt University 152


7. Load and Resistance Modelling
The magnitude of many loads varies with time and location. In principle, such loads should be
modelled as stochastic processes (i.e., as random functions of time and possibly coordinates). In this
case the probability of failure of a structure should be estimated as the first-passage probability (i.e., the
probability of the first occurrence of the event when the stochastic process representing load exceeds
the structure resistance). Evaluation of the first-passage probability usually requires rather complicated
time-dependent probabilistic analysis and is not considered in this module. In the following the time-
integrated approach is used, when the whole lifetime of the structure is considered as a unit and
statistical properties of random variables representing loads should relate to this lifetime (Melchers
1999). Thus, the probability distribution of interest for loads is that for lifetime maximum loads.

The process of constructing a probabilistic model of a particular load typically includes the following
elements:

- identification and definition of random variables which can be used to represent the uncertainties in
loading description;

- choice of an appropriate probability distribution for each random variable; and

- determination of the distribution parameters using available data.

7.2.2 Dead (or permanent) loads

Dead (or permanent) loads considered in design are usually gravity loads due the self-weight of
structural and non-structural elements permanently connected to the structure. Variability of the self-
weights occurs mainly due to variability of the element dimensions and material densities. Because the
individual permanent loads are additive, the variability of the total permanent load is less than that of
the individual elements; this also suggests that the central limit theorem (see Section 3.8) can be
applied. Therefore, dead load is usually modelled by a normal distribution. There is also some evidence
that dead loads are often underestimated (by about 5%). Thus, a mean of dead load is usually taken as
1.05 times its nominal value; a coefficient of variation of dead load is 0.08-0.10. It is usually assumed
that dead load remains constant throughout the lifetime of the structure.

7.2.3 Live loads in buildings

Live loads on floors represent the weight of people, furniture, equipment, movable partitions and
stored objects. Live loads should be modelled since long-term records are not available and there are
many different factors, which may influence these loads. For the purpose of modelling a number of

Safety, Risk and Reliability, Heriot-Watt University 153


7. Load and Resistance Modelling
studies have been undertaken and the following features if live loads have been identified (Melchers
1999):

- live loads consist of sustained loading plus short-term transient (or extraordinary) loading;

- reduction of loading per unit area as the total area being considered increases;

- occupancy changes produced changes in the sustained loading;

- there is variation of loading within rooms;

- variation of loading between rooms shows some dependence;

- there is some dependence between loadings on different floors; and

- different usage of building space produces quite different ‘arbitrary-point-in-time’ loads, which vary
with floor area.

The probabilistic live load models presented in the following sections are mainly based on
recommendations of the Probabilistic Model Code (JCSS).

7.2.3.1 Sustained loads

Sustained loads represent typical weight of people, furniture, equipment, movable partitions and
stored objects. The term ‘sustained’ indicates that the load remains unchanged for a relatively long
period of time and corresponds to a usual situation in a building (i.e., nothing extraordinary).

Probabilistic modelling of sustained loading is usually based on the so-called ‘arbitrary-point-in-time’


load, i.e., on the floor load intensity at some arbitrary location in a building at arbitrary time which is
then converted to an equivalent uniformly distributed load. The ‘arbitrary-point-in-time’ load intensity
W(x,y), where x and y coordinates in the plane of the building floor, can be expressed as

W x, y   m  V  U x, y  (7.1)

where m is the overall mean load intensity for a particular user category (see Table 7.1), V is a normal
random variable with zero mean which represents variability of load between different buildings and
different floors within the same building, and U(x,y) is a zero mean random field with a characteristic
skewness to the right which represents spatial variability for a given floor.

For linear elastic systems, where superposition is possible, the load effect S can be expressed as

S   W x, y ix, y dA (7.2)


A

Safety, Risk and Reliability, Heriot-Watt University 154


7. Load and Resistance Modelling
where i(x,y) is the influence function for the load effect over the considered area A. An equivalent
uniformly distributed sustained load per unit area, Q, i.e., the load which has the same load effect as
W(x,y) is then

 W x, y ix, y dA


Q A
(7.3)
 i( x, y)dA
A

The mean and the variance of the sustained uniformly distributed load are

 Q  E Q  mq (7.4)

 ix, y  dA
2

A0 2
 Q2  Var Q    V2  A
2
 U2   V2   U (7.5)
  A
  i x, y dA
A 
where σV and σU are the standard deviations of V and U(x,y), respectively, A0 is the so-called reference
area, and κ is the factor depending on the shape of the influence function i(x,y). Values of κ are
presented in Figure 7.2, while values of the other parameters are given in Table 7.1. Note that for A<A0
one should take A0/A = 1.

i κ = 1.0 i κ = 1.4

0 1 ξ= x/l 0 1 ξ

η η

κ =2.0 κ =2.4

i 1 i 1

ξ ξ
0 1 0 1

Figure 7.2 Shapes of the influence function and the corresponding values of κ (adapted from JCSS).

Safety, Risk and Reliability, Heriot-Watt University 155


7. Load and Resistance Modelling
It has been found that the sustained live load is best represented by a Gamma distribution, which has
the following PDF and CDF

  q k 1
f Q q   exp   q  (7.6)
( k )

k , q 
FQ q   q ≥ 0, k > 0 (7.7)
( k )

where Γ(k) and Γ(k,νq) are the gamma function and incomplete gamma function, respectively. The
parameters of the Gamma distribution – ν and k, can be found from the mean and standard deviation of
Q (given by Eqs. (7.4) and (7.5), respectively)

k
Q  (7.8)

k
 Q2  (7.9)
2

In the time-integrated approach the distribution of maximum values of the sustained load over the
lifetime of the building is of interest. The time between load changes is assumed to be exponentially
distributed, then the number of load changes is Poisson distributed. The CDF for the maximum
sustained load is given by


FQ,max q   exp  T 1  FQ q   (7.10)

where FQ(q) is the probability function of the sustained load given by Eq. (9.7), T is the reference time
(i.e., the anticipated lifetime of the building), and λ is the occurrence rate of sustained load changes.
Thus λT is the mean of the number of occupancy changes. Values of λ can be found from Table 7.1.

7.2.3.2 Transient loads

Transient (or extraordinary) live loads represent the weight of people, furniture, equipment, and
stored objects that might occur during an unusual event such an emergency, when everybody gathers in
one room, or when all furniture and equipment are stored in one room. Transient loads can be modelled
similar to sustained loads but, of course, with different parameters. The mean and the variance of an
equivalent uniformly distributed transient load P can be found as

 P  E P   m p ;  P2  Var P    V2 (7.11)

Safety, Risk and Reliability, Heriot-Watt University 156


7. Load and Resistance Modelling
where values of mp and σV depending on the user category are given in Table 7.1. The transient load can
be modelled by a Gamma distribution (Eqs. (7.6)-(7.9)) with μQ and σQ replaced by μP and σP.

The probability distribution of the maximum transient loads over the period T may be expressed as

FP,max  p   exp T 1  FP  p  (7.12)

where ν is the mean occurrence rate of the transient load, whose values are given in Table 7.1.

Table 7.1 Parameters for live loads depending on the user category (adapted from JCSS).

The total live load is the sum of the sustained live load and the transient live load. The maximum
total live load, Lmax, corresponding to a reference period T (e.g., the lifetime of the building) can assessed
as the maximum of the following two sums

Lmax  max Qmax  P; Q  Pmax  (7.13)

where Q is the arbitrary-point-in-time sustained load, Qmax the maximum sustained load over the period
T, P the arbitrary-point-in-time transient load, and Pmax the maximum transient load over the period T.
This approach to the evaluation of the maximum live load can easily be implemented in Monte Carlo
simulation.

Safety, Risk and Reliability, Heriot-Watt University 157


7. Load and Resistance Modelling
7.2.4 Wind loads

Wind loads on structures depend on various factors such as wind climate, the exposure, shape,
dimensions and dynamic properties of the building. In accordance to the Probabilistic Model Code (JCSS)
a probabilistic model for wind loads can be formulated as:

- for rigid structures of smaller dimensions

w  ca c g cr Qref  ca ce Qref (7.14)

- for structures sensitive to dynamic effects (natural frequency < 1Hz) and for large rigid structures

w  cd ca ce Qref (7.15)

where

w - wind force acting per unit area of structure,

Qref - reference (mean) velocity pressure,

cr - roughness factor,

cg - gust factor,

ca - aerodynamic shape factor,

cd - dynamic factor, and

ce=crcg – exposure factor.

The reference velocity pressure is defined as

1
Qref  U ref (7.16)
2

where ρ is the air density (ρ=1.25 kg/m3 for standard air) and Ūref is the reference wind velocity. The
latter is defined as the mean velocity of the wind averaged over a time interval of 10 min determined at
an elevation of 10 m above ground, in horizontal open terrain exposure. Ūref is modelled by the Weibull
distribution (see Section 3.6.3) with the scale parameter m close to 2. The annual maximum wind
velocity is modelled by a Gumbel distribution. Since the relationship between the velocity pressure and
the wind velocity is given by Eq. (7.16), the annual maximum velocity pressure is also modelled by a
Gumbel distribution.

According to JCSS, the factors for terrain roughness, gust effects and aerodynamic shape can be
treated as lognormal random variables. Their coefficients of variation are summarised in Table 7.2.

Safety, Risk and Reliability, Heriot-Watt University 158


7. Load and Resistance Modelling
The coefficient of variation of the wind force acting per unit area of the structure can then be
estimated as

- for rigid structures of smaller dimensions

COVw2  COVc2a  COVc2g  COVc2r  COVQ2ref (7.17)

- for structures sensitive to dynamic effects

COVw2  COVc2d  COVc2a  COVc2g  COVc2r  COVQ2ref (7.18)

For more detailed description of the model see JCSS.

Table 7.2 Statistical properties of random variables describing wind load.


Variable Distribution type COV
Qref Gumbel 0.20 – 0.30
cr Lognormal 0.10 – 0.20
ca pressure coefficient Lognormal 0.10 – 0.30
force coefficient Lognormal 0.10 – 0.15
cg Lognormal 0.10 – 0.15
cd Lognormal 0.10 – 0.20

7.2.5 Wave loads

For marine structures perhaps the most important source of loading is that due to extreme storms
waves. The usual practice when designing offshore structures is to consider the most extreme or
‘design’ storm. This may be the largest storm in the predicted lifetime for the structure or it may be the
‘100 year storm’. It has been found that the Rayleigh distribution represents the heights of storm waves
quite well. The peak of the extreme storm is usually considered to last 3 hours and what is required for
design purposes is the largest wave in this period. This is usually known as the design wave, or the 100
year wave if the 100 year storm is considered. In a storm the typical average wave period may be
anything from around 8 seconds to 18 seconds depending on the location. So in 3 hours one would
expect to get somewhere between about 600 and 1350 waves. Typically we are looking for the largest of
the 1000 or so waves that occurs in the extreme storm.

The Rayleigh distribution for wave heights, h, is given by

4h  2h 2 
f H ( h)  exp  2  (7.19)
H S2  H 
 S 

Safety, Risk and Reliability, Heriot-Watt University 159


7. Load and Resistance Modelling
Here HS is the significant wave height, a parameter that is used to characterise sea-states (and storm
intensity) by calculating the average of the one third highest waves. Figure 7.3 below shows this
distribution (PDF) of wave heights for Hs= 10m.

0.14

Probability density (m-1)


0.12
0.10
0.08
0.06
0.04
0.02
0
0 5 10 15 20 25
Waveheight (m)

Figure 7.3 The Rayleigh distribution for individual wave heights when the significant wave height Hs=
10m.

The CDF of the Rayleigh distribution is simply the integral of Eq. (7.19), i.e.

 2h 2 
FH (h)  1  exp   2  (7.20)
 HS 

Consider now the wave whose height has only a 1 in n chance of being exceeded (i.e., the probability
of being exceeded equals 1/n), where n is the number of waves. That means the probability of a wave of
being less than or equal to this height is 1-1/n. Thus we can write

 2h 2 
1  1 / n  FH (h)  1  exp   2 
 HS 

and simply rearranging this we get

ln n
h  HS (7.21)
2

This gives us the ‘most likely’ extreme wave height or the so-called ‘return period’ wave height when
there are n waves under consideration. For example suppose there are 1000 waves in a storm where HS
is 10 metres, then we get h = 1.86HS = 18.6 metres.

Worked Example 7.1

Obtain the extreme value distribution (both probability density function and cumulative distribution
function) for the largest wave in a 3-hour storm of significant wave height 10 metres and mean wave

Safety, Risk and Reliability, Heriot-Watt University 160


7. Load and Resistance Modelling
period 10.8 seconds. Find the wave height that this largest wave has 5% chance of exceeding. Plot both
wave height PDF and the extreme value PDF.

Solution

3  60  60
In 3 hours we get n   1000 waves.
10.8

We are interested in the highest waves, thus we need to substitute Eq.(7.20) into Eq. (3.66) (of the
extreme value distribution of maximum values) and obtain

1000
  2h 2 
FH1000 (h)  1  exp   2  (7.22)
  HS 

To find a wave height which has only a 5% chance of being exceeded we require that FH1000 (h)  0.95 ,

i.e.

1000
  2h 2 
0.95  1  exp   2 
  HS 

This expression can be rearranged so h is the subject of the formula. First we write

1  0.95   exp   2Hh 


2

1
1000
2 
 S 


and then h  H S  0.5 ln 1  0.95
1
   2.22 H  22.2 metres .
 
1000
  S

The PDF of the distribution of the extreme wave heights is obtained by differentiating Eq. (7.22) with
respect to h

999
dFH1000   2h 2   4h  2h 2 
f H1000 (h)   10001  exp   2   2 exp   2 
dh   H S  HS  HS 

The plot of the Rayleigh distribution and the corresponding extreme value distribution for the largest of
1000 waves is shown below in Figure 7.4.

Safety, Risk and Reliability, Heriot-Watt University 161


7. Load and Resistance Modelling
0.30
Return period/

Probability density (m )
0.25

-1
most likely value
0.20

e value
0.15
Ra
yle
0.10 i

Extrem
gh
0.05

0
0 5 10 15 20 25
Waveheight (m)

Figure 7.4. The probability density of wave heights and extreme wave heights.

Note: In Figure 7.4 it is seen that the maximum density of the extreme value distribution occurs when
h=18.6 metres. This is the same value as we obtained when we considered the most likely extreme wave

Also note that the "100 year wave" has a 62.8 % chance of being exceeded in a 100 year period (see
Exercise 7.1). The value of 62.8% occurs because the probability density function is not symmetrical
about the maximum density but is skewed to the right as can be seen in Figure 7.4. This is one of the
limitations of the deterministic approach to extreme load estimation.

The Rayleigh distribution can be used to represent the magnitude of the peak values of a whole
range loadings and responses as long as they can be assumed to be part of a Gaussian random linear
process. For example, pressure surges, vibrations and weight loads often have Gaussian distributions by
virtue of the Central Limit Theorem. Hence, the Rayleigh distribution and related extreme value
distributions are widely used in the structural assessment of marine structures and systems.

Worked Example 7.2

A structural reliability assessment is being undertaken on the hull girder of an old ship as there is
concern that it might ‘break its back’ in a storm because it has suffered a deterioration in strength due
to corrosion. The moment of resistance, Mr, of the corroded hull is log-normally distributed with a mean
value of 20 MNm and a coefficient of variation of 0.15. The hull must withstand the combined effects of
a still water bending moment and a wave bending moment. The maximum wave bending moment, MW,
can be estimated from MW =C×H, where C is a normally distributed coefficient with a mean value of 0.3
MN and a standard deviation 0.018 MN, and H is the wave height described by a Rayleigh distribution.
The worst storm for the remaining life is estimated to have a significant wave height of 12 m and it is
estimated there will be about 1000 waves during the 3-hour duration of this storm. The still water
bending moment MS is estimated to have a normal distribution with a mean of 4 MNm and a standard

Safety, Risk and Reliability, Heriot-Watt University 162


7. Load and Resistance Modelling
deviation of 0.8 MNm. Write a program to perform a Monte Carlo simulation to estimate the probability
of the ship ‘breaking its back’ in the biggest wave in the worst storm.

Solution

In this case the limit state function is

GX  M r  C  H  M S

The wave height is described by a Rayleigh distribution

 2h 2 
FH (h)  1  exp   2 
 HS 

where HS is the significant wave height. We need the corresponding extreme value distribution for the
largest of 1000 waves that occur in the worst storm, i.e.,

1000
  2h 2  
FH1000 (h)  1  exp   2 
  H S 

Thus, the weight height can be generated as

h  j   H S  0.5 ln(1  [u1 j  ])  12  0.5 ln(1  [u1 j 


1 1
1000 1000
])

where u1 is a random number uniformly distributed on [0,1).

The other variables are all normally and log-normally distributed and are treated as in Worked Example
6.2.

Parameters of the lognormal distribution of Mr:

  COVM  0.15 ,   ln  M  ln 1  COVM2   ln 20   0.15 2  2.9845


1 1
r r
2 r
2


M r( j )  exp 2.9845  0.15  2 ln u 2( j )  cos2u3( j )  
C ( j )  0.3  0.018  2 ln u 2( j )  sin2u3( j ) 

M s j   4  0.8  2 ln u 4( j )  cos2u5( j ) 

Safety, Risk and Reliability, Heriot-Watt University 163


7. Load and Resistance Modelling
The Excel spreadsheet with the Visual Basic coding for this worked example is available from the
course website. The Excel spreadsheet page is presented above. This shows the worksheet heading, the
Start Simulation button and the values of the variables returned at the end of the simulation. In this case
there are 1,000,000 cycles resulting in 398 failures giving an estimated probability of failure of 3.98×10-4.
The coefficient of variation of the estimate of Pf calculated based on Eq. (6.5), COV_Pf = 0.050. If to run
the program again, a slightly different estimate of Pf will be obtained. The accuracy will improve if to
increase the number of simulation trials.

7.2.6 Extreme values of repeated loads

Extreme value distribution of the largest values, Eq. (3.66), can be applied in principle to all
distributions. However, the CDF of a normal distribution cannot be expressed in an analytical form but
can only be evaluated numerically as the integral of its PDF. Thus, the use of Eq. (3.66) with a normal
distribution is not very convenient. Instead we can use an asymptotic extreme value distribution which
avoids the problems of numerical integration and gives a simple approximation to the extreme value
distribution. The term asymptotic means the distribution becomes an increasingly accurate
representation of the exact distributions as n  . In real problems the error involved in assuming the
approximate asymptotic form is almost always small compared with errors associated with the limited
samples of data upon which our statistics are based. Certainly this is true when n is around 1000 or
more.

We know that the Gumbel distribution (also known as the Extreme value Type 1 distribution of the
largest values) can serve as an asymptotic distribution. Its CDF and PDF are (see Eqs. (3.70) and (3.71)

FYn  y   exp  exp   n  y  un  (7.23a)

Safety, Risk and Reliability, Heriot-Watt University 164


7. Load and Resistance Modelling
f Yn  y    n e  n  y un exp - exp   n  y  u n  (7.23b)

When an individual random variable has a standard normal distribution, the parameters of the Gumbel
distribution approximating the distribution of its n largest values can be found as

ln(ln n) + ln 4π
α n  2 ln n ; u n  2 ln n  (7.24)
2 2 ln n

Note: these two parameters only depend on the sample size n. The corresponding mean value and the
variance of the Type I distribution are

 2
Y  un  and Y 2

n
n n
6 n
2

where  = 0.577216 and is known as Euler's number.

In the more general normal form the corresponding expressions can be shown to be

 γ 
2
π 2 .σ y
μ Yn  μ y   u n  σ y and σ Yn 
2

 αn  6αn2

The CDF is then

   
FYn  y   exp  exp   n  y   y  u n y  (7.25)
   
 y 

and the PDF is

 α    α 
exp  n  y  μ y  u n σ y  exp - exp  - n  y  μ y  u n σ y  (7.26)
αn
f Yn  y  
σy  σ y    σ 
 y 

Worked Example 7.3

A light hoist cable has a demand D given by N(5.886, 0.6) (i.e., normal distribution with mean 5.886
kN and standard deviation 0.6 kN) for each load cycle. Determine the mean and standard deviation of
the corresponding Type 1 extreme value distribution for the maximum demand in 1000 load cycles, and
find the probability that this maximum load will exceed 10kN.

Solution

ln(ln n) + ln 4π
For n =1000 : αn  2 ln n  3.7169 and u n  αn   3.116 .
2αn

Safety, Risk and Reliability, Heriot-Watt University 165


7. Load and Resistance Modelling
 0.577216 
Therefore, μYn  5.886   3.116  0.6  7.849 kN.
 3.7169 

π 2  0.6 2
and σ 2
 0.0429 , so that  Yn  0.207 kN.
6  3.7169 2
Yn


Hence FYn 10  exp  exp  
 3.7169
10  5.886  3.116 x 0.6  1  9.156 x 10 7
  0.6 

and therefore Px1000  10  9.156 x 10-7 i.e. less than one chance in a million of exceeding a load
of 10kN.

It is worth noting that in this example the mean of the extreme value distribution for the maximum
of 1000 cycles was 33% higher than the underlying "parent" distribution; however, the standard
deviation was 65% smaller. For the extreme value distributions as n   the standard deviation tends
to zero.

Worked Example 7.4

The Monte Carlo method is to be used to evaluate the probability of failure corresponding to the
limit state function shown below. This limit state function is for a nominal 6-mm diameter solid steel tie-
bar that may fail under excessive tensile loading


GX  D 2 f ult  L (N)
4

where D is the tie bar diameter which is normally distributed with mean μD=6 mm and standard
deviation σD=0.01 mm, fult is the tensile strength of the material of the tie- bar that is log-normally
distributed with the parameters λ=6 and ξ=0.06. The loading L is the largest load from 2000 loading
cycles drawn for a normal distribution with mean of 5000 N and standard deviation of 1000 N.

Solution

The loading distribution will be an Extreme Value Type 1 distribution with the following parameters

αn  2 ln n  2 ln 2000  3.90 and

ln(ln n) + ln 4π ln ln 2000  ln 4


un  2 ln n   3.90   3.32
2 2 ln n 2  3.90

Using the following independent random numbers u1 = 0.17, u2 = 0.48, u3 = 0.727 one simulation trial
can be performed as shown below.

Safety, Risk and Reliability, Heriot-Watt University 166


7. Load and Resistance Modelling
D (1)  6  0.1  2 ln 0.17 cos(2  0.48)  5.81 mm
 
f ult(1)  exp 6  0.06  2 ln 0.17 sin(2  0.48)  409 N/mm 2
L(1)   L
ln  ln u 3(1)   3.32  ln  ln 0.727   3.61
1 1
 un 
L n 3.90
L (1)
 3.61   L   L  3.61  1000  5000  8610 N
  5.812  409
therefore G X    8610  2233 N  0 - no failure
(1)

In order to obtain a sufficiently accurate estimate of Pf millions more trials need to be run with the help
of a computer (of course, a computer program should be written for that).

7.3 Resistance modelling

7.3.1 Introduction

The load-carrying capacity of a structure depends on the resistance of its components and
connections. The resistance of a component is generally a function of material(s) strength, section
geometry and dimensions. Although in design these quantities are often considered deterministic, in
reality there is some uncertainty associated with each quantity. Therefore, the resistance is a random
variable.

The possible sources of uncertainty associated with resistance include:

- Uncertainty in material properties (e.g., strength, modulus of elasticity, ultimate strain);

- Uncertainties in dimensions of the components, which affect the cross-section area, moment of
inertia, and the section modulus;

- Model uncertainty, i.e., uncertainty resulting from approximate models used in analysis.

The process of constructing a probabilistic model of a particular resistance parameter includes the same
elements as for loads (see Section 7.2.1).

7.3.2 Structural steel

For probabilistic modelling of the material properties of rolled structural steel sections a model
proposed in the Probabilistic Model Code (JCSS) can be used. The model is applicable to structural steels
with yield stresses up to 380 N/mm2, i.e., to S235, S275 and S355. Statistical description of the steel
properties used in the model is presented in Table 7.3.

Safety, Risk and Reliability, Heriot-Watt University 167


7. Load and Resistance Modelling
Table 7.3 Statistical description of material properties of rolled steel (adapted from JCSS).

Description Variable Distribution type Mean COV


Yield stress fy Lognormal  uCOVf y 0.07
f y , spe C
Ultimate stress fu Lognormal B f y 0.04

Modulus of Es Lognormal Es,sp 0.03


elasticity
Poisson’s ratio ν Lognormal νsp 0.03
Ultimate strain εu Lognormal εu,sp 0.06

The subscript ‘sp’, which appears in Table 7.3, stands for a specified or nominal value of the
parameter provided by the code. The coefficients of variation given in Table 7.3 have been assessed on
the basis of European studies from 1970 onwards. The factor α is a spatial position factor, which equals
1.05 for webs of hot rolled sections and 1 otherwise. The factor u depends on the quantile value
corresponding to the nominal yield stress and normally is in the range from -2.0 to -1.5.

C is a constant reducing the yield strength as obtained from usual mill tests (which are conducted at a
higher strain rate than is normal for conventional ‘quasi-static’ loads on structures) to the static yield
strength; a value of 20 N/mm2 is recommended but attention should be given to the rate of loading
used in the mill tests. The factor B depends on the type of steel: B=1.5 for structural carbon steel, 1.4 for
low alloy steel, and 1.1 for quenched and tempered steel.

Within-batch COVs can be taken as one fourth of the values given in Table 7.3, while within-batch
variability of the modulus of elasticity, Es, and Poisson’s ratio can be neglected. Variations along the
length of a rolled section are normally small and can be neglected.

A number of steel properties appearing in Table 7.3 are considered to be correlated. The coefficients
of correlation between the properties are given in Table 7.4

Table 7.4 Correlation matrix of the properties of structural steel (adapted from JCSS).
fy fu Es ν εu
fy 1 0.75 0 0 -0.45
fu 1 0 0 -0.60
Es 1 0 0
ν Symmetry 1 0
εu 1

Safety, Risk and Reliability, Heriot-Watt University 168


7. Load and Resistance Modelling
7.3.3 Reinforcing steel

The sources of variability and the physical properties of interest for reinforcing bars are quite similar
to those of rolled structural steel. A major study of statistical properties of reinforcing steel was carried
out by Mirza and MacGregor (1979).

There is negligible variation of yield stress and ultimate strength within the length of a typical
reinforcing bar. Thus, full correlation of the steel properties along the length of a reinforcing bar can be
assumed. For reinforcing bars of the same size and in the same job lot it is likely that the bars originate
from the same steel mill, in which case according to European data the coefficient of variation of the
yield stress of steel is about 0.01-0.04 and the coefficient of correlation between the yield stress of
individual bars is around 0.9 (JCSS). The COV for reinforcing bars from different sources and in different
locations in the structure is about 0.04-0.07.

Variability of reinforcing bar sizes typically is small, with the ratio of actual area to nominal area
having a mean of 1.00 and a COV of around 2%. There is an effect on yield stress and ultimate strength
resulting from the rate of specimen testing at steel mills, similar to that noted for structural steel.
According to Mirza and McGregor (1979), the mean yield stress of Grade 300 reinforcing bars is 310
N/mm2 and standard deviation 35 N/mm2; for Grade 410 these parameters are 461 N/mm2 and 38
N/mm2, respectively. The modulus of elasticity of reinforcing steel was considered to have a mean of
2.01×105 N/mm2 with a COV of 0.033.

Mirza and McGregor (1979) suggested using a Beta distribution for yield stress. In the Probabilistic
Model Code (JCSS) the use of a lognormal distribution is recommended.

7.3.4 Concrete

Variability of the compressive strength of concrete is usually has much smaller influence on structural
resistance than the properties of reinforcing steel. This is due to design philosophy of attempting to
ensure ductility in the structure. Nevertheless, it is important for estimating the reliability of reinforced
concrete elements subject mainly to compression (e.g., columns) and for serviceability investigations.

Based on many test results for cast ‘on-site’ concrete test cylinders and cubes the values of COV or
standard deviation given in Table 7.5 are appropriate for between-batch variation (i.e., considering
concrete from different sources). The COVs are roughly halved for within-batch variation (i.e., for
concrete from one source). It is evident that quality control of concrete is an important parameter.

Safety, Risk and Reliability, Heriot-Watt University 169


7. Load and Resistance Modelling
Table 7.5 Variation of ‘on-site’ concrete compressive strength for control cylinders and cubes (adapted
from Melchers (1999)).
Control COV (fc < 28 N/mm2) Standard deviation
(28 ≤ fc ≤ 50 N/mm2)
Excellent 0.10 2.8 N/mm2
Average 0.15 4.2 N/mm2
Poor 0.20 5.6 N/mm2

For reliability assessments, the in-situ compressive strength of concrete is of most interest, rather
than results for control cylinders and cubes. For concrete compressive strength the relationship
between in-situ strength fcis and the characteristic (of specified) strength fc may be taken as (Mirza et al.
1979):

 f  0.675 f c  7.7  1.15 f c (N/mm 2 )


cis
(7.27)

COV f2cis  COV f2c  0.0084 (7.28)

where COVfc can be taken (or estimated) using data from Table 7.5. A lognormal distribution can be
adopted for the compressive strength of concrete.

Dimensional variability is also of interest for reinforced concrete structures. In most cases it has been
found that the actual thickness of slabs is greater than their nominal thickness by ratios varying up to
about 1.06 with a COV of about 0.08; for high quality bridge decks and precast slabs these values are
1.005 and 0.02, respectively (Melchers 1999).

In contrast, the effective depth to the reinforcement for in-situ slabs is generally less than specified,
in the range (actual/nominal) 0.93-0.99 with a COV of 0.08. There is some evidence that this variability is
considerably smaller in good-quality work and that in precast slabs the deviation and variability are
almost negligible.

7.3.5 Use of extreme value distributions for resistance modelling

For a chain the strength is governed by the weakest link. Similarly for a corroded tie bar or a pipe the
strength is governed by the minimum remaining cross section or the minimum remaining wall thickness.
In these cases the minimum extreme value is needed to estimate strength. A general expression for the
extreme distribution of minimum values is given by Eq. (3.68). Differentiating it yields

f Z ( z )  n1  FX z  f X z 
n 1
(7.29)

There are a set of distributions for minimum extreme values which correspond directly to those for
maximum extreme values. Note that the subscript 1 in the following is used to denote the minimum

Safety, Risk and Reliability, Heriot-Watt University 170


7. Load and Resistance Modelling
value, i.e. the first in the rank order where previously n was used to denote the largest. The Type I
distribution is given here

FZ1 z   1  exp  exp 1 z  u1  for    z   (7.30)

and for a standard normal parent distribution

ln(ln n)+ ln 4π
α1  2 ln n and u1  α1  (7.31)
2α1

The mean and variance for a minimum Type I when the underlying distribution is a standard normal are
given by

0.5772 2
 Z  u1  and  Z21  (7.32)
1
1 612

and for an underlying normal distribution with mean x and standard deviation x then

 0.5772  π2 2
μZ1  μ x  σ x  u1   and σ Z21  σx (7.33)
 α1  6α12

Worked Example 7.5

Round poles from forestry ‘thinning’ are to be used for the construction of a truss-work bridge. There
is concern about the stiffness of the poles and a random sample of 30 poles, from the 300 to be used,
are tested to see how much they deflect in bending under a central load and their stiffness (EI – flexural
rigidity) calculated. The sample mean stiffness is found to be 20 kNm2 with a standard deviation of 3
kNm2. The deflections seem to have a normal distribution when plotted. Find the corresponding
minimum extreme value distribution for the least stiff of the 300 beams and its mean and standard
deviation. What is the chance this beam will have a flexural rigidity of less than 10 kNm 2? Find the
flexural rigidity for which there is a just a 5% chance of being lower than?

Solution

Using the equations above α1  2 ln n  2 ln 300  3.378

ln(ln n)+ ln 4π ln(ln 300)  ln 4


and u1  α1   3.378   2.746
2α1 2  3.378

Hence the minimum extreme value distribution is

    Z  20   
FZ1 Z   1  exp  exp  3.378    2.746  
   3   

Safety, Risk and Reliability, Heriot-Watt University 171


7. Load and Resistance Modelling
Substituting in 10 for Z we find

    10  20   
FZ1 z   1  exp  exp  3.378    2.746   = 0.128
   3   

so there is about a 13% chance that the most flexible pole will have EI less than 10 kNm2.

    Z  20   
For the last part of the question 0.05  1  exp  exp  3.378    2.746  
   3   

which needs to be rearranged to find Z. Straightforward algebraic manipulation gives

ln  ln 0.95 Z  20
 2.746 
3.378 3

Hence z = 9.1 kNm2 i.e. there is only a 5% chance that the most flexible timber pole will have a flexural
rigidity less than this.

Exercises
Exercise 7.1

A 3-hour storm has significant wave height 10 metres and mean wave period 10.8 seconds. What is
the ‘most likely’ extreme wave height in this storm? What is the probability that the extreme wave
height during this storm exceeds this ‘most likely’ extreme wave height?

Note that this result is the general result for the probability of exceeding the most likely extreme
value of wave height for all extreme value distributions as n becomes large.
Ans. 0.628

Exercise 7.2

A pressure vessel is subject to 2000 loading cycles during its lifetime. The individual loads are
normally distributed with a mean value of 20 bar (1 bar = 105 Nm-2) and a standard deviation of 3.5 bar.
Find the corresponding extreme value distribution for the maximum of 2000 load cycles. What is the
probability that the extreme loading will exceed 35 bar? What pressure has only a 5% chance of being
exceeded during the largest pressure cycle (Hint – the formula for the extreme value CDF must be
rearranged to make y the subject)?
Ans. 0.0225; 34.27 bar.

Exercise 7.3

Safety, Risk and Reliability, Heriot-Watt University 172


7. Load and Resistance Modelling
A light hoist is designed with a cable that has a strength S with a mean of 12 kN with a standard
deviation σS = 2.4 kN which is assumed to be normally distributed. The hoists are all marked "maximum
load - 500kg" but experience shows this is frequently ignored and the estimated load is assumed to be
normally distributed with a mean value of 6 kN (about 600kg) with a standard deviation σL = 0.6 kN. The
hoist is expected to make about 12000 lifts during its life. Estimate the probability that it will fail under
the heaviest of these 12000 loads using Monte Carlo simulation.

(a) Perform one simulation trial manually with the following random numbers: u1 = 0.0872, u2 =
0.4807, u3 = 0.9421.

(b) Adapt the Visual Basic code from either Worked Example 6.2b or Worked Example 7.2 and run the
program with sufficient number of simulation trials to ensure that there are at least 100 failures
during the simulation.
Ans. Pf ≈ 0.063

Revision Questions
7.1 Give a brief description of the main types of loads acting on buildings.

7.2 What are the main elements of constructing a probabilistic load model?

7.3 What distribution can be used to model extreme values of repeated loading?

7.4 What are possible sources of uncertainty in modelling structural resistance?

7.5 What extreme values of resistance are usually of interest? What distributions can be used for
their modelling?

SUMMARY

 Different types of loads acting on structures have been described and their probabilistic models
have been presented.

 The use of an extreme value distribution for modelling repeated loads has been explained.

 Different sources of uncertainty associated with modelling structural resistance have been
described. Probabilistic models for different resistance parameters of steel and reinforced concrete
structures have been presented.

 The use of an extreme value distribution for modelling structural resistance has been explained.

Safety, Risk and Reliability, Heriot-Watt University 173


7. Load and Resistance Modelling
References
JCSS. Probabilistic Model Code. Joint Committee on Structural Safety.
http://www.jcss.ethz.ch/publications/publications_pmc.html
Melchers, R.E. (1999). Structural Reliability: Analysis and Prediction. 2nd Ed., John Wiley & Sons.
Mirza, S.A. and MacGregor, J.G. (1979). Variability of mechanical properties of reinforcing bars. Journal
of Structural Division, ASCE, 105(ST5), 921-937.
Mirza, S.A., Hatzinikolas, M. and MacGregor, J.G. (1979). Statistical description of strength of concrete.
Journal of Structural Division, ASCE, 105(ST6), 1021-1037.

Safety, Risk and Reliability, Heriot-Watt University 174


7. Load and Resistance Modelling
8 RELIABILITY ANALYSIS OF STRUCTURAL SYSTEMS

8.1 Introduction
In the previous chapters we have considered how to calculate the safety index (or the probability of
failure) when failure of the structure (or, more exactly, its component) is defined by a single limit state
function. We have also discussed possible probabilistic models for individual loads and structural
properties. This chapter will address the problems of combining individual loads acting on a structure
into the total load and evaluating the probability of failure of a structure when it is associated with
failure of a number of components or/and with several possible failure modes (i.e., several limit state
functions).

8.2 Load combinations


The total load acting on a structure at any point in time is usually a combination of various individual
loads (e.g., dead, live, wind, snow, etc). In the time-integrated approach probabilistic modelling of the
maximum total load over the lifetime of a structure is of interest. In principal, the maximum total load
acting on the structure within a reference period T (e.g., the structure lifetime), Xmax(T), may be assessed
as the maximum of the sum of individual loads, Xi (i=1,2,…,n), i.e.,

X max T   max X 1 t   X 2 t     X n t  (8.1)


T

Unfortunately, since individual loads vary with time (e.g., see Figure 8.1) and this variation is unknown a
priori, it is very difficult and, in most cases, impossible to obtain the probability distribution of the
maximum. A conservative approach is to assume that

X max T   max X 1 t   max X 2 t     max X n t  (8.2)


T T T

However, it is most unlikely that all maxima will occur simultaneously at the same point in time so that
this approach will grossly overestimate the maximum total load. A number of more realistic
approximate approaches to solve Eq. (8.1) have been proposed and one of them, the Turkstra’s rule
(Turkstra and Madsen 1980), will be described further in this section.

As mentioned previously, it is highly unlikely that all individual loads will attain their maximum values
at the same time. According to the Turkstra’s rule, for each individual load Xi two distributions are
needed:

Safety, Risk and Reliability, Heriot-Watt University 175


8. Reliability Analysis of Structural Systems
Permanent load

Time

Sustained live load

Several years

Time
Transient live load Several months or days

Time
Snow load
Winter months

Time
Wind load
Minutes

Time
Figure 8.1 Temporal variations of different load acting on structures.

- the distribution for the arbitrary-point-in-time value with the CDF

FX i xi   P X i  xi at any moment of time (8.3)

- the distribution for the maximum value within a reference period T with the CDF
Safety, Risk and Reliability, Heriot-Watt University 176
8. Reliability Analysis of Structural Systems
FX i ,T xi   Pmax X i  xi in T years (8.4)

It is assumed that the probability of occurrence of maximum value of any two individual loads at the
same time is negligible. The total maximum load within the reference period is then found as the
maximum of the following n load combinations

max X t   X t     X t 
 T 1 2 n

X max T   max  X 1 t   max X 2 t     X n t  (8.5)


 X t   X t     max X t 
T

 1 2
T
n

where the distribution of Xi(t) is described by Eq. (8.3) and the distribution of max X i t  by Eq. (8.4).
T

The mean of Xmax(T) can now be calculated as

 max, X1   X 2     X n

X max
 max  X1   max, X 2     X n (8.6)
      
 X1 X2 max, X n

For the combination that yields the largest mean value we can also calculate the variance. For example,
if the k-th combination yields the largest mean value then assuming that the individual loads are
uncorrelated the variance is

n
 X2 max
  max,
2
Xk  
i 1,i  k
2
Xi (8.7)

Worked Example 8.1

Individual loads acting on the structure include permanent, live and wind loads, which are described
by the following distributions:

- The permanent load is time invariant, thus, maxD and D are the same. Their distribution is normal
with a mean μD=20 and COVD=0.10 so that σD=2.

- The arbitrary-point-in-time live load L follows a Gamma distribution with μL=9 and COVL=0.31 so that
σL=2.8;

The maximum live load maxL is described by the Gumbel distribution with μmax,L=30 and COVmax,L=0.12
so that σmax,L=3.6.

- The arbitrary-point-in-time wind load W follows a lognormal distribution with μW=1 and COVW=0.60
so that σW=0.6;

Safety, Risk and Reliability, Heriot-Watt University 177


8. Reliability Analysis of Structural Systems
The maximum wind load is described by the Gumbel distribution with μmax,W=24 and COVmax,W=0.20 so
that σmax,W=4.8.

Calculate the mean and standard deviation of the total load using Turkstra’s rule.

Note that this example is for illustrative purpose only and the above parameters of the individual
loads do not represent those of actual loads acting on structures; the units of the load values are not
given by the same reason.

Solution

According to Turkstra’s rule the following three load combinations need to be considered

max D  L  W

X max  max  D  max L  W
 D  L  max W

However, since maxD and D are the same only two combinations need to be considered

max L  W
X max  D  max 
L  max W

The mean value of Xmax is then

 30  1 31
X  20  max  20  max   53
9  24 33
max

The second load combination gives the largest value of μXmax so the corresponding value of the variance
is

 X2 max
  D2   L2   max,
2
W  2  2.8  4.8  34.9
2 2 2

Thus,  X max  34.9  5.9 .

8.3 System reliability

8.3.1 Introduction

In the previous chapters, we have considered only one failure mode for a single structural
component (i.e., failure has been described by a single limit state function). While it may be sufficient to
solve some problems, in many practical applications this is not the case. Most buildings and bridges
consist of a system of interconnected components and their failure may be associated with
simultaneous failure of several of their components defined by different limit state functions. There are

Safety, Risk and Reliability, Heriot-Watt University 178


8. Reliability Analysis of Structural Systems
also cases when failure of single component may occur in different failure modes at different cross-
sections (e.g., a beam can usually fail in two ways – bending and shearing, which are described by
different limit state functions). Thus, to make clear differentiation between component reliability
analysis and system reliability analysis we define component failure as a failure event associated with a
single failure mode (i.e., described by a single limit state function). This means that such a component
does not necessarily correspond to a structural component of a structural system. Accordingly, system
failure represents the situation, in which the failure event involves several failure modes. For example, a
beam is a component of a building but if it fails in two failure modes – bending and shear, and we want
to account for both of them then system reliability analysis is required.

There are two extreme types of components that are commonly considered in reliability analysis of
structural systems: brittle and ductile. Figure 8.2 shows symbols which will be used to distinguish these
two types of components.

Brittle Ductile

Figure 8.2 Symbols used for brittle and ductile components.

A component is defined as brittle if it becomes completely ineffective after it fails. Figure 8.3a shows
a typical load-displacement curve of a brittle component. Examples of brittle components are
unreinforced concrete members in tension (or shear), welded joints failing due to fatigue crack growth,
or cables from composite materials. A component is defined as ductile if it maintains its load-carrying
capacity after its failure and, hence, still actively participates in redistribution of loads acting on the
structural system. Figure 8.3b shows a typical load-displacement curve of a ductile component. A ductile
component ‘fails’ when the yield point is reached but after that it is still capable to sustain load along
with development of large displacements. Examples of ductile components are members from mild steel
in tension, reinforced concrete beams in bending.

Load Rupture Load Yield point

Displacement Displacement
(a) (b)
Figure 8.3 Load-displacement curves for (a) brittle and (b) ductile components.

Safety, Risk and Reliability, Heriot-Watt University 179


8. Reliability Analysis of Structural Systems
In system reliability analysis it may be important to distinguish between brittle and ductile
components. It is also worth to note that real components do not necessarily demonstrate perfect
brittle or ductile behaviour (e.g., elements from high strength steels have limited ductility and fail at
relatively small displacements, at the same time to treat them as perfectly brittle would also be
incorrect).

8.3.2 Series systems

A series system is a system which fails when one of its components fails. A series system is also
sometimes referred to as a weakest link system because its failure corresponds to the failure of its
weakest component. Examples of a series system are chains and statically determinate trusses. If any of
the chain links or the truss members fails the chain or the truss will fail as well. For reliability analysis of
a series system the distinction between brittle and ductile components is unimportant since the system
fails when its weakest component fails (i.e., no possibility of the load redistribution to other
components). Figure 8.4 shows schematic description of series systems.

Figure 8.4 Schematic descriptions of series systems from brittle and ductile components.

Consider a series system consisting of n components. The strength of each component, Ri (i=1,2,…,n),
is a random variable with the CDF FRi(r). Denote the strength of the entire system as R. The CDF for R can
be derived by considering the probability of failure of the system. Suppose the system is loaded by a
deterministic load q. Failure of the system occurs when the strength R is less than the load q. In terms
of the probability of failure

Pf  PR  q   FR q  (8.8)

Note that the use of ≤ instead of < is acceptable because we assume that R is a continuous random
variable.

When the load q is applied to the system, the load in each element qi (i=1,2,…,n) depends on the
geometry and layout of the system. For example, in a simple chain the load in each link is also q, while in
a statically determinate truss loads in its members may be different. We assume that the strengths of

Safety, Risk and Reliability, Heriot-Watt University 180


8. Reliability Analysis of Structural Systems
the system components are uncorrelated random variables. The probability of failure in Eq. (8.8) can
then be found as follows

Pf  FR q   PR  q   1  PR  q 
 1  PR1  q1   R2  q 2   ...  Rn  q n 
 1  PR1  q1 PR2  q 2  PRn  q n  (8.9)
 1  1  PR1  q1 1  PR2  q 2 1  PRn  q n 

   
n n
 1   1  FRi qi   1   1  Pf i
i 1 i 1

where Pfi denotes the probability of failure of the i-th component. The replacement of the probability of
the interaction of the events appearing in the second line of the derivation by the product of the
probabilities of the events in the third line is possible because Ri’s are uncorrelated so we can assume
that the events Ri > qi (i=1,2,…,n) are statistically independent.

Worked Example 8.2

Consider the series system shown in Figure 8.5, which consists of a steel beam with 4-m span and a
steel cable supporting the beam at the right end. The system fails when either the beam or the cable
fails. The bending resistance of the beam, Rb, is a lognormal random variable with a mean of 15 kNm and
a COV of 0.10. The resistance of the cable, Rc, is a normal random variable with a mean of 10 kN and a
COV of 0.13. Calculate the probability of failure of the system. Assume that Rb and Rc are statistically
independent, the load P is deterministic, and the self-weight of the beam and the cable can be
neglected.

P=10 kN

2m 2m

Figure 8.5 Two-component series system from Worked Example 8.2.

Solution

The parameters of the lognormal distribution of the bending resistance of the beam are:

  COVR  0.10;   ln 15  0.5  0.10 2  2.7031


b

The maximum bending moment in the beam is

Safety, Risk and Reliability, Heriot-Watt University 181


8. Reliability Analysis of Structural Systems
PL 10  4
M max    10 kNm
4 4

The probability of the beam failure is

 ln 10  2.7031 
Pf1  PRb  10      4.00  3.17  10
5

 0.10 

The cable fails when its resistance is less than 0.5P=5 kN

 5  10 
Pf 2  PRc  5      3.85  5.91  10
5

 0.13  10 

The system fails when either the beam or the cable fails. Thus, the probability of failure of the system is
given by Eq. (8.9)

    
2
Pf  1   1  Pfi  1  1  3.17  10 5 1  5.91  10 5  9.08  10 5
i 1

As can be seen, the probability of failure of the system is larger than the probability of failure of either
component.

The corresponding safety index of the system is

   1 Pf    1 9.08  10 5   3.74

The safety indices of the beam and the cable are 4.00 and 3.85, respectively, i.e., larger than the safety
index of the system. The example illustrates that a series system is less reliable than any of its
components.

8.3.3 Parallel systems

A parallel system fails when all its components fail. For reliability analysis of a parallel system is very
important whether its components brittle or ductile.

Safety, Risk and Reliability, Heriot-Watt University 182


8. Reliability Analysis of Structural Systems
8.3.3.1 Parallel system with perfectly ductile components

Consider a parallel system consisting of n ductile components, which is schematically shown in Figure
8.6.
q

Figure 8.6 Parallel system consisting of ductile components.

Denote the strength of the i-th component as Ri (i=1,2,…,n). The system is in a state of failure when
all of its components fail (i.e., yield). Thus, the strength of the system, R, is the sum of the strengths of
its components

n
R   Ri (8.10)
i 1

If the strengths of the components are all uncorrelated random variables then the system strength is a
random variable with the following mean and variance

n n
 R    R ;  R2    R2
i i
(8.11)
i 1 i 1

Based on the central limit theorem, it is usually reasonable to assume that R is a normal random variable
even when Ri are non-normal random variables provided that n is sufficiently large.

Let’s determine the probability of failure of the system, when it is loaded by a deterministic load q.
The load is redistributed between the system components so denote the portion of the load in the i-th
component as qi. Failure of the i-th component occurs when Ri ≤ qi. The probability of failure of the
system can then be found as follows:

Pf  FR q   PR  q   PR1  q1   R2  q 2   ...  Rn  q n 


 PR1  q1 PR2  q 2  PRn  q n   FR1 q1 FR2 q 2  FRn q n  (8.12)
n n
  FRi qi    Pf i
i 1 i 1

The multiplication of the probabilities in the second line of Eq. (8.12) is possible because the strengths of
the components are uncorrelated so we assume that their failures are statistically independent events.

Safety, Risk and Reliability, Heriot-Watt University 183


8. Reliability Analysis of Structural Systems
Now consider a special case when the system components are identical. Thus, the random variables
Ri have the same distribution parameters, i.e., the same mean and standard deviation, which are
denoted as μRi and σRi. The mean and variance of the system strength is then

 R  n R ; i
 R2  n R2 i
(8.13)

The coefficient of variation of the system strength is

 n R 1 R COVR
2

COVR  R    i i i
(8.14)
R n R n R n i i

Thus, the COV of the strength of the parallel system of n identical components is smaller than the COV
of the strength of the component.

Worked Example 8.3

The parallel system consists of two perfectly ductile components, whose strengths are uncorrelated
random variables with the following distribution parameters:

 R   R  5 kN;
1 2
COVR1  COVR2  0.20

Find the mean, standard deviation and COV of the strength of the system.

Solution

The standard deviation of the strength of each component is

 R   R  0.20  5  1 kN
1 2

Since both components have the same distribution parameters we can use Eqs. (8.13) and (8.14) with

n=2

 R  n R  2  5  10 kN
i

 R2  n R2  2  1  2 →  R  2 kN
i

COVRi 0.20
COVR    0.14
n 2

8.3.3.2 Parallel system with brittle components

Consider a parallel system consisting of n brittle components, which is shown in Figure 8.7. In this
system, when one brittle components fails it loses completely its capacity to carry load. Thus, the load

Safety, Risk and Reliability, Heriot-Watt University 184


8. Reliability Analysis of Structural Systems
carried by this component must be redistributed to the remaining (n-1) components. If, after the load
has been redistributed, the system does not fail, the load can be increased until the next component
fails. Then the load is once again redistributed among the remaining (n-2) components. The process of
the component failure and load redistribution continues until the system fails.

Figure 8.7 Parallel system consisting of brittle components.

Denote the strength of the components as R1, R2,…, Rn. Assume that the strengths are arranged in the
following way: R1 < R2 < … < Rn. The strength of the system is then

R  max nR1 , (n  1) R2 , (n  2) R3 ,,2Rn1 , Rn  (8.15)

8.3.4 Hybrid (combined) systems

Often structures can be modelled as a combination of series and parallel systems. Such systems are
referred to as hybrid or combined systems. Figure 8.8 shows an example of a hybrid system consisting of
two components 1 and 2 in parallel and then component 3 in series.

1
3
2

Figure 8.8 Example of a hybrid (combined) system.

Many hybrid systems may be analysed using the techniques described in Sections 8.3.2 and 8.3.3.
This is illustrated by the following exercise.

Safety, Risk and Reliability, Heriot-Watt University 185


8. Reliability Analysis of Structural Systems
Worked Example 8.4

A steel frame is shown in Figure 8.9. The columns in the frame are fixed at the supports and the
beam-column joints are modelled as pinned connections. The applied loading includes a vertical force Q
and a horizontal force W. To simplify the problem it can be assumed that the compression capacity of
the columns is considerably larger than the load effects so that their failure due to bending is the critical
one. Moreover, the influence of the axial compressive forces in the columns on the bending capacity of
the columns is neglected. It is also assumed that the loads are deterministic while the column and the
beam bending resistances are independent random variables. The coefficient of variation of the bending
resistance of the columns is 0.125, while the COV of the bending resistance of the beam is 0.14.
Determine the required mean bending resistances of the columns and the beam if the target safety
index for the frame is 4.5 (i.e., βT=4.5).

Q=30 kN
W=20 kN

3m

3m 3m

Figure 8.9 Frame structure considered in Worked Example 8.4.

Solution

The frame will fail if either the moment-carrying capacities of both columns near the fixed ends are
exceeded or the moment-carrying capacity of the beam in the middle of the span is exceeded. Thus, the
frame may be modelled as the following hybrid system.

Column 1
Beam

Column 2

Let us denote by Pf the probability of failure of the frame, Pfc the probability of failure of the parallel
subsystem consisting of the two columns and Pfb the probability of failure of the beam. Note that Eq.
(8.12) cannot be used for the parallel subsystem since we do not know how to divide the load between

Safety, Risk and Reliability, Heriot-Watt University 186


8. Reliability Analysis of Structural Systems
the two columns. It would be incorrect to assume that the load W is divided equally between the
columns since the division depends on actual moment-carrying capacities of the columns, which are
random variables. For example, if the moment-carrying capacity of the left column is 21 kNm it will
resist only 7 kN (×3=21 kNm) out of W=20 kN, while at the same time the moment-carrying capacity of
the right column may be either less or greater than 39 kNm (in the former case the subsystem will fail
and in the latter – survive).

The probability of failure of the frame (i.e., the system) is then


Pf  1  1  Pfb  1  Pfc 
For the frame the target safety index βT=4.5. Hence, the target probability of failure

PfT    T    4.5  3.40  10 6

It is reasonable to assume that in a cost-efficient series system all components have the same
probability of failure. For the system under consideration this means Pfb=Pfc that leads to

Pf T  1  1  Pfb,T   3.40  10 6 . Solving this equation for Pfb,T gives Pfb,T=1.70×10-6 and
2

 b,T   1 1.70  10 6   4.65 . Similar, Pfc,T=1.70×10-6 and βc,T=4.65.

30  6
The limit state function for the beam is: G  Rb   Rb  45 (kNm).
4

G  R  45
The safety index for the beam is then  b   b
and it should be equal to the target
 G  R COVR
b b

 R  45 45
value, i.e., b
 4.65 so that  Rb   128.9 kNm.
 R  0.14
b
1  0.14  4.65

The capacity of the parallel subsystem consisting of the two columns is Rc,Σ=2Rc (see Eq. (8.10)), its
mean value μRc,Σ=2μRc (see Eq. (8.13)) and coefficient of variation COVRc,Σ=COVRc/√2 (see Eq. (8.14)).

The limit state function for the subsystem is: G  Rc,  20  3  2Rc  60 (kNm).

G 2 R  60
The safety index for the subsystem is then  c   c
and it should be equal to the
 G 2 R COVR c
2 c

 R  30 30
target value, i.e., c
 4.65 so that  Rc   50.9 kNm.
 R  0.125
c
2 1  0.125  4.65 2

Safety, Risk and Reliability, Heriot-Watt University 187


8. Reliability Analysis of Structural Systems
8.3.5 System reliability bounds

In the previous sections it was assumed that strengths of the system components were uncorrelated.
However, this assumption is not always valid. When correlation exists between the system components,
an exact analytical calculation of the system reliability (or the probability of failure) is usually difficult if
not impossible. However, simple bounds can established for series and parallel systems with positive
correlation between their components. In other words, these bounds are applicable when the
coefficient of correlation between the strengths of the i-th and j-th components, ρij, is greater than or
equal to zero.

8.3.5.1 Series system with positive correlation

For a series system with positive correlation, the probability of failure should be within the following
bounds

   
n
max Pf i  Pf  1   1  Pfi (8.16)
i
i 1

The lower bound is the probability of failure when all components are fully correlated (i.e., ρij = 1). In
this case, all the components will fail if one of them fails so the probability of failure equals the largest
probability of failure among the system components. The upper bound is the probability of failure when
all components are uncorrelated, which is given by Eq. (8.9).

8.3.5.2 Parallel system of ductile components with positive correlation

Similar to the case of a series system with positive correlation, we can establish bounds for the
probability of failure of a parallel system consisting of ductile components with positive correlation.
Once again we consider the two extreme cases – perfect correlation and no correlation. The bounds in
this case are as follows

 
n

P
i 1
fi  Pf  min Pfi (8.17)

The lower bound represents the case where the components are uncorrelated and the system fails only
when all the components fail, i.e., the probability of failure is defined by Eq. (8.12). The upper bound
represents the case in which all components are perfectly correlated. In this case, the safest component
controls the reliability of the system.

Safety, Risk and Reliability, Heriot-Watt University 188


8. Reliability Analysis of Structural Systems
Exercises
Exercise 8.1

A steel beam is designed. The beam is loaded by four load components: dead load, sustained live
load, transient live load and wind load. The load data are summarised in the table below. The bending
strength of the beam, R, is a random variable with a coefficient of variation of 0.11.
Load Nominal Arbitrary point in time 50-year extreme
value Mean COV Mean COV
Dead, D Dn=40 kNm 1.05Dn 0.10 1.05Dn 0.10
Sustained live, Q Qn=25 kNm 0.25Qn 0.40 0.85Qn 0.12
Transient live, P Pn=35 kNm 0.17Pn 0.60 0.75Pn 0.15
Wind, W Wn=30 kNm 0.10Wn 0.85 0.80Wn 0.20

Determine the required mean value of R if the target safety index is 3.5. Use Turkstra’s rule to calculate
the mean and standard deviation of the total load. Assume that the strength and the total load are
normally distributed.
Ans. 137.7 kNm

Exercise 8.2

A four-bar system shown in the figure below is loaded by a force Q. Denote the strength of the i-th
bar as Ri. The strengths of the bars 1, 2 and 3, R1, R2 and R3, are normally distributed with a mean of 10
kN and standard deviation of 2 kN. The strength of the bar 4 is log-normally distributed with a mean of
30 kN and a standard deviation of 4.5 kN. Assume that the bars are perfectly ductile and their strengths
are uncorrelated.

1 2 3

(a) What is the probability of failure of the system when Q=15 kN?
Ans. 1.02×10-5

Safety, Risk and Reliability, Heriot-Watt University 189


8. Reliability Analysis of Structural Systems
(b) What is the probability of failure of the system when Q=25 kN?
Ans. 0.1925

Exercise 8.3

A series system consists of two components. The safety index for each component is 3.5. The
strengths of the components are normally distributed. Determine the upper and lower bounds on the
probability of failure of the system.
Ans. 2.33×10-4 and 4.66×10-4

Exercise 8.4

A parallel system consists of two perfectly ductile components. The safety index for one component
is 3.00 and for the other component 3.25. The strengths of the components are normally distributed.
Determine the upper and lower bounds on the probability of failure of the system.
Ans. 7.79×10-7 and 5.77×10-4

Revision Questions
8.1 Explain the problem of load combination. How is the problem solved in the Turkstra’s rule?

8.2 Give definitions of brittle and ductile components.

8.3 What is a series system? How is its reliability calculated?

8.4 What is a parallel system?

8.5 How is the reliability of a parallel system calculated when its components are brittle? Ductile?

8.6 What is a hybrid (or combine) system?

8.7 What are the system reliability bounds? When are they applied?

SUMMARY

 The problem of combining various loads acting on a structure over its lifetime has been explained.
The Turkstra’s rule for estimating the total load has been described and its use has been illustrated
by an example.

 Definitions of brittle and ductile components have been given.

Safety, Risk and Reliability, Heriot-Watt University 190


8. Reliability Analysis of Structural Systems
 Series and parallel systems have been defined. Formulas for calculating their reliabilities, when
failures of their components are independent events, have been derived and their use has been
illustrated by examples.

 The use of system reliability bounds for estimating the reliability of systems with correlated failures
of the components has been explained.

References
Turkstra, C.J. and Madsen, H. (1980). Load combinations for codified structural design. Journal of the
Structural Division, ASCE, 106(ST12), 2527-2543.

Safety, Risk and Reliability, Heriot-Watt University 191


8. Reliability Analysis of Structural Systems
9 RELIABILITY-BASED CODE CALIBRATION

9.1 Introduction
One of the main aims of the design process is to ensure safety of structures, which is normally
achieved through the use of design codes and specified in them partial safety factors. This chapter will
present how the reliability methods studied previously can be used to determine values of the partial
safety factors; this process is usually referred to as the reliability-based calibration of design codes. It is
necessary to note that that the role of design codes is wider than just addressing safety requirements.
Although the design codes may not include explicit requirements concerning structural performance
and costs, the latter obviously depend on the codes as well. Thus, the role of the design codes might be
better seen in terms of national economic competitiveness and decision-making.

9.2 Levels of reliability-based design


Depending on the way the safety of a structure is provided, the design methods can be classified as
follows:

- Level I methods, which use deterministic design equations. The safety is provided through the use of
safety factors (the ratio of design resistance to design load) or partial safety factors (see also Section
4.2).

- Level II methods, in which the safety of the structure is characterised by the safety index. The latter
is estimated by the FOSM methods or FORM and compared with its target value.

- Level III methods, which require full probabilistic analysis (e.g., Monte Carlo simulation) to calculate
the probability of failure of the structure under different loads. The calculated probability of failure
is then compared with its target values.

- Level IV methods, which require solution of an optimisation problem, taking into account different
sources of uncertainty. The optimal design solution maximises the expected utility, which represents
the difference between the benefits and costs associated with a particular design solution.

The current design codes belong to the Level I methods, in which partial safety factors are derived
using the Level II methods. Such codes are also referred to as semi-probabilistic codes. Level III and IV
methods are normally used in advanced research and for validation of the Level II methods.

Connections between the safety approaches, design and code calibration are shown in Figure 9.1.

Safety, Risk and Reliability, Heriot-Watt University 192


9. Reliability-Based Code Calibration
9.3 Basic elements of code development
The following basic elements of the code development will be considered in this section:

- Establishing scope of the code

- Selection of calibration points

- Establishing target safety levels

- Selection of the code format

Deterministic methods Probabilistic methods

(experience + intuition) (Level II and III)

Calibration Calibration

Deterministic codes Semiprobabilistic Probabilistic


codes Model Code

Direct probabilistic
Partial factor design
design

Figure 9.1 Current approaches to safety provisions in design and code calibration.

9.3.1 Establishing scope of the code

A code is usually developed for a certain group or class of structures. It is necessary to define the
group/class by defining parameters covered by the code. The parameters usually include the type of
material (e.g., steel, concrete, masonry, timber) and the type of usage/function (e.g., buildings -
residential, office, hospital, hotel, industrial; bridges – highway, railway, pedestrian), structural type
(frames, beams, columns, connections, foundations), etc.

The scope of the code should be clearly defined in terms of such parameters. The scope can be very
wide (e.g., all types of buildings or bridges) as well as very narrow (e.g., anchor bolts used in concrete
walls of nuclear power plants). The code may cover various limit states – ultimate limit states (e.g.,

Safety, Risk and Reliability, Heriot-Watt University 193


9. Reliability-Based Code Calibration
flexure, shear, compression, tension, buckling), serviceability limit states (e.g., cracking, deflections,
vibrations), fatigue limit states, or loads acting on structures (e.g., live, wind, snow, earthquake).

The scope of a code is always compromise between simplicity and closeness to the safety objectives.
On one hand, it is desirable to have a small number of codes with each code covering a wide range of
structures so that it is easier for a designer to follow. On the other hand, a wide code with relatively
simple requirements cannot provide uniform safety close to the target levels for all structures covered
by it. If the scope is narrowed to structures with similar parameters, it is easier to ensure that the safety
of the structures will be close to target levels.

9.3.2 Selection of calibration points

Generally, it is very difficult (or even impossible) for a design code to satisfy exactly its safety
objectives for the whole possible range of its parameters/basic variables (e.g., beam spans, cross-
sectional dimensions, material strengths, range of applied loads and loading types, etc). Thus, it is
necessary to select values of the basic variables, at which the safety objectives will be checked. At the
same time, some values of the basic variables will occur in practice more frequently than others. For
example, the ratio of dead load to live load is typically between 0.5 and 2.0, while values like 0.25 or 3.0
may occur as well but with lower frequency.

For code calibration, the possible range of values of a basic variable should be defined and then
divided into a reasonably small number of intervals, with each interval represented by a single value of
the basic variable which called a calibration point. As noted previously, different frequencies of
occurrence may correspond to each calibration point. In order to take this into account, the so-called
demand function assigning weights to the calibration points depending on their frequency of occurrence
may be introduced. Obviously, the higher the frequency of occurrence the higher the value of the
demand function, i.e., the more important the calibration point for the code calibration.

The frequency of occurrence of each calibration point can be estimated by statistical analysis of data
from current design practice. In the development of a new code, it is also important to consider future
trends in construction practice (e.g., an increase in the strength of construction materials), which may
change the frequency of occurrence of different calibration points.

9.3.3 Establishing target safety levels

The selection of an appropriate value of the target safety index βT (or the target probability of failure
PfT) is a difficult task. As discussed at the end of Section 5.3.2, a probability of failure obtained using

Safety, Risk and Reliability, Heriot-Watt University 194


9. Reliability-Based Code Calibration
reliability analysis represents the notional probability of failure, which may be not close to the actual
(real) probability of structural failure. Moreover, a relationship between nominal probability of failure
and the corresponding real one cannot usually be established. Thus, the data on acceptable levels of risk
considered in Lecture 1 cannot be directly used to set values of βT (or PfT).

A possible approach used in practice is that new design codes should be calibrated against existing
practice, i.e., on average provide more or less the same level of structural safety as existing design codes
(of course, if the latter are perceived adequate). Of course, there will be variations of the safety levels
across different design situations, both in existing codes and in new ones. Generally, calibration of a new
design code aims to achieve more uniform safety levels, closer to βT, across the considered range of
design situations compared to the corresponding existing code. Practical implementation of this
approach will be explained in more detail further in the chapter.

For serviceability limit states, which are not associated with risk to human life (or injury), βT (or PfT)
can be determined using an approach based on minimisation of the total cost of a structure. If to neglect
maintenance and demolition costs, the total cost of a structure, CT, against a single limit state can be
expressed as

CT  C I  C F Pf (9.1)

where CI is the initial cost of the structure and CF is the cost of failure. The initial cost can be expressed
as a function of the probability of failure Pf expressed via the safety index β (β=-Φ-1(Pf)) (Nowak and
Collins 2000)

C I  a1  b  (9.2)

where a and b are constants. According to Eq. (9.2), the initial cost CI increases as β increases that is
reasonable, since it is expected that the initial cost of materials and construction should be higher in
order to achieve a safer structure. Assuming that the cost of failure CF is independent of the safety index
β, the total cost can be expressed as

CT  a1  b   C F    (9.3)

where Φ(.) is the cumulative distribution function of the standard normal distribution. Thus, CT is the
sum of the two functions of β – the first of them increases with an increase in β, while the second
decreases. Figure 9.2 illustrates the relationship between CT and β. The optimum design corresponds to
the minimum value of CT. The corresponding value of β can be set as the target value, βT.

Safety, Risk and Reliability, Heriot-Watt University 195


9. Reliability-Based Code Calibration
CT

CT,min

βT β
Figure 9.2 Total costs CT versus the safety index β.

9.3.4 Code formats

One of the main elements of the code development is the selection of an appropriate safety-
checking format. Safety-checking formats used in existing design codes are developments of
deterministic formulations rather than being developed from probabilistic considerations. As noted
previously, they all represent Level I methods and do not require any probability calculations from the
code user. The only difference of the current generation of design codes compared to the old ones is
that the partial safety factors (or the load and resistance factors) have been derived from probabilistic
data rather than being merely selected by a code committee. A number of safety-checking formats have
been proposed. Two of them – the one used in Eurocodes and the other one employed in US codes, are
described in more detail further in this section.

A safety-checking format usually referred to as the partial safety factors format used in Eurocodes
has been developed from earlier code formats (CEB 1976). It has the following general form (EN 1990):

Rd  R X k  m , ad   Rd  Ed   Sd E  fiFki , ad  (9.4)

where Rd – design value of the resistance,

Ed – design value of effect of actions,

Xk – characteristic values of a material property,

γm – partial factor for a material property,

η – conversion factor,

ad – design value of a geometric property,

γRd – partial factor associated with the uncertainty of the resistance model,
Safety, Risk and Reliability, Heriot-Watt University 196
9. Reliability-Based Code Calibration
γSd – partial factor associated with the uncertainty of the action model,

Fki – characteristic value of action

γfi – partial factor for action

ψ – action combination factor.

Another popular safety-checking format is the so-called load and resistance factor design (LRFD)
format proposed by Ravindra and Galambos (1978) for the use in US codes. In this format the design
formula is expressed as

Rn    i Qni (9.5)

where Qni is the nominal value of the i-th load component, γi the load factor for the i-th load
component, Rn the nominal value of resistance, and ϕ the resistance factor.

9.4 Derivation of partial factors by FOSM/FORM


In this section simple formulae for calculation of the partial safety factors based on the FOSM
method/FORM will be derived. First, we consider the case when the basic random variables are normal.
Denote a basic random variable on the resistance side representing a material property as X. On one
hand, for the target safety index βT the design value of X can then be found using Eq. (5.35)

xd   X   X T   X 1  COVX T  (9.6)

where μX, σX and COVX are the mean, standard deviation and coefficient of variation of X, and α is the
direction cosine for X of the unit outward normal to the limit state surface at the design point also
known as the sensitivity factor. On the other hand, the design value of a material property is

xk
xd  (9.7)
m

where xk is the characteristic value of X and γm the partial factor for the material property.

The characteristic value of X is given by

xk   X  k X  X   X 1  k X COVX  (9.8)

where kX is the appropriate coefficient corresponding to the characteristic fractile of the standard
normal distribution (e.g., if the characteristic fractile is the 5% fractile then kX=-Φ-1(0.05)=1.645). Thus,
from Eqs. (9.6)-(9.8)

Safety, Risk and Reliability, Heriot-Watt University 197


9. Reliability-Based Code Calibration
xk 1  k X COVX
m   (9.9)
xd 1  COVX  T

If X represents action (load), then the partial factor for action, γf, is

xd 1  COVX  T
f   (9.10)
xk 1  k X COVX

Note that the value of the sensitivity factor α is positive for resistance variables (material properties)
and negative for actions (or action effects).

EN 1990 permits to use the following values of the sensitivity factors: for resistance parameters
αR=0.8 and for action effects (loads) αE=-0.7, provided that

0.16   E  R  7.6 (9.11)

where σE and σR are the standard deviations of the action effect and resistance, respectively.

The target values of the safety index recommended by EN 1990 for residential and office buildings,
public buildings where consequences of failure are medium are shown in Table 9.1.

Table 9.1 Target values of the safety (reliability) index recommended by EN 1990.

Limit state Target safety index


1 year 50 years
Ultimate 4.7 3.8
Fatigue 1.5 to 3.8
Serviceability (irreversible) 2.9 1.6

9.5 Code calibration procedure


There is general agreement on the procedure of code calibration, which includes the following steps:

1) Define scope of the code (see Section 9.3.1).

2) Set the target safety levels (values of βT)

2.1) Select calibration points and evaluate their relative frequencies pi (Σpi=1) (see Section 9.3.2).

2.2) For each appropriate combination of calibration points (further referred to as a design case)
design structural elements using the existing design code.

2.3) Determine statistical properties (distributions, means, coefficients of variations) of the basic
random variables.

Safety, Risk and Reliability, Heriot-Watt University 198


9. Reliability-Based Code Calibration
2.4) Calculate the safety indices βi for the selected design cases. For this purpose the FOSM
methods of FORM may be used.

2.5) Determine the target safety index βT. If the level of safety of the new code is based entirely on
that of the existing code, then βT can be found as weighted average of βi’s, i.e., Φ(-βT)=ΣpiΦ(-
βi). In general, some allowance can be made for consequences of failure, i.e., by assigning
higher values of βT for high consequences failures. For example, in Eurocodes all structures are
divided into three groups, depending on consequences of their failures, and for each group
different values of βT are used.

3) Determine new code parameters (e.g., partial safety factors).

3.1) Choose the code format (see Section 9.3.4).

3.2) Make assessment of the new code parameters, e.g., partial factors γ(j) (as will be clear from
the following, the determination of the new code parameters is an iteration process and the
superscript j denotes the iteration number). For this purpose, for a considered design case,
set initial values of the partial safety factors and design the element in accordance to the
new code. Calculate then the safety index using exactly the same statistical properties of the
basic random variable as in 2). If the calculated β is less than βT, change values of the partial
factors and repeat the reliability analysis until β=βT (or slightly higher). Obviously, for
different design cases different values of the partial factors may be obtained. For normal
design this is unacceptable, since it is expected that the partial factors will be constant, at
least over large groups of design cases. To achieve this, the obtained partial factors should be
modified that will lead to some deviations from βT at a number of design cases. This involves
a certain amount of subjective judgement.

3.3) For each design case design structural elements using the new code format and values of the
code parameters from 3.2) (i.e., γ(j)).

3.4) Calculate the safety indices βn,i (γ(j)) for the selected design cases (the subscript n is used to
distinguish that the safety indices are calculated for the new design code and obviously the
safety indices will depend on γ(j)).

 p   
  n ,i  ( j )
2
3.5) Check if i T   . If “yes” – stop the iteration process, if “no” – go to

3.2). The aim of this check is to ensure that in the new code the safety levels for to different
design cases do not deviate significantly from the target safety levels. The value of Δ controls
the acceptable level of the deviation.

The flow chart of the code calibration process is shown in Figure 9.3.
Safety, Risk and Reliability, Heriot-Watt University 199
9. Reliability-Based Code Calibration
1 Define scope of code

Choose design cases and evaluate their


Collect data for input variables – strengths, relative frequencies
loads, dimensions

For each design case obtain dimensions

Determine statistical parameters for input


2 according to existing code

variables (e.g., distributions, means, For each design case calculate reliability
variances) indices i

Determine the target reliability index T

Choose new code format and make first


Modify code parameter values assessment of the code parameters

no For each design case obtain dimensions


according to the new code

Is s  
3
For each design case calculate reliability
indices n,i

yes
Calculate
  
s   pi  T   n,i  ( j )
2

STOP

Figure 9.3 Flow chart of the code calibration process.

9.6 Example of code calibration (adapted from Melchers (1999))


The calibration process will be illustrated by the following example, adapted from Melchers (1999).
The ultimate limit state associated with bending is considered for a steel beam. This defines the scope
for calibration. For simplicity, only dead load and live load are considered. Assume that the existing code
safety-checking format for this design situation is

Rn   F Dn  Ln  (9.12)

where γF=1.7 is the load factor, and Rn, Dn and Ln are the nominal values of resistance, dead load and live
load, respectively. In the following Ln will be expressed via Dn so it will not be necessary to deal with
beam dimensions, area supported, etc. Eq. (9.12) specifies the required resistance given the nominal
values of load, Dn and Ln. In general, Rn depends on material and geometric properties as well as
resistance modelling rules defined by the code committee.

The new code format will be the LRFD format so that

Safety, Risk and Reliability, Heriot-Watt University 200


9. Reliability-Based Code Calibration
Rn   D Dn   L Ln (9.13)

It is also assumed, for simplicity, that the specification of Dn, Ln and Rn remains the same as in the
existing code. Thus, it is required to determine φ, γD and γL such that βT is approximately constant and
consistent with the existing code.

The limit state function can be expressed as

GX  R  D  L (9.14)

Statistical properties of the basic random variables are defined as follows and, for simplicity, it is
assumed that R, D, and L are normal random variables.

R
 1.18 COVR  0.13
Rn

D
 1.05 COVD  0.10
Dn

L
 1.00 COVL  0.25
Ln

Since the limit state function is a linear function of normal random variables, the reliability analysis
can be carried out using the FOSM method. The safety index is

G

G

where  G   R   D   L  1.18Rn  1.05Dn  1.0Ln

From Eq. (9.12)

 L 
Rn   F 1  n  Dn
 Dn 

Thus

 Ln  L   L  L 
 G  1.18 F 1   Dn  1.05Dn  n Dn  1.18 F 1  n   1.05  n  Dn
 Dn  Dn   Dn  Dn 

The variance of the limit state function is

Safety, Risk and Reliability, Heriot-Watt University 201


9. Reliability-Based Code Calibration
 G2   R COVR 2   D COVD 2   L COVL 2
2
 
2
 L  L 
 1.18 F 1  n  Dn  0.13  1.05Dn  0.10   n Dn  0.25 
2

  Dn    Dn 
  Ln  
2
 Ln   2
2

 1.18 F 1    0.13  1.05  0.10  


2
 0.25   Dn
  D n    D n  

As can be seen, in the formula for β Dn reduces so that β becomes just a function of Ln/Dn. Therefore, the
calibration points are values of Ln/Dn, for example, 0.5, 1.0,…, 4.0. Figure 9.4 illustrates the relationship
between β and Ln/Dn, e.g., when Ln/Dn=1.0 β =3.3377.

For purposes of illustration set the target safety index as 3.0, i.e., βT =3.0, although as can be seen
from Figure 9.4 its value should be a bit higher, around 3.2.

Initial values of the load and resistance factors for the new code should now be selected. A possible
approach is to find the design point based on the existing code format and from that result to obtain
initial estimates of these factors. The target safety index for the new code is βT =3.0. If we start with the
calibration point Ln/Dn=1.0, we need to adjust a value of γF since for γF=1.70 used in the existing code the
safety index corresponding to this calibration point is 3.3377. By trial and error it can be found that
γF=1.57 leads to β =3.0 when the nominal resistance is calculated using Eq. (9.12).

3.5

3.4
Safety index

3.3

3.2

3.1

2.9
0 1 2 3 4 5 6

Ln/Dn

Figure 9.4 Relationship between the safety index and Ln/Dn ratio for the existing code.

To calibrate the new code the sensitivity factors, α’s, need to be known. In order to find the design
point with the corresponding sensitivity factors we solve the problem in the reduced space of the
standard normal variables:

Safety, Risk and Reliability, Heriot-Watt University 202


9. Reliability-Based Code Calibration
R  R R  1.18 F 1  Ln Dn Dn
yR  
R 1.18 F 1  Ln Dn Dn COVR

R
Substituting Ln/Dn=1.0 and γF=1.57 we obtain:  0.482 y R  3.705
Dn

D  D D  1.05Dn D
For D and L: yD   →  0.105 y D  1.05
D 1.05Dn COVD Dn

L  L L  1.0 Ln L
yL   →  0.25 y L  1.0
L Ln COVL Dn

The limit state function in the reduced space is

g y   0.482 y R  3.705  0.105 y D  1.05  0.25 y L  1.0Dn

To find the sensitivity factors

g g g
 0.482 Dn ;  0.105Dn ;  0.25Dn
y R y D y L

2 12
  g  
l       0.553Dn
 i  yi  

0.482  0.105  0.25


so that  R   0.872;  D   0.190;  L   0.452
0.553 0.553 0.553

In the LRFD format Rd  Rn , thus the resistance factor can be found as

Rd 1   R  T COVR  R
   1  0.872  3.0  0.13  1.18  0.779
Rn Rn

The load factors are

Dd 1   D  T COVD  D
D    1  0.190  3.0  0.10  1.05  1.11
Dn Dn

Ld 1   L  T COVL  L
L    1  0.452  3.0  0.25  1.0  1.34
Ln Ln

Safety, Risk and Reliability, Heriot-Watt University 203


9. Reliability-Based Code Calibration
Thus, the LRFD format for Ln/Dn=1.0 and βT =3.0 is 0.78Rn  1.11Dn  1.34Ln . The process may be

repeated for other calibration points (i.e., other values of Ln/Dn).

For the purpose of illustration, let now decide that φ=0.80 and γD=1.20 so that only γL needs to be
found. Let try γL=1.40. Then

Rn 
1
1.2Dn  1.4Ln 
0.8

The mean value of the limit state function is then

1.18  L  L 1.18  L  L 
G  1.2  1.4 n  Dn  1.05Dn  n Dn   1.2  1.4 n   1.05  n  Dn
0.8  Dn  Dn  0.8  Dn  Dn 

while the variance is given by

2
1.18  
2
L  L 
 
2
G
1.2  1.4 n  Dn  0.13  1.05Dn  0.102   n Dn  0.25 
 0.8  Dn    Dn 
1.18  L  
2
L 
2

  1.2  1.4 n   0.13  1.05  0.102   n  0.25  2
 Dn
 0.8  Dn    Dn  

Values of the safety index can now be calculated for various values of Ln/Dn. Results are presented in
Figure 9.5 and Table 9.2 (together with an illustrative set of relative frequencies and the calculation of
s). These calculations can be repeated for other trial values of γL (results for γL=1.35 and 1.45 are also
shown in Figure 9.5). Obviously, the procedure can also be repeated for other values of φ and γD if this is
desired.

3.25
gamma_L=1.40
3.2 gamma_L=1.35
3.15 gamma_L=1.45
Safety index

3.1

3.05

2.95

2.9

2.85
0 1 2 3 4 5 6
Ln/Dn

Figure 9.5 Relationship between the safety index and Ln/Dn ratio for the new code (φ=0.80 and γD=1.20).

Safety, Risk and Reliability, Heriot-Watt University 204


9. Reliability-Based Code Calibration
Table 9.2 Selected results of code calibration.
Ln/Dn βi pi s   T   i  pi
2

0.5 3.1373 0.15 7.16×10-4

1.0 3.1453 0.15 3.418×10-3

1.5 3.1191 0.20 3.539×10-3

2.0 3.0925 0.20 2.220×10-3

2.5 3.0701 0.10 6.52×10-4

3.0 3.0518 0.10 3.66×10-4

3.5 3.0369 0.05 9.67×10-5

4.0 3.0245 0.05 4.61×10-5

βT = 3.00 Σ=1 Σ = 0.01105

Revision Questions
9.1 How can the design methods be classified depending on their approach to safety provision?

9.2 What are the basic elements of the code development?

9.3 What is a calibration point?

9.4 How can the target safety levels be established for code calibration?

9.5 What safety-checking formats do you know?

9.6 What are the steps of the code calibration procedure?

SUMMARY

 Different levels of the reliability-based design have been described.

 Basic elements of the code development have been considered.

 Derivation of the partial safety factors using the FOSM method/FORM has been explained.

 The code calibration procedure has been described and illustrated by an example.

Safety, Risk and Reliability, Heriot-Watt University 205


9. Reliability-Based Code Calibration
References
CEB (1976). Common Unified Rules for Different Types of Construction and Materials. Bulletin
d’Information No. 116-E, Comite Europeen du Beton, Paris.
EN 1990. Eurocode – Basis of structural design.
Melchers, R.E. (1999). Structural Reliability: Analysis and Prediction. 2nd Ed., John Wiley & Sons.

Nowak, A.S. and Collins, K.R. (2000). Reliability of Structures. McGraw-Hill, Boston.
Ravindra, M.K. and Galambos, T.V. (1978). Load and resistance factor design for steel. Journal of
Structural Division, ASCE, 104(ST9), 1337-1353.

Safety, Risk and Reliability, Heriot-Watt University 206


9. Reliability-Based Code Calibration
10 RELIABILITY-BASED ASSESSMENT OF EXISTING
STRUCTURES

10.1 Introduction
Many structures deteriorate with time and this needs to be taken into account both at the design
stage and when they are reassessed in service. In order to ensure safety of a structure, it is necessary to
have knowledge of its strength at all times during its operational life. If there is significant deterioration
in strength or serviceability then part of the structure may need to be repaired or replaced, or the
loading on the structure restricted. The principal modes of deterioration are:

 Fatigue – a crack starts and grows progressively under cyclic loading. This is a very common cause
of structural failure and is considered further below.

 Corrosion – this is common in steel structures where rusting occurs at the surface, and is discussed
below. Corrosion is also a problem in reinforced concrete as a result of chloride ions or carbonation
penetrating down to the steel reinforcement. This aspect is not covered in this module.

 Cracking and fracture – this can occur in a wide variety of materials and may be due to material
deterioration in cold or hostile environments, structural overloading or as a result of fatigue or
corrosion. Cracks may grow from defects and thus lead to fracture. This is also considered further
below.

 Creep – this is the gradual stretching of materials under a nominally steady load over a period of
time. Creep is more rapid at higher temperatures and the creep strain rate varies with time. It
usually becomes a problem at around at about 30% of the melting temperature. It can be a
problem with high temperature systems such as gas turbines where the blades may stretch.

 Material decay – this is commonly associated with timber structures which are liable to wet or dry
rot. It can also occur in plastics, rubber and other materials as a result of ‘ageing’ that may be
associated with solar radiation and/or contact with certain fluids, e.g., osmosis in glass fibre
reinforced plastic boats. Composite structures may delaminate as a result of bonding failure due to
deterioration of adhesives or ingress of water and other fluids, e.g., failure of timber based boards
such as plywood.

 Spalling – this happens to masonry and concrete when the outer layers flake off. In concrete this
can expose steel reinforcing, which then suffers corrosion; this can also be caused by corrosion.

Safety, Risk and Reliability, Heriot-Watt University 207


10. Reliability-Based Assessment of Existing Structures
 Abrasion – this happens to the surface of structures due to a rubbing action or the impingement of
small particles in a fast moving fluid. It often occurs at bends in pipes.

This list is not exhaustive and there are other modes of structural deterioration; some particular to
given materials and conditions.

10.2 Dealing with structural deterioration


There are a number of ways of dealing with structural deterioration. In many cases a combination of
techniques is used. The principal techniques are listed below.

 ‘Over-designed’ - at the design stage a structure may be over-designed to allow for the inevitable
deterioration that will occur during its service life. This approach is common for steel structures
such as pipelines and ships where pipe or plating thickness is increased by a corrosion margin. It
also sometimes occurs for masonry, concrete and timber structures where either explicit or implicit
margins are added to allow for loss of material or strength during the structure’s lifetime.

 Protective coatings – these are usually thin surface layers designed to protect the material of the
structure from contact with an ‘aggressive’ environment. The simplest example is painting the
surface of timber to reduce water absorption. Galvanising steel, anodising aluminium and providing
cathodic protection of marine structures are all examples of protective coating or shielding.

 Inspection, repair and maintenance – inspection may be on a regular basis, or when measurable
deterioration is predicted to have occurred. There are a variety of inspection techniques ranging
from simple visual inspection with the naked eye to complex, remotely-operated, X-ray systems.
The choice of inspection system and the time interval between inspections are often related as
discussed latter. When inspection reveals deterioration then a decision has to be made concerning
maintenance or repair. When and how to repair deterioration will depend on several factors.
Sometimes the right decision is not to maintain or repair.

Which approach, or combination of approaches, to use depends on the type of structure, the
material from which it is made, the consequences of failure and the cost and economics involved. In
some cases, such as merchant ships, the classification society will allow a trade-off between corrosion
margins, which have an associated weight penalty, and protective coatings that are initially more
expensive. It will then be a matter of looking at the life cycle economics to decide about the best option
for any particular set of circumstances.

Safety, Risk and Reliability, Heriot-Watt University 208


10. Reliability-Based Assessment of Existing Structures
10.3 Corrosion modelling

10.3.1 Types of corrosion

Corrosion is an electro-chemical reaction that occurs between the surface of the material and the
fluid with which it has contact. For steel it is essentially a process of oxidation. There are several types of
corrosion.

 Uniform corrosion – is the usually expected form, leading to a loss of material all over the surface
as illustrated in Figure 10.1. In reality it is rarely uniform but does spread over the whole surface.
Uniform corrosion is commonly seen on flat surfaces, such as the plating on a ship, or plate girders
on a bridge and also on 1-D elements, such as scaffold poles and bars. This type of corrosion
reduces overall strength and failure may occur at the weakest cross section.

Figure 10.1 Uniform corrosion.

 Pitting corrosion – this a localised attack on an otherwise resistant surface. The pits may be deep,
shallow or undercut as shown schematically in Figure 10.2. Pitting corrosion often occurs when the
protective layer on the surface breaks down at a number of points. This sort of corrosion often
occurs in pipes and can lead to local failures and leaking when corrosion goes right through the pipe
wall. Thus pitting corrosion can lead to earlier failures than uniform corrosion.

Figure 10.2 Schematic examples of pitting corrosion – note that in reality the surface of the pit can be
very irregular.

The rates for these types of corrosion can be estimated by various theoretically-based formulae that
take into account the chemistry at the metal surface and the temperature. Unfortunately, there is
considerable uncertainty (and often bias associated with these models) and so more reliance is placed
on inspection results and empirical modelling.

Safety, Risk and Reliability, Heriot-Watt University 209


10. Reliability-Based Assessment of Existing Structures
There are also other types of corrosion, for example:

 Galvanic and crevice corrosion are associated with contact between dissimilar metals.

 Hydrogen damage is often associated with welding.

 Intergranular corrosion and dealloying are related to the metallurgy of the alloy.

 Environmentally induced cracking can be linked to material stress, fatigue loading and welding.
These types of cracking are illustrated in Figure 10.3.

Hydrogen-induced cracking Stress-corrosion Corrosion fatigue

Figure 10.3 Various types of environmentally induced cracking.

10.3.2 Corrosion and minimum remaining strength

Widespread corrosion, nominally uniform can be assessed by a series of point measurements. This is
illustrated below for a tie rod Figure 10.4a. Measurements of the diameter of the rod can be made at
several sections. At each section a number of measurements are needed because the diameter will not
be reduced uniformly (as illustrated in Figure 10.4b).

a) b)

c)
Figure 10.4 Illustration of the measurement of a tie rod to determine diameter and thus estimate
remaining strength.

The mean diameter at each section can then be used to estimate the remaining cross sectional area
at that section. For a long tie rod, measurements can only economically be made at a few sections. Thus,
effectively a sample is made of all the possible sections that could be measured. Inspection will reveal
over what length increment of the tie rod the section can be considered sensibly constant. The whole tie

Safety, Risk and Reliability, Heriot-Watt University 210


10. Reliability-Based Assessment of Existing Structures
rod can then be viewed as series of short sections, each of the same cross section, as illustrated in Figure
10.4c. The problem then becomes one of estimating the smallest of these cross section areas, as this will
be the weakest cross section. This is a problem for extreme value theory as illustrated in the example
below. Note: in this example no allowance is made for variation in strength along the rod. There is some
evidence that this may occur but that is not considered in this example.

Worked Example 10.1

A series of measurements of the remaining cross-sectional area of a tie-bar are taken at intervals
along its length after it has been in service 20 years. The original cross section area was 100 mm 2 and
proof testing has shown that the minimum cross section must be more than 40 mm 2 or the tie rod
would have failed under the proof load. The results are plotted as a histogram and the following PDF is
found to fit the data quite well

7  x  40 
6

f X ( x)    40 ≤ x ≤ 100, as shown in Figure 10.5.


60  60 

(a) If the total length of the tie-bar is equivalent to 90 sample measurement lengths, find the CDF and
PDF for the cross-sectional area for the weakest section.

(b) What is the probability that the cross-sectional area of the weakest section is less than 60 mm 2?

fx(x)

0 40 100
2
x = cross sectional area (mm )

Figure10.5 Probability density function that fits the measurements of the cross-sectional area of a tie
bar after 20 years of service.

Solution

(a) Using the minimum extreme value theory the CDF is (see Eq. (3.68))

Safety, Risk and Reliability, Heriot-Watt University 211


10. Reliability-Based Assessment of Existing Structures
Fz1 ( z )  1  [1  FX ( z )]n

7  x  40   z  40 
z 6 7

where FX ( z )     dx   
40
60  60   60 

90
  z  40  7 
so that Fz1 ( z )  1  1    
  60  

The PDF is then (see Eq. (7.29))

89
  z  40  7  7  z  40  6
f z1 ( z )  n1  FX ( z )
n 1
f X ( z )  901      
  60   60  60 
89
  z  40  7   z  40  6
 10.51      
  60    60 

90
  60  40  7 
(b) Fz1 (60)  1  1      0.0403  4%
  60  

It is of interest to compare this probability with that for an arbitrarily selected section. What is the
probability that any section picked at random has a cross-sectional area less than 60 mm2?

 60  40 
7

FX (60)     0.000457  0.05%


 60 

Of course, this approach assumes that sample measurements properly represent the corrosion
pattern over the whole tie rod. If corrosion is concentrated in one region then this region must be
examined carefully. The most heavily corroded areas must be investigated in order that residual
strength is not overestimated.

Another, equivalent approach to this problem is to look at the amount material lost in corrosion. In
this approach we are looking for the maximum amount that may be lost which will give the smallest
remaining thickness and cross section, and hence the lowest strength.

Over a 2-D surface, such as a plate or a shell, the thickness must be measured over a grid of points as
illustrated below in Figure10.6. Here the question is of what elemental area of plate around each
measurement point can the plate thickness be considered sensibly constant? As will be seen in the
example below the result does not depend too much on the size of the elemental area selected.

Safety, Risk and Reliability, Heriot-Watt University 212


10. Reliability-Based Assessment of Existing Structures
x

y
t xi yk = thickness
at xi, yk

Figure 10.6 Grid of point measurements of plate thickness.

These measurements now characterise the plate. How they are used depends on what type of plate
loading is being investigated. In the example below we consider the membrane loading of a plate and
here the interest is in the weakest cross section. Thus each of the cross sections on which there are
measurements points are considered and the average thickness for each of these cross sections is
found. The problem is then one of finding the weakest cross section.

Worked Example 10.2

A series of measurements have been made of a badly corroded plate in the bottom of a ship’s hull.
Measurements are made at regular intervals across the width of the plate. The results are shown below
in Figure 10.7. Note the caret, “^”, is used to denote an estimate based on a sample of measurements at
points across the width of the plate. It is not the exact or true average. More sample points would give
an improved estimate.

Average plate
thicknesses across
plate width are
found to be:
tˆAv = 10.7

tˆAv = 10.2 Uncorroded plate


tˆAv = 9.8

tˆAv = 11.1
3m
Corroded plate
tˆAv = 11.7

tˆAv = 10.3

tˆAv = 10.9 Model 1


tˆAv = 11.4
1.3m

a) b) Model 2

Figure 10.7 a) Plan of corroded ship’s hull plate with measurements of average thickness, b) cross-
section showing how measurements were made.
Safety, Risk and Reliability, Heriot-Watt University 213
10. Reliability-Based Assessment of Existing Structures
The plate is subject to a uniform axial tension. If the yield stress of the plate is 280 N/mm2 estimate
the maximum load to which it may be subjected so that the probability of failure is not more than 10 -2.
Assume the plate thickness is normally distributed and each thickness measurement characterises a
strip 50 millimetres wide (Model 1 in Figure 10.6b).

Solution

For all across-width averages the overall average is

tˆAv 86.1
ˆ t    10.7625 mm
Av
n 8

  tˆAv2  n
and the variance is  2
  ˆ t2Av 
t Av  n  n 1
 

since t Av
2
 929.53

then ˆ t2  0.4113 and ˆ t  0.6413 mm.


Av Av

For a length of 3m there are (3103)/50 = 60 = n, 50 mm wide strips from which only 8 have been
chosen at random for sample measurements.

Using EV1 to characterise the distribution of the thinnest strip (see Section 9.3.5),

Fz1 ( z )  1  exp[  exp(1 ( z  u1 ))]

1  2 ln 60  2.862

ln ln n  ln 4
u1   1   2.174
21

 0.5772   0.5772 
and  z1   tAv   tAv  u1    10.7625  0.6413  2.174    9.239 mm
 1   2.862 

 t
also  z1  Av
 0.287 mm
1 6

z1   tAv
Now z  and for a minimum plate thickness having a probability of occurring of 10 -2,
t Av

Fz1 ( z )  0.01  1  exp  exp 1 z  u1 

i.e. ln 0.99   exp α1 z  u1 

Safety, Risk and Reliability, Heriot-Watt University 214


10. Reliability-Based Assessment of Existing Structures
ln  ln 0.99  α1 z  u1 

ln  ln 0.99 4.600
So z  u1   2.174   3.781 .
1 2.862

z1  10.7625
Hence  3.781  and thus z1  8.34 mm . So there is a probability of 10-2 that the
0.6413
thickness of the thinnest plate cross section is less than 8.34 mm (Figure 10.8).

The corresponding maximum load allowable is

F = 2801.31038.34 N = 3.036106 N

where the plate width = 1.3103 mm.

Note that this example assumes that the plate will fail at the weakest transfer strip; however, the
failure path could actually follow a zigzag path connecting weakest points resulting in slightly lower
strength than estimated here.

Now suppose that our strips are only 25mm wide rather than 50mm wide, as shown in Model 2 of
Figure 10.6b. How will this affect the results? In this case

3 x 103
n  120
25

Hence 1 = 3.094, u1 = -2.453 and z = -2.453 – 4.6/3.094 = -3.940

and z1 = -3.9400.6413 + 10.7625 = 8.24 mm, i.e. thickness estimate only ~ 1.2% different from the
case above. Thus, the choice of area characterised by each measurement is not too critical.

Figure 10.8 The area of the minimum extreme value distribution that corresponds to a probability of
failure of 10-2 is the area where the thickness is less than 8.34 mm.

Safety, Risk and Reliability, Heriot-Watt University 215


10. Reliability-Based Assessment of Existing Structures
10.4 Fracture

10.4.1 Types of fracture mechanism

There are three distinct types of fracture mechanism to which materials are subject:

 Brittle fracture

 Ductile fracture

 Fatigue cracking

Brittle fracture is normally fast and not preceded by significant plastic distortion or stretching. Ductile
fracture can occur slowly, and is often a tearing-like action, preceded by significant plastic distortion.
Fatigue cracking is the result of cyclic loading which produces an increment of crack growth on each
cycle. The increments get progressively larger until final fracture occurs after hours or years depending
upon the stress level and cycle frequency.

Fracture may be a mixture of fatigue and ductile or brittle fracture. Indeed many structures or
components contain small cracks or crack like defects during their manufacturing or fabrication process.
Others acquire them in service due to abrasion and/or fatigue; a small fatigue crack often leads to brittle
fracture in mechanical components.

Until recently fatigue, brittle and ductile fractures were treated in different ways but the advent of
fracture mechanics has provided a common basis for treating all fracture modes.

10.4.2 Notches and stress concentrations, cracks and the stress intensity factor

All the above types of fracture can occur when the average stress is below the yield stress with the
crack starting where there is a notch or hole or some other type of geometric discontinuity as illustrated
in Figure 10.9. In welded structures, fatigue cracking almost invariably begins in the heat affected zone
at the weld toe.

Safety, Risk and Reliability, Heriot-Watt University 216


10. Reliability-Based Assessment of Existing Structures
Regions of Crack starting from
Notch high stress welding defect
r

F A F

Figure 10.9 Regions of high stress occur at notches, geometric discontinuities and at welds.

The presence of a discontinuity causes the stress to rise locally to a much higher level; considerably
above the average stress (so-called "nominal stress") which would occur in the absence of the
discontinuity. The magnitude of this local stress, known as the hot spot stress, is often expressed in
terms of a stress concentration factor, denoted by KG.

Maximum stress at the root of the notch


KG 
Nominal stress  n 

then local stress = KG  n (10.1)

NB Here  corresponds to a stress and not a standard deviation.

For a notch the stress concentration factor increases as the radius at root of the notch gets smaller.
For an elliptical notch in a semi-infinite plate the stress concentration factor of the root can be
calculated from the expression

K G  1  2 Notch Depth/Root Radius   1  2 d / r (10.2)

The stress concentration factors for a whole range of differently shaped holes, notches, joints, etc., has
been calculated or determined experimentally and these have long been used in traditional fatigue
calculations.

In the fracture mechanics approach, however, a measure of the stress field at the actual crack tip is
needed rather than the stress level in the area where the crack is likely to start. At the very tip of a fine
crack the root radius tends towards zero and hence, from the expression above for an elliptical notch,
the stress concentration factor tends to infinity. To overcome this problem the "stress intensity factor"
KI is introduced

K I   a  (10.3)

Safety, Risk and Reliability, Heriot-Watt University 217


10. Reliability-Based Assessment of Existing Structures
where a is the depth of the crack (often simply related to its length), σ is the local stress (calculated
assuming the crack is not present), and ψ is a non-dimensional parameter which depends on the
geometric shape of the crack. It is known as the "crack shape parameter" and usually has a value around
unity (e.g., for a long surface crack it is 1.12). KI is fundamental to the fracture mechanics method and is
a measure of the severity or importance of a crack under given loading conditions. KI is defined in terms
of a crack being pulled open; cracks can also grow as a result of shearing and tearing action. These
modes of cracking are illustrated in Figure 10.10 below. The stress intensity is thus referred to as KI, KII or
KIII depending on the cracking mode. The opening mode is by far the most common and the only one
considered here.

a) Opening mode (I) b) Shearing mode (II) c) Tearing mode (III)

Figure 10.10 Cracking modes: opening, shearing and tearing.

Microscopic imperfections and small defects which occur frequently in welded structures are effectively
small cracks and these will grow and cause fracture under the right conditions.

10.4.3 Toughness and brittle or ductile fracture

Whether or not a piece of material under load fails in a brittle or ductile manner depends on the
"fracture toughness" of the material. Toughness is the ability to absorb energy both elastically and by
plastic distortion without fracturing. Brittle materials, like cast iron, have low toughness whereas plain
mild steels have relatively high toughness. The toughness of material can be measured in a number of
ways and the most popular method is the Charpy impact test. In the test a small sample containing a
notch is hit by a hammer mechanism and the energy required to cause fracture is recorded. More
recently fracture toughness has been measured in terms of the "critical stress intensity factor", KIC. This
can be done by taking a small coupon of the material, machining a sharp notch in it and applying a few
cycles of loading at a low stress level to start a fatigue crack. The specimen is then loaded statically until
the crack grows and fracture occurs. This is illustrated in Figure 10.11 below (see Benham et al. 1996, pp

Safety, Risk and Reliability, Heriot-Watt University 218


10. Reliability-Based Assessment of Existing Structures
521 - 523 for details). The critical stress intensity factor, KIC, at which fracture occurs is calculated from
the size of the initial crack and the applied stress.

For high toughness materials another test, known as the crack opening displacement test (COD) test
can be used. This is similar to the test just described but involves measuring how much the notch must
be opened before the crack starts to grow. This provides the ‘critical crack opening displacement’,
denoted by δC. There are simple equations that relate δC to KIC and so either of these measures of
fracture toughness can be used in fracture mechanics calculations. Students not familiar with fracture
mechanics can read pages 521 to 530 of Benham et al. (1996).

Figure 10.11 Three point bending test to determine KIC; the fatigue is started from the notch before the
test and its depth a is noted.

Now, if the loading around a small crack or defect gives rise to a stress intensity factor greater than
the critical stress intensity factor, then crack growth will occur, i.e. cracking occurs when

K I  K IC (10.4)

Bearing in mind that KI is proportional to crack size , it is desirable for structures to both be made
of a material with high KIC and for fabrication defects and any fatigue cracks to be as small as possible.
Critical defect, or crack sizes are often specified and if these are exceeded brittle fracture is likely. This
critical size can be estimated combining Eqs. (10.3) and (10.4) to give

K IC   aC  (10.5)

and rearranging gives

K IC2
aC  (10.6)
 2 2

Table 10.1 gives typical values of KIC at normal ambient temperatures. Note that there is a wide range
of values for KIC, not only between various materials but also for similar samples of the same material.

Safety, Risk and Reliability, Heriot-Watt University 219


10. Reliability-Based Assessment of Existing Structures
This uncertainty means that predicting critical crack size deterministically is prone to error and structural
reliability and risk assessment is widely used in considering the fracture resistance of critical
components and structures.

Table 10.1 Values of KIC for different materials.


Material KIC (MNm-3/2)
Mild steel 100 – 200
High-strength steel 30 – 150
Cast iron 6 – 20
Titanium alloys 30 – 120
Aluminium alloys 22 – 45
Carbon fibre reinforced plastic (uniaxial fibres)* 20 – 45
Glass fibre reinforced plastic (uniaxial fibres)* 10 – 100
Glass fibre reinforced plastic (laminate) 10 – 60
Wood* 8 – 13
Glass 0.3 – 0.7
Acrylic (PMMA) 1.0 – 2.0
Polycarbonate 1.0 – 3.5
Concrete 0.2 – 0.4
Epoxy 0.5 – 0.7
* Perpendicular to fibre direction

10.5 Fatigue

10.5.1 Introduction
Even in material of very high fracture toughness fatigue cracking can occur under cyclic loading if the
stress intensity is above a threshold level denoted by KIthres, although in corrosive environments, such as
sea water, all the evidence points to the threshold being so low as to be non-existent for practical
purposes.
For purposes of static loading we can consider steel and other structural materials as homogeneous
and isotropic, as it is the average properties we are concerned with. However, at a microscopic level this
is not true. Metallurgical specimens show steel to be composed of irregular shaped grains, with
inclusions, voids, slip-planes, etc. This means that under uniformly applied loading above KIthres, the
stress-strain pattern and the local strength and toughness vary considerably at the microscopic level.
Even in a smooth polished specimen subject to uniformly applied cyclic loading fatigue cracks will
eventually be initiated at a microscopic discontinuity and then grow rapidly until fracture occurs. For
machined items the onset of a measurable crack is usually taken as failure and the item replaced after
inspection reveals cracking.

In a welded structure, which will usually contain several small defects, there is no crack initiation
period and cracks will start to grow immediately at the defect sites. The rate at which a crack grows and

Safety, Risk and Reliability, Heriot-Watt University 220


10. Reliability-Based Assessment of Existing Structures
the time to failure will depend on the type of steel, the range of the cyclic stress in the region of the
crack and the size of the crack. Failure here is often taken as the point where the structure or
component can no longer perform its function, or when the fatigue crack is close to the critical crack size
for fracture.

10.5.2 S-N curves

Traditionally fatigue lives have been calculated using diagrams of hot spot stress range (where stress
range is constant and denoted by S) plotted against number of stress cycles (N); these are known as "S-N
curves" and a typical example is shown in Figure 10.12 below.

S-N curves are obtained by experimentally by testing a whole series of specimens under different
stress ranges until they break. A conservative design line is then drawn to the left of the failure points,
which are usually well scattered. Any combination of stress range and number of cycles falling to the left
of the design line is then unlikely to result in failure. For example, offshore structures that have a life of
30 years and are subjected to waves of average period of around about 6 seconds must be able to
endure more than 1.5108 load cycles. The crankshaft in car engine will have done around 3108 load
cycles when the car has done 100,000 miles.

The nominal stress range is the difference between the maximum and minimum stress in each load
cycle in the region of the joint

n   max   min (10.7)


Stress range S (log scale)

Co 22
ns
SAFE er UNSAFE
UNSAFE
va
ti ve
d es
i gn
l in
e
2 4
10
0
10 10 106 108 10
10

Number of cycles

Figure 10.12 Typical S-N curve for steel, showing mean line through experiment data and design line two
standard deviations to the left.

Safety, Risk and Reliability, Heriot-Watt University 221


10. Reliability-Based Assessment of Existing Structures
Here we are concerned with the so called ‘hot spot’ stress range (S) where the crack is likely to occur.
This is calculated by multiplying the nominal stress range (Δσn) by the stress concentration factor KG.

S   G  KG  n (10.8)

S-N curves are usually straight lines on log-log plots and the equation of the design line can be written as

log N  m log S  log C (10.9)

where C and m are constants.

This equation can also be written in the following manner, which is the usual form for expressing
fatigue design curves.

NS m  C (10.10)

Fatigue design curves have been produced for a wide range of materials, including composites, and a
wide range of component geometries including all common types of welded connections (e.g. British
Standard BS 5400 part 10).

10.5.3 Stress ranges and the use of Miner’s rule

For some structures and components the stress range is more or less constant throughout their lives;
for example the crankshaft on a diesel generator that operates continuously at the same speed. Other
structures such as the suspension units on a jeep that operates over different terrains and different
speeds will have a variety of stress ranges.

Here, as an example, we consider a tubular offshore truss structure (known as a ‘jacket’) where wave
action will produce cyclic loading. Waves come in different heights and consequently produce different
stress ranges. When a wave of a particular size passes the structure the corresponding cyclic stresses
can be calculated using frame analysis and the nominal stress range Δσn found and hence S obtained.
This can be repeated for all the wave sizes the structure is likely to encounter in its lifetime. The number
of waves of each size is then estimated, enabling a histogram to be plotted of number of cycles versus S.
This process is illustrated in Figure 10.13. Note that, in practice, an adjustment is required for stress
ranges below the threshold value but this is not considered here.

Safety, Risk and Reliability, Heriot-Watt University 222


10. Reliability-Based Assessment of Existing Structures
Number of stress cycles
Number of waves

Wave height Hotpot stress range, S

Figure 10.13 Schematic showing histogram of waves converted to corresponding histogram of stress
ranges.

Now to calculate the related fatigue life we use a simple empirical formula known as the Miner's
rule. This states that fatigue failure occurs when

p
ni
N
i 1
1 (10.11)
i

where ni is the number of cycles at stress range Si, and Ni is the number of cycles at this stress range,
which would cause failure, p is the number of stress range groups (11 columns in Figure 10.13).

For each of the stress ranges Si in the histogram the corresponding number of cycles to failure Ni is

found by entering the S-N curve as shown below in Figure 10.14. In design it is common practice to use
the concept of ‘cumulative damage’. The cumulative damage illustrated in Figure 10.14 is

Cumulative damage = n1S1m  n2 S2m

Related to this is ‘annual cumulative damage’ where the number of stress cycles per year in each stress
range is used. This represents the fraction of the design fatigue life that has been used each year, as per
Eqs. (10.10) and (10.11).

Safety, Risk and Reliability, Heriot-Watt University 223


10. Reliability-Based Assessment of Existing Structures
S

Stress range
S

n1 n2 N1 N2
Number of cycles

Figure 10.14 Schematic diagram illustrating use of Miner’s rule.

It is often required that the total ‘cumulative damage’ given by the summation in Eq. (10.11), is much
less than 1.0. Typically values of around η=0.4 are used in Eq. (10.12) below and 1/η is treated as a
safety factor

p
ni
Cumulative damage = N i 1
 (10.12)
i

10.5.4 Factors affecting fatigue life

There are several factors that affect the fatigue lives of components and structures.

 Defect Size: A defect from a structural point of view is the same as a small crack. The larger the
defect the larger the effective crack and the shorter the fatigue life. Increasing the depth of a defect
from 1 to 2 mm will halve the fatigue life of a 30 mm thick pipe subject to cyclic loading.

 Residual Stresses: The internal forces set up a result of the welding process give rise to large tensile
stresses near the surface around welds. These will increase the mean stress level of the cyclic loading
and reduce the fatigue life. Post-weld heat treatment, or peening, can double fatigue lives.

 Weld Profile: A poor weld profile can give rise to stress concentrations because of its shape. The
grinding of welds to improve their profile, which may also remove surface defects can improve
fatigue lives significantly.

 Aggressive Environments: Sea water, for example, when it enters a fatigue crack can set up a
Galvanic cell causing corrosion and further growth of the crack. Sea water therefore reduces fatigue
life although this reduction can be alleviated by proper cathodic protection. In chemical plants
careful attention has to be paid to this type of problem.

Safety, Risk and Reliability, Heriot-Watt University 224


10. Reliability-Based Assessment of Existing Structures
The above factors must all be considered when choosing an appropriate S-N curve or by introducing
factors into the fatigue calculation procedure. However there is still considerable uncertainty about how
long components will actually last in practice and a very conservative approach is usually taken.

10.5.5 Fracture mechanics and crack growth

From an inspection point of view a fatigue calculation is not all that helpful. Although, by comparing
fatigue lives for various joints, an estimate of which is likely to crack first can be made. It is not possible
to determine when cracks are likely to be first seen or how big they are likely to be at each stage of the
fatigue life.

An alternative to the traditional fatigue life calculation using an S-N curve is to use a fracture
mechanics approach. A crack growth model using fracture mechanics allows the crack size to be plotted
as a function of time as illustrated below in Figure 10.15.

In an analysis of the crack growth during a large number of fatigue experiments Paris and Erdogan
(see p 554 Benham et al. 1996) plotted the log crack growth rate against the log of the stress intensity
range and established an approximate straight line relationship; i.e.

log(crack growth rate) = mlog(stress intensity range) + log(constant).

 da 
log   m log K I   log( C )
 dn 

da / dn  C  ΔΚ I  = C  a
m
 
m
(10.13)

where a is the crack depth, n is the number of load cycles and C and m are constants that depend on the
material and are found empirically. The constant m is found to be the same as that used in S-N curves.

Through
crack
Crack depth

Initial
defect
depth 0 40 60 120
Time (years)

Figure 10.15 Crack growth estimation using Paris equation.

Eq. (10.13) can be rearranged and integrated


Safety, Risk and Reliability, Heriot-Watt University 225
10. Reliability-Based Assessment of Existing Structures
ax n
1
 C 
a0 a 
m
da   dn  n
0
(10.14)

where a0 is the depth of the largest likely fabrication defect (initial crack size) and ax is the crack size
after n cycles. Rearranging gives

1
ax
 m  1 2 
m m
1
1 m 1
n  a 2 da  1    a x  a0 
2

   
(10.15)
 2
m m
C   a0 C    

So Eq. (10.15) provides an estimate of the number of stress cycles to obtain a crack of size ax from an
initial crack or defect of size a0. This is very useful as it provides an idea of how large a crack is expected
to be any stage through the fatigue life. Inspection can reveal whether the crack has reached this size or
not. It also allows estimates of remaining life to be made using the measured crack size. These issues are
considered in this lecture.

When there are stress cycles of varying amplitude then using the information from the stress range
histogram an effective stress range Δσeff can be defined as below

N x  eff    ni  i 


m m
(10.16)

and hence

1
1  m  1 m2 1 
m
Nx  m 
1   ax  a0 2 
    (10.17)
C  eff   2  

From an inspection point of view the following rearrangement of this formula is very helpful

2
2 m

  m  
 m

m
1
ax  a 0
2
  N xC  eff  0.5 1    (10.18)
   2  

which gives the crack depth ax after Nx cycles.

If the time period of the average stress cycle is Tav then the depth of a crack ax after time Lx is simply

2
2 m
 1 m  L m  
a x  a0 2   x C  eff  0.5

 
m
1   
 2  
(10.19)
  Tav 

where Nx=Lx/Tav.

Safety, Risk and Reliability, Heriot-Watt University 226


10. Reliability-Based Assessment of Existing Structures
10.6 Modelling structural deterioration and fatigue cracking
The way structural deterioration is modelled will depend on the structure and type of deterioration.
In some cases it may be a very simple heuristic process. For example the rate of decay of an exposed
painted timber structure may be estimated from past experience and a time interval set for inspection
and replacement of rotted sections. For a corroding steel structure the rate of corrosion can be
estimated from earlier experience of similar environments and the loss of strength estimated as
described in Section 10.3. A limit state function can be set up for the corroded structure and the
probability of failure estimated. In the case of a simple solid circular steel tie rod this would have the
following form

GX  f y  r  cr,maxT   Wmax


2
(10.20)

where fy is the yield stress, r is the tie bar radius, cr,max is the maximum annual corrosion rate (estimates
the weakest part of the tie bar), T is number of years in service and Wmax is the maximum load. Apart
from T all these quantities would be variables with associated uncertainties. When a risk assessment
shows the probability of failure is becoming intolerably high then inspection would be needed.
Measuring the actual extent of corrosion will allow this current (a priori) model to be updated and the
strength term to be estimated more precisely. The new limit state function would have the form

Gx   f y Amin  Wmax (10.21)

where Amin is the minimum estimated cross section for the tie bar based on the inspection results. The
corrosion rate may, of course, be updated subsequent to inspection and used to predict residual
capacity in the future. (N.B. This limit state therefore applies to the time of the inspection and becomes
progressively less valid as time passes.) If the corrosion rate is negligible then no action will be needed;
however, if it is more than expected then either the loading must be reduced, the tie bar replaced or a
higher load accepted for the adjacent elements if the tie bar is redundant.

We now consider cracking in welded steel structures. Any welded steel structure will contain small
crack-like defects when it is first installed. These will exist despite post-fabrication weld inspection for
two reasons: the fallibility of the technician operating the equipment and the limitations of the
equipment itself.

Studies have been made of as-built steelwork to find the distribution of initial defect sizes and it has
been found that a Weibull distribution or an exponential distribution usually fit the data quite well. The
exponential distribution is the more logical choice, and the use of the Weibull may be related to the fact
that very small cracks are difficult to detect and will be missed by much measuring equipment. Typically

Safety, Risk and Reliability, Heriot-Watt University 227


10. Reliability-Based Assessment of Existing Structures
initial crack lengths in welded structural steel may have mean values around 11 mm with coefficients of
variation (COV) up to 0.7, and initial crack depths have means of around 1.2 mm with COV up to 0.5.

Using generic data, relevant to the intended post-fabrication inspection procedure and technique,
the structure can be assessed at the design stage to ensure the probability of fracture is sufficiently low.
The limit state function for this is:

GX  K IC  a0  (10.22)

where σ here means the stress in the region of the crack, a0 is the distribution of initial defect (or crack
size) and the other terms have their usual meaning.

If the structure is subject to fatigue loading then any small crack will grow during its life. Checks need
to be made at the design stage that at the end of the design life there is still an acceptably low risk of
fracture. In this case

GX  K IC  aend _ of _ life  (10.23)

where aend_of_life is the estimated distribution of crack size at the end of the design life.

The distribution of crack size can be estimated at any time Tx during the life of the structure. This can
be done in various ways, including Monte-Carlo simulation, using Eq. (10.18), where Nx represents the
number of cycles up to time Tx.

The probability density function fa(a) of the crack size can be estimated from this equation for various
times during the life of the structure using the Monte Carlo simulation. These probability density
functions may appear as in Figure 10.16 below.

ac
crack size

f(a0)

Time

Figure 10.16 Uncertainty of crack size increasing with time during life of a structure. Curves represent
the PDFs at each time given by the vertical base line with the f(a) axis increasing parallel to the time axis.

Safety, Risk and Reliability, Heriot-Watt University 228


10. Reliability-Based Assessment of Existing Structures
Here ac represents the critical crack size, which may be interpreted as a through-crack. An important
point to note is that the uncertainty about the crack size grows with time. It has been found that
lognormal distributions of the following form can be fitted to some estimated crack size distributions.

  ln ET b a     
2

f a  
1
exp  0.5   (10.24)
a 2     

where T is time in years and E and b are constants to enable the numerical PDF found for crack size to be
fitted by this analytical form. Note: b > 0, else crack size will decrease with time.

Using these PDFs the probability of exceeding a critical crack size ac, which may be related to fracture
or possible leak from a vessel or pipe, can be estimated both at the design stage and at any other stage
during the life of the structure.

10.7 Structural inspection


In a risk-based structural inspection programme there are essentially four types of decisions to be
made:

 What to inspect and where to sample?

 When to inspect?

 What techniques to choose for the inspection?

 How should the structural strength model be updated in light of the results of the inspections?

Similar decisions are needed for all types of inspection. Inspection should concentrate on those
critical areas where the risks associated with deterioration are greatest. There is limited value in an
inspection unless there is a significant probability of finding some change in the structure from its
original installed condition. However, an inspection that provides no evidence of deterioration is still of
some value and can be used to update the structural model as illustrated below. The probability of
observing deterioration will depend on the resolution of the inspection technique and the change
predicted in the structure by the design/as-built structural model. There are a wide variety of non-
destructive techniques for assessing deterioration.

When inspecting for cracks the probability of detection, POD(a), varies with crack size and the
inspection technique employed and the condition under which the inspection occurs, as illustrated in
the Figure 10.17 below.

Safety, Risk and Reliability, Heriot-Watt University 229


10. Reliability-Based Assessment of Existing Structures
1.0

MPI
0.8 Visual

probability of detection
0.6

0.4

0.2

0
0 10 20 22
 (mm)

Figure 10.17 Probability of crack detection and its variation with crack length for visual and magnetic
particle inspections (MPI) based on a study of offshore structures.

The first question is: what is the probability of finding a crack at a particular location, at a given time
into the structure's life, using a given technique? The probability of finding a crack of a given size is the
product of the probability of cracks of this size existing and the probability of the inspection finding the
crack. This probability will vary according to the crack size and so summing these products over all crack
sizes, or integrating can find the probability of finding a crack of any size. This particular integration is
termed a ‘convolution’. The convolution of the PDF of the crack size and the probability of detection
(POD) curve is then

f a a PODa da
amax
Probabilit y of crack detection   (10.25)
0

This integral can be evaluated numerically as shown in the following example.

Worked Example 10.3

For visual inspection underwater the probability of detection (POD) is given typically POD(a)≈1-e-a/10.
What is the probability of detecting a crack after five years whose PDF is given by the following
lognormal distribution

f a  
1  1  ln a 0.2T 2  1.386  2 
exp    
 
0.45a 2  2   
0.45  

where a is the crack length and T is time in years?

Solution

The following table contains the PDF of the crack size and the POD, together with their product for
various crack sizes. The integration can be performed numerically using the trapezoidal rule. As the first
Safety, Risk and Reliability, Heriot-Watt University 230
10. Reliability-Based Assessment of Existing Structures
and last values are zero and almost zero, the trapezoidal rule just requires the products to be summed
and multiplied by the common interval, which is 2 mm in this case (see Table 10.2).

Hence the probability of detecting any sized crack is


 f a PODa da  0.1715  2  0.343 (using the trapezoidal rule),
o

i.e. there is about 35% chance of finding the crack using visual inspection.

Clearly if the inspection occurred later in the life of the structure, when the crack is expected to be
larger, there would be more chance of finding it. However, not finding a crack still allows the crack size
model to be updated as is seen in the next section.

Table 10.2 Calculations of Worked Example 10.3.


Crack size a (mm) f(a) POD(a) f(a)POD(a)
0 0 0 0
2 0.135 0.181 0.0244
4 0.222 0.330 0.0733
6 0.098 0.451 0.0442
8 0.034 0.551 0.0187
10 0.011 0.632 0.0069
12 0.004 0.699 0.0028
14 0.001 0.753 0.0008
16 0.0005 0.798 0.0004
Total 0.1715

10.8 Updating in light of inspection


After a structure has been inspected our perception of its condition will improve. Of course, the
structure has not changed; it is just that we know more about its condition. Just as an inspection of an
older car by an expert tells us more about its condition and enables us to make better judgements about
it, an inspection will reveal a structure to be in a better or worse condition than we anticipated. A very
detailed inspection using good, accurate techniques will allow us to be more precise about the
estimation of its condition. This is illustrated in the following worked examples where crack growth is
used to illustrate the ways in which our structural model can be updated.

Worked Example 10.4

Magnetic particle inspection (MPI) has been used to look for the five year old cracks in Worked
Example 10.3 and nothing has been found. If the probability of detection a crack of size x using MPI is
given by

Safety, Risk and Reliability, Heriot-Watt University 231


10. Reliability-Based Assessment of Existing Structures
 x 
PODx   1  exp 
 2.6 

(here we use a to denote the size of the crack and x to denote its measured size in order to distinguish
between them), what can be inferred about the actual distribution of crack size in the structure?

Solution

The probability density function of crack size can be updated in light of this knowledge using Bayes'
rule (in its continuous form)

f a PND x 
f nd a | x   
 f a PND x da
o

P X i PY | X i 
c.f. (see Section 2.4.4) P X i | Y   n
 P X i PY | X i 
i 1

where PNDx  is the probability of non-detection of a crack of size x. Thus fnd(a|x) is the size

distribution of the crack given that we haven’t been able to detect it. This process is known as Bayesian
updating.

If PODx  is the probability of crack detection, then the probability of non-detection is

 x 
PND x   1  PODx   exp 
 2.6 

The denominator in the expression for Bayes' rule above is then given by

 a 
f a exp  
amax

0
da  0.123279  2  0.246 (using the trapezoidal rule)
 2.6 

f a  exp  a / 2.6
and hence f nd (a | x)  , which is shown tabulated in the last column of the table
0.246
below. The revised (conditional) probability can then be obtained numerically as shown in Table 10.3.

Consider now how the perceived size of the crack has changed following inspection. From the table
above before the inspection the probability of the crack size being more than 10mm is given by


 f a da  0.5  0.011  0.004  0.0013  0.0005  ...  2  0.02
10

(using the trapezoidal rule to evaluate the integral for the first column in the table above).

Safety, Risk and Reliability, Heriot-Watt University 232


10. Reliability-Based Assessment of Existing Structures
However, after inspection the probability of the crack size being more than 10mm is given by

10

 
f nd a | x   0.5  0.001  1.6  10 4  ...  2  0.0014

Hence as a result of not finding a crack on inspection the perception is that the crack has a 2% chance of
being greater than 10 mm deep is reduced to less than 0.15%.

Table 10.3 Bayesian updating - no crack found (Worked Example 10.4).


Crack f(a) x
2.6
x
2.6 f nd (a x)
e f(a) e
depth
0 0 1.0 0 0
2 0.135 0.463 0.063 0.257
4 0.222 0.215 0.048 0.195
6 0.098 0.099 0.010 0.041
8 0.034 0.046 0.002 0.008
10 0.011 0.021 2.3x10-4 0.001
12 0.004 0.010 4x10-5 1.610-4
14 0.0013 0.006 810-6 3.310-5
16 0.0005 0.002 110-6 410-6
Total = 0.123279

Figure 10.18 shows the original perceived PDF of crack size (a priori distribution – in statistics jargon)
is significantly different from the PDF modified in light of inspection (posterior distribution). Clearly not
finding a crack, using a powerful inspection technique, when one is thought to exist allows the perceived
size to be reduced significantly. This new perceived size can be used as a new starting point.

0.4

0.3 fnd(a|x)

0.2

f(a)

0.1

2%
0.15%
0
0 5 10 15
Crack size a (mm)

Figure 10.18 Probability density functions for perceived crack before and after inspection when no crack
is found.

Safety, Risk and Reliability, Heriot-Watt University 233


10. Reliability-Based Assessment of Existing Structures
The revised crack size PDF fnd(a|x) can now be used as the new starting point for estimating how the
crack may grow over the remainder of the structure’s life as illustrated in the Figure 10.19 below.

ac

crack size

f(a0)

inspection Time
point

Figure 12.19 Revised crack growth estimation following inspection. Broken line shows distributions in
the absence of inspection and full lines those after inspection and Bayesian updating. Curves represent
the PDFs at each time given by the vertical base line with the f(a) or f(a│x) axis increasing parallel to the
time axis.

This inspection result may mean the next inspection can be put off as the structure is safer than we
thought it was prior to the inspection. We now consider what happens if we find a crack using a
measurement technique that is not exact.

Worked Example 10.5

Consider another crack, with the same (a priori) crack size distribution as that in Worked Examples
10.3.and 10.4. This is inspected using the Alternating Current Potential Drop (ACPD) technique and a
crack is detected and estimated to have a size of 10 mm. This measurement will not be exact and the
standard deviation is assumed to be 1 mm. The measured crack size is further assumed to have a normal
distribution. If the measured crack is denoted by x then

 1  x  10  2 
g x  
1
exp    
1 2  2  1  

Bayes' rule can again be used to update the crack PDF in light of this inspection result

f a g x 
f a | x   
0 f a g x da
Safety, Risk and Reliability, Heriot-Watt University 234
10. Reliability-Based Assessment of Existing Structures
NB Here x is the measured size of the crack and a is the perceived size.

The revised probability density can be obtained in a tabular manner as below in Table 10.4.

Table 10.4 Calculations of Worked Example 10.5.


Crack depth a f(a) g(x) f(a)g(x) f(a|x)

2 0.135
3 0.241
4 0.222
5 0.157
6 0.098 1.310-4 1.310-5 0.001
7 0.058 0.004 2.310-4 0.019
8 0.034 0.054 0.0019 0.154
9 0.017 0.242 0.0041 0.332
10 0.011 0.399 0.0044 0.356
11 0.006 0.242 0.0015 0.122
12 0.004 0.054 0.0002 0.016
13 0.002 0.004 810-6 0.001
14 0.0013 1.310-4
Total 0.01234


The denominator is then  f a g xda  0.01234 1  0.01234 and hence the revised crack PDF
o

is

f a g x  f a g x 
f a | x   
  81 f a g x 
 f a g x da
o
0.01234

Examining the final column in Table 10.4 it is clear that the revised crack size distribution has a mean
close to 10 mm with a standard deviation of not much more than 1 mm. In general, in Bayesian updating
the more precise the new information (or measurement) the more precise the revised distribution.

In summary, it is possible to do the following:

 evaluate which inspection techniques are appropriate for finding a crack of a given size,

 revise the crack size PDF even when no crack is found if the POD curve is known,

 revise the crack size in light of a crack size measurement of known accuracy.

This illustrates what is generally true about all types of inspection for all types of structural
deterioration. This type of approach is used in more advanced industries, including offshore oil and gas
and the nuclear industry, and interest in this approach is now being shown by other industries.

Safety, Risk and Reliability, Heriot-Watt University 235


10. Reliability-Based Assessment of Existing Structures
However, in many cases Bayesian updating is not used and the results of inspection are used in a
deterministic manner to estimate the current condition of the structure. A judgement is then made
about whether or not the structure needs to repaired/replaced or whether it can remain in service.

Exercises
Exercise 10.1

The steel pipeline with a diameter of 1 m has been inspected after 10 years in service and it has been
discovered that its wall thickness has been reduced due to corrosion. For the purpose of the inspection
the pipeline was divided into 1000 segments, of which 80 randomly selected segments were inspected
in detail. Analysis of the inspection data has shown that the wall thickness of the 80 segments can be
described by a normal distribution with a mean of 9.2 mm and a standard deviation of 1.1 mm. If the
target safety index is 3.6, determine the maximum allowable pressure in the pipeline. Consider failure of
the pipeline due to yielding. For simplicity, treat the pressure and the yield stress of steel as
deterministic parameters; for the yield stress use a value of 250 MN/m 2.
Ans. 1.59 MPa

Exercise 10.2

(a) Calculate the probability that the pipeline has of a crack exceeding 6 mm in size based on the prior

distribution of crack size a, which is a lognormal distribution with parameters λ=1.2, ξ=0.6.
Ans. 0.1611

(b) The pipeline is inspected for cracks using a device whose probability of detection (POD) is given by
the exponential distribution with a mean crack detection size of 9 mm. Obtain values of the PDF of
the posterior distribution for the crack size a given no crack is detected by the device, with a taken
in steps of 2-mm intervals (up to 14 mm). If no crack is detected by the inspection, determine the
probability that there is, in fact, a crack exceeding 6 mm in size.
Ans. 0.1035

Revision Questions
10.1 List the principal ways in which structures can deteriorate.

10.2 What are the three principal approaches to dealing with structural deterioration?

10.3 How are minimum extreme value distributions used to model corroded structures?

Safety, Risk and Reliability, Heriot-Watt University 236


10. Reliability-Based Assessment of Existing Structures
10.4 Distinguish between a ‘stress concentration factor’ and a ‘stress intensity factor’.

10.5 What are the three modes of cracking associated with three critical stress intensity factors?

10.6 What is an S-N curve and what is the general form of its equation?

10.7 What is Miner’s Rule and how does it work?

10.8 What factors affect fatigue lives?

10.9 Explain why a component undergoing cyclic loading may fail by fracture.

10.10 How can the limit state function be formulated for failures associated with structural
deterioration and fatigue?

10.11 How would you decide when to inspect a structure using a given technique?

10.12 What is Bayesian updating and how can it be useful for analysing inspection data?

SUMMARY

 The principal modes of structural deterioration such as fatigue, corrosion, cracking and fracture,
etc. have been briefly explained. The principal approaches of dealing with structural deterioration
have been presented.

 Different types of corrosion have been briefly described. It has been explained how extreme value
theory can be used to model the effect of corrosion on strength of metal structures. This has also
been demonstrated through worked examples.

 Basic concepts of fracture mechanics such as stress concentration, intensity factors, toughness, etc.
have been introduced.

 Basic tools for estimating fatigue lives such as S-N curves and Miner’s rule have been described.
Connection between fatigue and fracture mechanics has been explained and the Paris-Erdogan
equation has been presented.

 Modelling of structural deterioration has been discussed in the context of corrosion and fatigue
cracking and associated limit state functions have been presented.

 Updating crack size distributions, using inspection data and Bayesian updating, has been shown
through worked examples for the cases when no crack is found and when a crack is found and
measured.

Safety, Risk and Reliability, Heriot-Watt University 237


10. Reliability-Based Assessment of Existing Structures
References
Benham, P.P., Crawford, R.J. and Armstrong, C.G. (1996). Mechanics of Engineering Materials. 2nd
Edition, Longman.

Safety, Risk and Reliability, Heriot-Watt University 238


10. Reliability-Based Assessment of Existing Structures
APPENDIX 1 – BASICS OF STRENGTH OF MATERIALS
A1.1 Introduction
This appendix covers the basic mechanics of materials that is essential to the course. All students
should read this material and be sure that they are familiar with the material. Depending on your
background there may be bits you have not seen before. If you require more details then look at
Benham, Crawford and Armstrong or a similar mechanics of materials textbook. A tutorial sheet is
provided so you can test yourself.

This Appendix provides an introduction to elastic theory of thin shells and bending of beams, plastic
theory for beams, and buckling. Fatigue, brittle fracture and corrosion are introduced later in the
module.

A1.2 Elastic Theory


There are many types of structures and mechanisms whose design and safety assessment is based on
elastic theory. These are structures and components which are considered to fail when the loading is
sufficiently high to cause the stress to exceed the yield stress of the material from which they are made.
Failure may be distortion in a ductile material or fracture in a brittle material where fracture occurs at
the elastic limit. There are a number of yield criteria, which are described in Benham, Crawford and
Armstrong Chapter 12. However, in this module we will use just one criterion - the maximum principal
stress criteria - and that is considered below in the context of shells.

There are many structural configurations that can be analysed using elastic theory but in this module
we will cover only bars, beams and shells.

The stress in a bar is the simplest case. If we divide the applied axial load by the cross sectional area
of the bar we get the axial stress. If this exceeds the yield stress then we have failure (according to the
maximum principal stress theory - indeed the axial stress and the principal stress are one and the
same!). When the load is removed the bar will not return to its original length. If the stress exceeds the
ultimate stress then fracture will occur. For brittle materials the yield stress and the ultimate stress are
essentially the same.

A1.3 Theory of Thin Shells


Thin shells have been chosen for consideration here because: a) they exist in large numbers as
pressure vessels, silos, storage tanks, pipes etc.; b) many of them contain hazardous materials, such as
gas, under potentially hazardous conditions, high pressure; c) because they are simple to analyse.

Safety, Risk and Reliability, Heriot-Watt University 239


Appendix 1 – Basics of Strength of Materials
Shells are classified as thin shells when D/t > ~20, where D is the shell diameter and t is the
thickness. Other shells are classified as thick shells and a more complex theory must be used.

Membrane stresses only are considered in thin shell theory. A membrane can be viewed as a two
dimensional generalisation of a piece of string: a piece of paper is an example - it cannot resist a bending
load. A plate on the other hand can and a plate can be seen as a two dimensional generalisation of a
beam.

We start by considering a shell which is curved in both directions but with different radii of
curvature: a rugby ball is an example. The stress in the shell, as 1 and 2, must be in equilibrium with
the pressure in the shell.

R2

  
S

t 
R1

S



Figure A1.1 This sketch shows an elemental section of the shell that has a thickness t and where S1 and
S 2 are the lengths of the edge of the element.

Consider now the radial equilibrium for the stress 1 using Figure 2 below. There will be inward
component of force produced by the stress 1 acting on the lengths S 2 (see Figure A1.1).







R1



Figure A1.2 Sectional view of shell element.

Resolving horizontally:

Safety, Risk and Reliability, Heriot-Watt University 240


Appendix 1 – Basics of Strength of Materials
the inward force due to the stress 1 when the angles are small equals to

 1
2 1 S 2 t sin =  1 R2 2t 1
2

similarly, the inward force due to the stress 2 equals to

 2 R1 1t 2

This must be balanced by the force produced by the pressure in the shell acting on this element and
from Figure A1.1 this is clearly given by:

pressure force = pressurearea = pS1S 2 = pR1 1 R2 2

Therefore,  1 R2 2t 1 +  2 R1 1t 2 = pR1 1 R2 2 , which gives

1 2 p
  (A1.1)
R1 R2 t

This simple equation can be applied to all thin shell problems and so is very useful. Note that  1 and  2

are principal stresses and failure occurs when the larger exceeds the yield stress under the maximum
principal stress criteria.

Now we consider some particular cases:

(i) A sphere, where R = R1 = R2

2 p pR
which gives from equation (A1.1):  and so, 
R t 2t

(ii) A cylinder or section of pipe; here R2 = 

1 2 P pR
and thus from equation (A1.1)   so that the “hoop stress”  1 
R1  t t

In this case  1 is known as the hoop stress as in often denoted by  H .

Consider now the axial equilibrium of a cylinder or pipe as illustrated by the section in Figure A1.3
below. For axial equilibrium the force produced by the pressure acting on the internal cross section of
the pipe must be balanced by the longitudinal stress in the wall of the pipe. Note we do not distinguish
between internal and external radius because the shell is ‘thin’.

Safety, Risk and Reliability, Heriot-Watt University 241


Appendix 1 – Basics of Strength of Materials
m Pressure
P area = R2

Figure A1.3 Longitudinal equilibrium in a cylinder or pipe.

To maintain equilibrium in the longitudinal direction:  m 2Rt  pR 2

pR
i.e. the “meridinal” (or “axial”) stress m 
2t

It is important to note that that the hoop stress is twice the axial stress. So when a pipe fails under
pressure it bulges and then splits parallel to the axis.

(iii) A cylindrical tank on a solid base - here the pressure varies with the depth of liquid

Pressure
X

Figure A1.4 Tank of liquid on a solid base.

pR gxR
Consider the hoop stress at position X:  H   , where  is the density of the liquid. In this
t t
case axial equilibrium gives m = 0.

N.B. If the base is not solid bending occurs and thin shell theory does not apply. However, it will apply to
a silo as illustrated in Figure A1.5 below.

Figure A1.5 On the left is a tank with an unsupported flat base, and on the right a silo with a conical
base.

Safety, Risk and Reliability, Heriot-Watt University 242


Appendix 1 – Basics of Strength of Materials
General points

1. The theory above applies to both internal and external pressure but under external pressure
buckling may occur – see later notes

2. With gas pressure, variation of stress and strain with depth is negligible, and pressure is constant
at a given temperature. This is not so for a liquid where small volume changes can significantly
affect the pressure - hydraulic systems use this effect.

3. Plastic distortion/stretching will occur before fracture in most thin pressure vessel at ambient
temperature; but see Key Point below.

Key point: In this theory we have not considered any relationship between stress and strain; so it will
apply even when the stress is beyond yield and right up to fracture. However if the shell bulges out of
shape significantly before failure then the change in geometry must be taken into account.

Exercises
A1.1 In a pressure vessel comprising a cylinder with hemispherical end caps where does the maximum
principal stress occur and it what direction does it act?

A1.2 When selecting a pipe for a high pressure system why is diameter important from a structural
perspective? What happens to the stress if the diameter of the pipe is doubled? (Assume all other
parameters remain unaltered).

A1.3 In which direction would a crack run if a pipe where pressurised until rupture? Why?

A1.4 Beam theory – elastic bending of beams


The elastic theory of beams is used to calculate the stress in beams and also their deflection under
load, up to the point when first yielding occurs. Consider an element of a beam of constant symmetric
section bent by an applied moment M to some local radius R as shown in Figure A1.6. Before bending
the length of the shaded sub-element was R, i.e. the same length as along the neutral axis along
which there is no strain.

y
The new length = (R+y) and therefore the extension = y and the strain    so that
R  y 
y
 for y << R.
R

Ey  E
Now  = E, and thus   or  .
R y R

Safety, Risk and Reliability, Heriot-Watt University 243


Appendix 1 – Basics of Strength of Materials


y = -t1
ral
neut
axis
y
 
y
y = t2

Figure A1.6 Section of a beam bending to an arc of radius R.

Consider the moment produced by the stress acting on the end of the sub-element, about the
neutral axis as illustrated in Figure A1.7.

Ey Ey 2
M   Ay  Ay  by
R R

where b is the breadth of the section at y. Integrating the total moment is given by

 t1 E 2 E  t1 EI
M  y bdy   b y 2 dy 
t 2 R 
R 2 t R

M E
from the definition of I the second moment area. Hence  , and using the equation derived above
I R
M E 
we can write   .
I R y

Usually we are more interested in how much a beam deflects from the horizontal when it is under
load in its radius of curvature as illustrated in Figure A1.8 below.

Safety, Risk and Reliability, Heriot-Watt University 244


Appendix 1 – Basics of Strength of Materials
b

A

y
Neutral
axis

Cross- Stress
section distribution

symmetry
Axis of

Figure A1.7 Beam cross section and bending stress distribution.

kg LOAD

Figure A1.8 Deflection of a beam under load.

If x is the distance from one end of the beam to where the deflection y is measured a basic formula
from analytical geometry relates the deflection to the curvature

1  d 2 y dx 2

R 1  dy dx 3 2

1 d2y dy
which gives  2 when  1 , the sign ± depends on the sign convention.
R dx dx

Substituting this in the expression above yields

 M d2y
 E 2
y I dx

which is known as the engineer’s bending formula and all elastic beam theory is based on this equation.

The deflection in y is found by integrating this equation:

Safety, Risk and Reliability, Heriot-Watt University 245


Appendix 1 – Basics of Strength of Materials
d2y M M
2
 giving y   dxdx
dx EI EI

and substituting appropriate boundary conditions. The term EI is known as the elastic modulus.
Formulae for the deflection of beams with various types of support and patterns of loading are given in
the Formula Sheet. This also gives formulae for the second moment of area I for various section shapes
e.g. for a solid rectangular cross section of breadth b and depth d,

bd 3
I .
12

Further formulae can be found in Benham, Crawford and Armstrong.

Exercises
A1.4 If the depth of a beam of solid rectangular section is doubled by how much will the maximum
bending stress change?

A1.5 Where will the bending stress in a beam be zero? When will this the mid-depth point and when
not?

A1.6 Look at the Formula Sheet (Appendix 5). If the length of a cantilever beam with an end load is
increased by 50% by how much will the end deflection increase?

A1.5 Plastic theory for beams


Elastic theory breaks down after the onset of yielding. However, most structures and many
components can support an increased load beyond this point without collapsing. For simplicity, material
behaviour is usually simplified, and we use an idealised elasto-plastic material in our calculations as
shown in Figure A1.9.

egion
Yield stic r Yield
point Pla Fracture point
y y
Idealised plastic behaviour
gion
Stress 

Stress 
Elastic re

Strain  Strain 
REAL MATERIAL IDEALISED MATERIAL

Figure A1.9 Stress strain relationship for steel and corresponding idealised relationship.

Safety, Risk and Reliability, Heriot-Watt University 246


Appendix 1 – Basics of Strength of Materials
Consider a beam of idealised elastic-plastic material as the load is increased as illustrated in Figure
A1.10. When plasticity has spread right through the entire depth of the section of the simply supported
beam it is incapable of sustaining further load and will form what is known as a “plastic hinge” at this
point. It will simply fold up in the middle.

neutral
Elastic
axis

Cross-
section W+
y

Elastic neutral
Plastic axis
y

y Plastic regions
W++
neutral
Plastic
axis
y

Figure A1.10 Growth in plasticity as loading is increased and corresponding stress distribution.

y

c1 F1=A1y
A1

c2

A2 F2=A2y
y
Cross-
section Stress distribution
Figure A1.11 The cross section a beam which fully plastic showing the stress distribution and the internal
couple or moment generated.

Consider the beam cross-section when this happens (Figure A1.11). For equilibrium - F1 = F2 and so -
cross sectional area A
A1  A2   , i.e. the plastic neutral axis divides the section into two equal
2 2
areas; c1 and c2 are the centroids of each of these areas where internal forces F1 and F2 act.

The internal moment generated (known as Mp) is

Safety, Risk and Reliability, Heriot-Watt University 247


Appendix 1 – Basics of Strength of Materials
A
M p  F1 c1c2   y c1c2  S y
2

where S is the plastic section modulus (c.f. Z = I/y the elastic section modulus).

Hence for any section of any material the bending moment to cause a plastic hinge Mp (also known
as the ultimate moment of resistance) can be found. First the plastic section modulus is calculated; for
example for a rectangular section shown in Figure A1.12 below it is simply

bd d bd 2
S x 
2 2 4

This then multiplied by the yield stress of the material to get Mp.

c1

c2

Figure A1.12 Rectangular cross section of a beam.

A1.5.1 Collapse mechanisms

For a cantilever or a simply supported beam only one plastic hinge is required for a collapse
mechanism as illustrated in Figure A1.13.

w per metre Plastic hinge


kg kg kg kg kg kg kg kg kg kg kg kg kg kg kg kg kg kg kg kg kg kg kg k
g kg
kg k
w per metre
c
g kg
kg k
g kg
kg k
l g kg
kg k
g kg
kg kg
kg k
g kg
kg k
g

Figure A1.13 This shows what happens as the distributed load w on a cantilever is increased until
collapse occurs. The load at collapse has a subscript c to denote ‘collapse’.

wc l l wc l 2 wc l 2
Taking moments about the support at collapse M  .  , i.e., M p  .
2 2 4 4

At the point of collapse the external moment must equal the internal moment for equilibrium.

Safety, Risk and Reliability, Heriot-Watt University 248


Appendix 1 – Basics of Strength of Materials
However, for a fixed ended beam under a uniformly distributed load the load may still be increased
after the section at each end is fully plastic as shown in the series of illustrations in Figure A1.14. In this
case three plastic hinges are needed to get a collapse mechanism that will allow the beam to fold up.
(Note that we assume in this simple plastic theory that the beam can pull in horizontally from the
supports to allow the folding to occur.)

w
kg kg kg kg kg kg kg kg kg kg kg kg kg kg kg kg kg kg kg kg kg kg Elastic
case
l
Increasing w

After
kg kg kg kg kg kg kg kg kg kg kg kg kg kg kg kg kg kg kg kg kg kg Mp/2<M<Mp formation
of support
Plastic hinges Mp hinges

wc Mp
kg kg kg
kg kg
kg kg
kg kg 
kg kg
kg kg
kg kg Collapse
kg kg kg kg
kg kg kg kg
Mp

Collapse mechanism

Figure A1.14 Beam with ‘built-in’ ends under increasing load showing the development of sufficient
plastic hinges for a collapse mechanism and the corresponding bending moment diagrams.

Now at collapse the work done by the external load is equal to the product of the load and the
average deflection


WEx  wc l
2

Internal work is done due to the rotation at plastic hinges and in this case this is:-

 M p 
WInt  2   M p  2M p ,
 2 

  8M p 
but from the diagram  for small therefore WInt  .
2 l 2 l

 8M p 
Now for equilibrium WEx = WInt and so wc l  . Therefore, the distributed load to cause
2 l
collapse is

16M p 16 y S
wc  2

l l2

i.e., the collapse load for a beam of any section, length and material can easily be found.

Safety, Risk and Reliability, Heriot-Watt University 249


Appendix 1 – Basics of Strength of Materials
Plastic theory is frequently used in the design of structures and components which may fail under a
single large load application. The simple approach is to use a load factor (LF) specified in an appropriate
Code of Practice.

Collapse Load Wc 


LF 
Max Working Load WN 

Hence Wc  LF  WN .

By looking at the various collapse mechanisms the minimum safe value of Mp can be found for each
beam element. Then

M p min
S min 
y

Hence any actual sections for which Sactual  Smin are acceptably safe. Note that, in practice, there are
limits to be applied to the geometry of the section.

Exercises
A1.7 What is the difference between plastic failure and elastic failure?

A1.8 Explain the difference between plastic section modulus and elastic section modulus.

A1.9 Explain how you would calculate the internal work done in plastically deflecting a 2m long
cantilever by 0.3m.

A1.10 Draw the collapse mechanism for a propped cantilever carrying a distributed load showing the
positions of the plastic hinges.

A1.6 Buckling
Compressive loading on a slender column causes buckling when the load exceeds a certain limit.
There are two stages:

1. Elastic buckling where the shape is regained after removal of the load.

2. Plastic buckling where a larger load causes permanent distortion which remains after load
removal.

A1.6.1 Elastic buckling of columns

Pin-ended strut with axial load

Safety, Risk and Reliability, Heriot-Watt University 250


Appendix 1 – Basics of Strength of Materials
Consider the strut in Figure A1.15 where the ends are free to rotate - pin ends - under load in a
buckled position.

PE

PE

Figure A1.15 Buckled strut.

It is readily seen that the bending moment at x is

M = -PEy.

d2y
Now EI  M   PE y
dx 2

d 2 y PE
therefore  y  0,
dx 2 EI

d2y
or 2
 m2 y  0 .
dx

This has a general solution y  Asin mx  B cos mx

but at x = 0, y = 0 and therefore B = 0

and at x = l, y = 0 and so 0 = Asinml

PE l 2
i.e. ml = 0, , 2 etc. and  0,  , 2
EI

where zero is a trivial solution as it implies PE = 0, i.e. no buckling.

The solution  is a real physical solution where the observed buckled shape is

Safety, Risk and Reliability, Heriot-Watt University 251


Appendix 1 – Basics of Strength of Materials
x PE l 2
y  Constant.sin and 
l EI

The solutions ml = 2 etc. are only mathematically feasible possibilities and do not occur in practice.

Rearranging the last equation gives

 2 EI
PE 
l2

which is known as the Euler critical buckling load.

N.B. I is the lowest second moment of area of the cross section, i.e. buckling occurs perpendicular to the
axis of the lowest I value

Imin

section shape buckling direction

Figure A1.16 Direction of buckling perpendicular to axis of minimum I .

Now by dividing by the cross section area A of the strut or column we find the buckling stress:

PE  2 EI
2
k
E     2 E 
l
2
A Al

where we have written I = Ak2, and k is known as the radius of gyration and (l/k) is known as the
slenderness ratio (SR).

Similar expressions for the buckling load can be found for columns with other end conditions
(boundary conditions) as illustrated in Figure A1.17 below.

4 2 EI
a) Fixed Ends - PE 
l2

 2 EI
b) One free end - PE 
4l 2

2.05 2 EI
c) One end fixed, the other simply supported - PE 
l2

Safety, Risk and Reliability, Heriot-Watt University 252


Appendix 1 – Basics of Strength of Materials
PE PE PE

PE

Figure A1.17 Buckled columns - left to right - both ends fixed; one end free; one free and one end fixed.

2
k
In general we can write: PE = Constant.EI, or  E  Constant.E  
l

Exercises
A1.11 Why is a tube a more efficient cross section shape for a slender column than a hollow
rectangular section? In which direction will a rectangular section buckle?

A1.12 The mast, a solid circular wooden pole, on a small sailing boat is subject to a large compressive
load when the sail is raised. The mast is unstayed, i.e. the top of the mast has no support. If the
top of the mast were to be supported by stays by how much would the critical buckling load
increase? By what percentage could the mast diameter be reduced while still retaining the same
factor of safety?

A1.6.2 Plastic buckling of columns

If the loading on a slender column increases, yielding will occur when the combined axial and
bending stress reaches the yield stress, i.e. when

 A  B   y

For pin-ended columns

P  2 EI M M P
A   and  B   
A Al 2
I y  Z Z
N.B. Load increases only slightly after initial buckling to first yielding.

Safety, Risk and Reliability, Heriot-Watt University 253


Appendix 1 – Basics of Strength of Materials
P P P P
The total stress =  Y   and thus  Y  (A1.2)
A Z Z A

Z y
Z Z y l
2
Z
Therefore     2 
P A  EI A

Worked Example A1.1

What is the deflection at yield for a bar steel 1 m long and 12.54.8 mm cross section if the steel
has Y = 270 Nmm-2 and E = 2105 Nmm-2?

Now A = 12.54.8 = 60 mm2 and I = 12.54.83/12 = 115.2 mm4

I 115.2
Hence Z    48 mm3,
y 4.8 2

48  270  1000 2 48
and therefore    56.19 mm.
  2  10  115.2 60
2 5

P
N.B. From equation (A1.2) the deflection at the onset of yield becomes very small as   y i.e. as
A
E  y .

Now as illustrated in Figure A1.18, experiments show that elastic buckling does not occur in short
columns of low slenderness ratio.

cr

y

short stocky long thin


columns Slenderness columns
ratio (SR) = (l/k)

Figure A1.18 Experimental results for the buckling stress of columns compared with calculated Euler
buckling stress.

Experiments show that the Euler buckling theory breaks down when  cr  0.6 y . At this point

empirical formulae that provide a fit to the experimental data are used. There are several such
formulae. For example

Safety, Risk and Reliability, Heriot-Watt University 254


Appendix 1 – Basics of Strength of Materials
1 1 1
1. Rankine-Gordon formula  
 cr E y

 y 
2. Det Norske Veritas formula  cr   y 1   for  E  0.5 y
 4 E 

Other factors can also reduce the buckling load below PE including

a) initial imperfections, i.e. out of straightness;

b) eccentricity in applied load, i.e., load applied off centroid;

c) other applied moments on the column causing it to bend out of vertical.

(a) and (b) can occur due to poor manufacture or construction and can be handled using more complex
modelling. (c) can be treated using interaction formulae of the type shown in Figure A1.19 where the
crosses represent experimental results.

1.0
P/Pcr

M/Mult 1.0

Figure A1.19 Interaction diagram for beam or column subject to both bending and axial loads.

The buckling load Pcr and the ultimate moment of resistance Mult are calculated for the beam-
column, and the applied load P and moment M are also found. Then if the formula below is satisfied the
beam-column is deemed to be safe and if it is not it is likely to fail

q r
 P   M 
      1.0
 Pcr   M ult 

where Mult is the plastic moment to cause collapse and Pcr the buckling load, M and P are the design
loads, q and r are positive indices. If in doubt use q = r = 1 to provide a straight line – a “lower bound”.
Obviously, the lower the value from this expression the safer the beam-column. Values close to 1.0 will
have little margin of safety.

Safety, Risk and Reliability, Heriot-Watt University 255


Appendix 1 – Basics of Strength of Materials
Exercises
A1.13 What happens when a short stocky column is subject to excessive compressive load?

A1.14 Give an example of a structure that behaves as a beam columns.

Revision Questions
These questions are provided to help you with the material in this Appendix. If you are familiar with
this material then you do not need to do them.

A1.15 A circular metal rod of length 25 cm and diameter 1 cm is found to have an extension of
0.0227cm when loaded in tension with 5 kN. What is the Young's modulus? The loading is
increased and the bar is found to yield at a load of 20 kN and finally to break when the load
reaches 37.5 kN. What are the yield and ultimate tensile stresses?
A thin wire rod, 750 m long and 0.5 cm diameter, is made from this material and is working a
signal. Assuming a pull on the wire of 1.5 kN, find the movement which must be given to the
signal box end of the wire if the movement at the signal end is to be 17.5 cm. What would be
the minimum diameter for the movement at the signal box not to exceed 25 cm?

A1.16 A circular cylinder of internal bore 30 cm and wall thickness 1.25 cm which contains hydraulic
fluid and has one end sealed, has a piston sliding in the other end. If the yield stress of the
material is 210 MNm2 and the factor of safety against yielding is to be 1.6 calculate the
maximum force that can be applied to the piston.

A1.17 A small pressure vessel is to comprise a 20 cm long circular cylinder of 8 cm diameter with
hemispherical end caps. The thickness of the cylinder and end caps is to be 1mm. What is the
maximum pressure it operates if the factor of safety against yield is to be 1.4? The material
for the pressure vessel has the following properties:

y = 300MNm2

E = 100GNm2

ult = 400MNm2

ult = 0.12

Can the end caps be safely reduced in thickness? If so by how much?

Safety, Risk and Reliability, Heriot-Watt University 256


Appendix 1 – Basics of Strength of Materials
What pressure (approximately) is required to cause the cylinder to tear apart? Ignore the effects
of plastic straining. How would the effects of plastic straining effect the pressure at which
rupture occurs?

A1.18 A burglar wishes to break into premises through a window protected by iron bars set rigidly into
the sill and lintel of the window. The bars, he estimates, are between 25 mm and 30 mm in
diameter and appear to be of mild steel which probably has yield strength of 260 Nmm2. The
bars are 2m in length and the burglar intends to prise them apart using a car jack at their
midpoint. Estimate the minimum load capacity of the car jack he may need to employ.

A1.19 A series of metal plates 0.3 m wide and 6 mm thick are used to span a gap 1.2 m wide to form a
temporary walkway. The plates are simply laid across the gap with 40 mm overlapping the
supporting edges on each side. The public will be walking underneath the temporary walkway
and men working up to 10 metres above it. There is concern that an object dropped at 10
metres above the walkway and landing on one of the plates would bend it in the middle and
cause a large central deflection sufficient for the outer edges of the plate to pull in and off the
supporting edges allowing the plate to fall on the public below.

Calculate the rotation (in radians) required at a central plastic hinge for a plate to pull in 40 mm
from each edge. What mass would have to be dropped from 10 m above the walkway to cause a
plastic hinge with this amount of rotation?

Assume the yield stress of the material of the plates is 250 Nmm2.

A1.20 A student during rag week wants to fly an unusual flag from the top of an aluminium pole. The
pole is 8.6 m high with a constant tubular section of 54 mm, mean diameter and wall thickness 5
mm. The student appreciates that to minimise the chances of buckling he must keep his centre
of gravity at the axis of the pole. If he estimates his weight as between 70 and 75 kg what would
you estimate are his chances of putting the flag on the top of the pole? Assume E = 70 × 109 Nm2

The student decides to repeat the trick, but up a solid wooden flag pole which is known to be
6.6m high. However he is more concerned about the timber pole breaking. What minimum
diameter would it have to have at the base to ensure a factor of safety of 1.5 if E = 12109 Nm2
and he weighed 75 kg?

Safety, Risk and Reliability, Heriot-Watt University 257


Appendix 1 – Basics of Strength of Materials
APPENDIX 2 - Numerical values of the standard normal CDF, (u).
u 0.00 0.01 0.02 0.03 0.04 0.05 0.06 0.07 0.08 0.09
-4.90 0.0000005 0.0000005 0.0000004 0.0000004 0.0000004 0.0000004 0.0000004 0.0000003 0.0000003 0.0000003
-4.80 0.0000008 0.0000008 0.0000007 0.0000007 0.0000006 0.0000006 0.0000006 0.0000006 0.0000005 0.0000005
-4.70 0.0000013 0.0000012 0.0000012 0.0000011 0.0000011 0.0000010 0.0000010 0.0000009 0.0000009 0.0000008
-4.60 0.0000021 0.0000020 0.0000019 0.0000018 0.0000017 0.0000017 0.0000016 0.0000015 0.0000014 0.0000014
-4.50 0.0000034 0.0000032 0.0000031 0.0000029 0.0000028 0.0000027 0.0000026 0.0000024 0.0000023 0.0000022
-4.40 0.0000054 0.0000052 0.0000049 0.0000047 0.0000045 0.0000043 0.0000041 0.0000039 0.0000037 0.0000036
-4.30 0.0000085 0.0000082 0.0000078 0.0000075 0.0000071 0.0000068 0.0000065 0.0000062 0.0000059 0.0000057
-4.20 0.0000134 0.0000128 0.0000122 0.0000117 0.0000112 0.0000107 0.0000102 0.0000098 0.0000093 0.0000089
-4.10 0.0000207 0.0000198 0.0000189 0.0000181 0.0000174 0.0000166 0.0000159 0.0000152 0.0000146 0.0000140
-4.00 0.0000317 0.0000304 0.0000291 0.0000279 0.0000267 0.0000256 0.0000245 0.0000235 0.0000225 0.0000216
-3.90 0.0000481 0.0000462 0.0000443 0.0000425 0.0000407 0.0000391 0.0000375 0.0000359 0.0000345 0.0000330
-3.80 0.0007235 0.0006948 0.0000667 0.0000641 0.0000615 0.0000591 0.0000567 0.0000544 0.0000522 0.0000501
-3.70 0.0001078 0.0001036 0.0009961 0.0009574 0.0009201 0.0008842 0.0008496 0.0008162 0.0007841 0.0007532
-3.60 0.0001591 0.0001531 0.0001473 0.0001417 0.0001363 0.0001311 0.0001261 0.0001213 0.0001166 0.0001121
-3.50 0.0002326 0.0002241 0.0002158 0.0002078 0.0002001 0.0001926 0.0001854 0.0001785 0.0001718 0.0001653
-3.40 0.0003369 0.0003248 0.0003131 0.0003018 0.0002909 0.0002803 0.0002701 0.0002602 0.0002507 0.0002415
-3.30 0.0004834 0.0004665 0.0004501 0.0004342 0.0004189 0.0004041 0.0003897 0.0003758 0.0003624 0.0003495
-3.20 0.0006871 0.0006637 0.0006410 0.0006190 0.0005976 0.0005770 0.0005571 0.0005377 0.0005190 0.0005009
-3.10 0.0009676 0.0009354 0.0009043 0.0008740 0.0008447 0.0008164 0.0007888 0.0007622 0.0007364 0.0007114
-3.00 0.0013500 0.0013060 0.0012640 0.0012230 0.0011830 0.0011440 0.0011070 0.0010700 0.0010350 0.0010010
-2.90 0.0018660 0.0018070 0.0017500 0.0016950 0.0016410 0.0015890 0.0015380 0.0014890 0.0014410 0.0013950
-2.80 0.0025550 0.0024770 0.0024010 0.0023270 0.0022560 0.0021860 0.0021180 0.0020520 0.0019880 0.0019260
-2.70 0.0034670 0.0033640 0.0032640 0.0031670 0.0030720 0.0029800 0.0028900 0.0028030 0.0027180 0.0026350
-2.60 0.0046610 0.0045270 0.0043960 0.0042690 0.0041450 0.0040250 0.0039070 0.0037930 0.0036810 0.0035730
-2.50 0.0062100 0.0060370 0.0058680 0.0057030 0.0055430 0.0053860 0.0052340 0.0050850 0.0049400 0.0047990
-2.40 0.0081980 0.0079760 0.0077600 0.0075490 0.0073440 0.0071430 0.0069470 0.0067560 0.0065690 0.0063870
-2.30 0.0107200 0.0104400 0.0101700 0.0099030 0.0096420 0.0093870 0.0091370 0.0088940 0.0086560 0.0084240
-2.20 0.0139000 0.0135500 0.0132100 0.0128700 0.0125500 0.0122200 0.0119100 0.0116000 0.0113000 0.0110100
-2.10 0.0178600 0.0174300 0.0170000 0.0165900 0.0161800 0.0157800 0.0153900 0.0150000 0.0146300 0.0142600
-2.00 0.0227500 0.0222200 0.0216900 0.0211800 0.0206800 0.0201800 0.0197000 0.0192300 0.0187600 0.0183100
-1.90 0.0287200 0.0280700 0.0274300 0.0268000 0.0261900 0.0255900 0.0250000 0.0244200 0.0238500 0.0233000
-1.80 0.0359300 0.0351500 0.0343800 0.0336200 0.0328800 0.0321600 0.0314400 0.0307400 0.0300500 0.0293800
-1.70 0.0445700 0.0436300 0.0427200 0.0418200 0.0409300 0.0400600 0.0392000 0.0383600 0.0375400 0.0367300
-1.60 0.0548000 0.0537000 0.0526200 0.0515500 0.0505000 0.0494700 0.0484600 0.0474600 0.0464800 0.0455100
-1.50 0.0668100 0.0655200 0.0642600 0.0630100 0.0617800 0.0605700 0.0593800 0.0582100 0.0570500 0.0559200
-1.40 0.0807600 0.0792700 0.0778000 0.0763600 0.0749300 0.0735300 0.0721500 0.0707800 0.0694400 0.0681100
-1.30 0.0968000 0.0951000 0.0934200 0.0917600 0.0901200 0.0885100 0.0869100 0.0853400 0.0837900 0.0822600
-1.20 0.1151000 0.1131000 0.1112000 0.1093000 0.1075000 0.1056000 0.1038000 0.1020000 0.1003000 0.0985300
-1.10 0.1357000 0.1335000 0.1314000 0.1292000 0.1271000 0.1251000 0.1230000 0.1210000 0.1190000 0.1170000
-1.00 0.1587000 0.1562000 0.1539000 0.1515000 0.1492000 0.1469000 0.1446000 0.1423000 0.1401000 0.1379000
-0.90 0.1841000 0.1814000 0.1788000 0.1762000 0.1736000 0.1711000 0.1685000 0.1660000 0.1635000 0.1611000
-0.80 0.2119000 0.2090000 0.2061000 0.2033000 0.2005000 0.1977000 0.1949000 0.1922000 0.1894000 0.1867000
-0.70 0.2420000 0.2389000 0.2358000 0.2327000 0.2297000 0.2266000 0.2236000 0.2206000 0.2177000 0.2148000
-0.60 0.2743000 0.2709000 0.2676000 0.2643000 0.2611000 0.2578000 0.2546000 0.2514000 0.2483000 0.2451000
-0.50 0.3085000 0.3050000 0.3015000 0.2981000 0.2946000 0.2912000 0.2877000 0.2843000 0.2810000 0.2776000
-0.40 0.3446000 0.3409000 0.3372000 0.3336000 0.3300000 0.3264000 0.3228000 0.3192000 0.3156000 0.3121000
-0.30 0.3821000 0.3783000 0.3745000 0.3707000 0.3669000 0.3632000 0.3594000 0.3557000 0.3520000 0.3483000
-0.20 0.4207000 0.4168000 0.4129000 0.4090000 0.4052000 0.4013000 0.3974000 0.3936000 0.3897000 0.3859000
-0.10 0.4602000 0.4562000 0.4522000 0.4483000 0.4443000 0.4404000 0.4364000 0.4325000 0.4286000 0.4247000
0.00 0.5000000 0.4960000 0.4920000 0.4880000 0.4840000 0.4801000 0.4761000 0.4721000 0.4681000 0.4641000

Safety, Risk and Reliability, Heriot-Watt University 258


Appendix 2 – Numerical Values of the Standard Normal CDF
u 0.00 0.01 0.02 0.03 0.04 0.05 0.06 0.07 0.08 0.09
0.00 0.5000000 0.5040000 0.5080000 0.5120000 0.5160000 0.5199000 0.5239000 0.5279000 0.5319000 0.5359000
0.10 0.5398000 0.5438000 0.5478000 0.5517000 0.5557000 0.5596000 0.5636000 0.5675000 0.5714000 0.5753000
0.20 0.5793000 0.5832000 0.5871000 0.5910000 0.5948000 0.5987000 0.6026000 0.6064000 0.6103000 0.6141000
0.30 0.6179000 0.6217000 0.6255000 0.6293000 0.6331000 0.6368000 0.6406000 0.6443000 0.6480000 0.6517000
0.40 0.6554000 0.6591000 0.6628000 0.6664000 0.6700000 0.6736000 0.6772000 0.6808000 0.6844000 0.6879000
0.50 0.6915000 0.6950000 0.6985000 0.7019000 0.7054000 0.7088000 0.7123000 0.7157000 0.7190000 0.7224000
0.60 0.7257000 0.7291000 0.7324000 0.7357000 0.7389000 0.7422000 0.7454000 0.7486000 0.7517000 0.7549000
0.70 0.7580000 0.7611000 0.7642000 0.7673000 0.7703000 0.7734000 0.7764000 0.7794000 0.7823000 0.7852000
0.80 0.7881000 0.7910000 0.7939000 0.7967000 0.7995000 0.8023000 0.8051000 0.8078000 0.8106000 0.8133000
0.90 0.8159000 0.8186000 0.8212000 0.8238000 0.8264000 0.8289000 0.8315000 0.8340000 0.8365000 0.8389000
1.00 0.8413000 0.8438000 0.8461000 0.8485000 0.8508000 0.8531000 0.8554000 0.8577000 0.8599000 0.8621000
1.10 0.8643000 0.8665000 0.8686000 0.8708000 0.8729000 0.8749000 0.8770000 0.8790000 0.8810000 0.8830000
1.20 0.8849000 0.8869000 0.8888000 0.8907000 0.8925000 0.8944000 0.8962000 0.8980000 0.8997000 0.9014700
1.30 0.9032000 0.9049000 0.9065800 0.9082400 0.9098800 0.9114900 0.9130900 0.9146600 0.9162100 0.9177400
1.40 0.9192400 0.9207300 0.9222000 0.9236400 0.9250700 0.9264700 0.9278500 0.9292200 0.9305600 0.9318900
1.50 0.9331900 0.9344800 0.9357400 0.9369900 0.9382200 0.9394300 0.9406200 0.9417900 0.9429500 0.9440800
1.60 0.9452000 0.9463000 0.9473800 0.9484500 0.9495000 0.9505300 0.9515400 0.9525400 0.9535200 0.9544900
1.70 0.9554300 0.9563700 0.9572800 0.9581800 0.9590000 0.9599400 0.9608000 0.9616400 0.9624600 0.9632700
1.80 0.9640700 0.9648500 0.9656200 0.9663800 0.9671200 0.9678400 0.9685600 0.9692600 0.9699500 0.9706200
1.90 0.9712800 0.9719300 0.9725700 0.9732000 0.9738100 0.9744100 0.9750000 0.9755800 0.9761500 0.9767000
2.00 0.9772500 0.9777800 0.9783100 0.9788200 0.9793200 0.9798200 0.9803000 0.9807700 0.9812400 0.9816900
2.10 0.9821400 0.9825700 0.9830000 0.9834100 0.9838200 0.9842200 0.9846100 0.9850000 0.9853700 0.9857400
2.20 0.9861000 0.9864500 0.9867900 0.9871300 0.9874500 0.9877800 0.9880900 0.9884000 0.9887000 0.9889900
2.30 0.9892800 0.9895600 0.9898300 0.9900970 0.9903580 0.9906130 0.9908630 0.9911060 0.9913440 0.9915760
2.40 0.9918020 0.9920240 0.9922400 0.9924510 0.9926560 0.9928570 0.9930530 0.9932440 0.9934310 0.9936130
2.50 0.9937900 0.9939630 0.9941320 0.9942970 0.9944570 0.9946140 0.9947660 0.9949150 0.9950600 0.9952010
2.60 0.9953390 0.9954730 0.9956040 0.9957310 0.9958550 0.9959750 0.9960930 0.9962070 0.9963190 0.9964270
2.70 0.9965330 0.9966360 0.9967360 0.9968330 0.9969280 0.9970200 0.9971100 0.9971970 0.9972820 0.9973650
2.80 0.9974450 0.9975230 0.9975990 0.9976730 0.9977440 0.9978140 0.9978820 0.9979480 0.9980120 0.9980740
2.90 0.9981340 0.9981930 0.9982500 0.9983050 0.9983590 0.9984110 0.9984620 0.9985110 0.9985590 0.9986050
3.00 0.9986500 0.9986940 0.9987360 0.9987770 0.9988170 0.9988560 0.9988930 0.9989300 0.9989650 0.9989990
3.10 0.9990324 0.9990646 0.9990957 0.9991260 0.9991553 0.9991836 0.9992112 0.9992378 0.9992636 0.9992886
3.20 0.9993129 0.9993363 0.9993590 0.9993810 0.9994024 0.9994230 0.9994429 0.9994623 0.9994810 0.9994991
3.30 0.9995166 0.9995335 0.9995499 0.9995658 0.9995811 0.9995959 0.9996103 0.9996242 0.9996376 0.9996505
3.40 0.9996631 0.9996752 0.9996869 0.9996982 0.9997091 0.9997197 0.9997299 0.9997398 0.9997493 0.9997585
3.50 0.9997674 0.9997759 0.9997842 0.9997922 0.9997999 0.9998074 0.9998146 0.9998215 0.9998282 0.9998347
3.60 0.9998409 0.9998469 0.9998527 0.9998583 0.9998637 0.9998689 0.9998739 0.9998787 0.9998834 0.9998879
3.70 0.9998922 0.9998964 0.9999004 0.9999043 0.9999080 0.9999116 0.9999150 0.9999184 0.9999216 0.9999247
3.80 0.9999277 0.9999305 0.9999333 0.9999359 0.9999385 0.9999409 0.9999433 0.9999456 0.9999478 0.9999499
3.90 0.9999519 0.9999539 0.9999557 0.9999575 0.9999593 0.9999609 0.9999625 0.9999641 0.9999655 0.9999670
4.00 0.9999683 0.9999696 0.9999709 0.9999721 0.9999733 0.9999744 0.9999755 0.9999765 0.9999775 0.9999784
4.10 0.9999793 0.9999802 0.9999811 0.9999819 0.9999826 0.9999834 0.9999841 0.9999848 0.9999854 0.9999861
4.20 0.9999867 0.9999872 0.9999878 0.9999883 0.9999888 0.9999893 0.9999898 0.9999902 0.9999907 0.9999911
4.30 0.9999915 0.9999918 0.9999922 0.9999925 0.9999929 0.9999932 0.9999935 0.9999938 0.9999941 0.9999943
4.40 0.9999946 0.9999948 0.9999951 0.9999953 0.9999955 0.9999957 0.9999959 0.9999961 0.9999963 0.9999964
4.50 0.9999966 0.9999968 0.9999969 0.9999871 0.9999972 0.9999973 0.9999974 0.9999976 0.9999977 0.9999978
4.60 0.9999979 0.9999980 0.9999981 0.9999982 0.9999983 0.9999983 0.9999984 0.9999985 0.9999986 0.9999986
4.70 0.9999987 0.9999988 0.9999988 0.9999989 0.9999989 0.9999990 0.9999990 0.9999991 0.9999991 0.9999992
4.80 0.9999992 0.9999992 0.9999993 0.9999993 0.9999994 0.9999994 0.9999994 0.9999994 0.9999995 0.9999995
4.90 0.9999995 0.9999995 0.9999996 0.9999996 0.9999996 0.9999996 0.9999996 0.9999997 0.9999997 0.9999997

Safety, Risk and Reliability, Heriot-Watt University 259


Appendix 2 – Numerical Values of the Standard Normal CDF
APPENDIX 3 – Answers to exercises from Appendix 1
Answers to questions in Appendix 1

Exercise A1.1

The maximum principal stress is the hoop stress in the cylindrical section. The axial or meridinal
stress is 50% of this, as are the stresses in the end caps.

Exercise A1.2

The hoop stress in a pipe increases linearly with the diameter, so it is best to use as small a diameter
as practical. If the diameter doubles then so does the hoop stress.

Exercise A1.3

The crack will run along the length of the pipe as the maximum stress that tends to pull the pipe
apart is at right angles to this direction. A crack once ran for 7 miles along an underground plastic
pipe in the USA.

Exercise A1.4

If the depth of a beam is doubled I will increase by a factor of 8 (= 2 3) and ymax by a factor of 2. So for
a given applied moment M, the stress will be divided by 4, i.e. reduced to 25% of the original value.

Exercise A1.5

The stress is zero at the neutral axis, which for elastic bending passed through the centroid of the
beam. For a symmetrical section this will be at the mid-depth but for asymmetric sections, such as a
‘T’ bar, it won’t be.

Exercise A1.6

For a cantilever with an end load the deflection will increase by 50% if the load does.

Exercise A1.7

Elastic failure occurs when any points in a structure reach the yield stress. Plastic failure occurs when
there are sufficient plastic hinges or areas of yielding for a collapse mechanism to occur.

Exercise A1.8

Elastic section modulus is the second moment of area divided by ymax. Plastic section modulus is half
the cross-sectional area multiplied by the distance between the centroids of the top half of the
section and the bottom half of the section. The plastic section modulus is always larger than the
elastic section modulus.

Exercise A1.9

If the 2 m cantilever is deflecting 0.3 m at its free end then the rotation at the fixed end is
(approximately) 0.3/2 radians. Multiplying this angle by them moment of resistance (Mp = Sy) will
give the interval work done.

Safety, Risk and Reliability, Heriot-Watt University 260


Appendix 3 – Answers to Exercises
Exercise A1.10

Plastic hinges
Exercise A1.11

The second moment of area for a tube is the same about all axes from the centre. Thus it is equally
resistant buckling in all directions. However, a rectangular section has a lower 2 nd moment of area
about an axis running across the wider direction of the section.

Low I value
about this axis

Buckles in
this direction

Exercise A1.12

The buckling constant for an unsupported mast is 0.25. For a mast supported by stays the constant is
2.05. So the buckling load is 8.2 times higher. The buckling strength is proportional to

D 4
I ,
64
4
Doriginal
so D 4
new  , i.e. Dnew = 1.69Doriginal
8.2
And so it would be reduced by 41%

Exercise A1.13

A short stocky column will crush under excessive load. This will cause crack and fracture in a brittle
material or gross plastic distortion in a ductile one.

Exercise A1.14

A mast supporting a platform subject to wind load.

RQ A1.15. a)

extension 0.0227
   9.08  10 4
length 25

Safety, Risk and Reliability, Heriot-Watt University 261


Appendix 3 – Answers to Exercises
load 5  10 3
   63.66 Nmm-2
c.s.a 5 2

 63.66
E   7.01 10 4 Nmm-2 = 70 GNm-2
 9.08  10 4

20  10 3
 yield   254.6 Nmm-2
5 2

37.5  10 3
 ult   477.5 Nmm-2
5 2

b)

displacement = x + 175 mm
 F
x  l ,   
E AE

Fl 1.5  10 3  750  10 3
x  = 818.5 mm
AE 7  10 4    2.5 2

Therefore the displacement = 818.5 + 175 = 993.5 mm


For a displacement of 250mm, x = 250 – 175 = 75 mm
Fl 4 Fl
75  
AE D 2 E

4  1.5  10 3  750  10 3
So D = 16.5 mm
  7  10 4  75

RQ A1.16.

Mean diameter = 30 + 1.25 = 31.25 cm


So R = 15.625 cm
pR
Max stress = hoop stress =
t
Max stress with safety factor of 1.6
Y 210
 max    131 MNm-2
1.6 1.6

131 0.0125
So p  10 MNm-2
0.15625

Safety, Risk and Reliability, Heriot-Watt University 262


Appendix 3 – Answers to Exercises
RQ A1.17.

For a factor of safety of 1.4:


 yield 300
 working    214 Nmm-2
1.4 1.4

For a factor of safety of 1.4:


pR
Largest stress is the hoop stress  H 
t
p  40
So 214  , therefore p = 5.36 Nmm-2
1
pR
Maximum stress in end caps  
2t
5.36  40
214  tends  0.5 mm
2t

Therefore the end caps can be reduced to 0.5mm thickness


To tear cylindrical vessel apart
   ult  400 MNm-2 (Nmm-2)

p  40
400  , p = 10 Nmm-2
1

This is the pressure to cause rupture.


Due to plastic stretching the radius will increase and as the pressure is inversely proportional to the
radius then failure may occur at a slightly lower pressure.

RQ A1.18.


1m

W?
l

1m

Initial work done = 2Mp + Mp2


S 2S S
= 2Mp  Mp  8Mp
l/2 l/2 l
Extensional work done = WS

Safety, Risk and Reliability, Heriot-Watt University 263


Appendix 3 – Answers to Exercises
S 8Mp 8 Y S
WS  8Mp , so W  
l l l

4 3 A r 2 4r 4r 3
Now S  r - Note that S  c1c2  .2. 
3 2 2 3 3
3
4  30 
S max   
3 2 

3
4  30 
Mpmax     300  1.35  10 6 Nmm
3 2 

3 3
4  25  4  25 
Similarly S min    and Mpmin     260  677  10 3 Nmm
3 2  3 2 

8  1.35  10 6
Therefore Wmax   5.4 kN
2000
8  677  10 3
and Wmin   2.708 kN
2000

RQ A1.19.

1.2

0.04 0.04


 

  0.642  0.62  0.223

 0.223
sin  ,   0.712 radians (/2 = 20.39)
2 0.64

0.3  0.006 2
Mp  S Y   250  10 6  675 Nm
4

At collapse of plate Work Done = 6750.712 = 480.6 J


1 2
Now Kinetic Energy = Work Done = mv
2

Safety, Risk and Reliability, Heriot-Watt University 264


Appendix 3 – Answers to Exercises
and v 2  u 2  2as for constant acceleration
v 2  0  2  9.8110  196.2 , so v = 14 ms-1

2  480.6
Therefore m  4.90 kg
196.2

RQ A1.20.

 2 EI
PCr 
4l 2

I  r 3t    0.027 3  0.005  3.09 x10 7

 2  70  10 9  3.09  10 7
PCr   721.6
4  8.6 2 , N = 73.55 kg

Therefore assuming the student’s weight has a uniform distribution in interval between 70 and 75
kg, then the probability that his weight is less than PCr is given by
3.55
Probability of success =  0.71
5
Assuming student weighs 75 kg, the diameter of solid timber pole 6.6m high for same PCr
In this case for a safety factor of 1.5

 2 E D 4  3 ED 4
PCr  (75  9.81) 1.5  
4l 2 64 256l 2
1
 256l 2  PCr  1.5  4
D   
  3
E 
1
 256l 2  75  9.81  1.5  4
  
  3
E 

If E = 12109 Nm-2 and l = 6.6 m


1
 256  6.6 2  1103.6  4
D   
   12  10
3 9

 0.0758 m

Safety, Risk and Reliability, Heriot-Watt University 265


Appendix 3 – Answers to Exercises
APPENDIX 4 – INFORMATION SHEET

This information will be available in the examination for this module.

Basic Mechanics of Materials

k 2 EI
Euler column buckling PE 
2

where k varies with end conditions


-1.0 simple supports;
4.0 ends fixed;
0.25 one fixed, one free;
2.05 one fixed, one simple

Pr
Hoop stress 
t

Stress intensity KI   a

 C K m
da
Paris - Erdogan equation
dN

Mgl 3
y - end deflection of light cantilever with concentrated weight Mg at end
3EI

y

mg 4l 3 x  6l 2 x 2  x 4 
- deflection at point x of cantilever under distributed load mg
24 EI

Mga 2 l  a 2
y - deflection of a simply supported beam at a with concentrated Mg at a
3lEI

y

mg l 3 x  2l 2 x 2  x 4 
- deflection at point x of simply supported beam under distributed
24 EI
load mg

Geometrical Formulae

4r
Centroid semi-circular section of a semi circle =
3

r 4
For a circular section I
4

Safety, Risk and Reliability, Heriot-Watt University 266


Appendix 4 – Information Sheet
bd 3
For rectangular section I 
12

For thin walled tubular section I  r 3t

Probability Density Functions

Exponential f ( x)   exp  x  F ( x)  1 exp  x  where     1


 1  x   2 
Normal f  x  
1
exp    
 2  2    

 1  nx    2 
f x  
1
Lognormal exp    
x 2  2    


where   exp   1 / 2 2  and   exp 2  2 2    2


Weibull f x   x  1 exp  x   
F x   1  exp  x  
- maximum Fn  x   Fx  x 
n
Extreme value distributions

- minimum F1 x   1  1  Fx x 
n

Maximum Extreme Value Type 1

F x   exp  exp   x  u  and f x    exp   x  u exp  exp   x  u 

1n1nN  1n4 
where   21nN and u   
2

 0.5772 
for an underlying Normal distribution  xn   x   x  u  
  

2 2
and  xn  x
2
6 2
Minimum Extreme Value Type I

F x   1  exp  exp  x  u  and f x    exp  x  u exp  exp  x  u 

1n1nN  1n4 
where   21nN and u   
2

 0.5772   2
for an underlying normal distribution  x1   x   x  u   and  x1  2  x
2 2

   6

Safety, Risk and Reliability, Heriot-Watt University 267


Appendix 4 – Information Sheet
f a g  x 
Bayes' rule f a x  
 f a g  x da

Reliability and Simulation

 f x     2 f x  
f x  h   f x    hi 
1
  2!  hi h j  x x   ........
   
  xi x  i j x

G  
FOSM -  
 G  x   2
2

 
i 1, N xi  
  i

2
 G  x  
FORM -     
 xi 

 G  x  
 i    / 
 xi 

    Gx  /  
m1
x

  x
2
i

 ax 
SIMULATION - xi 1  xi  mINT  i 
 m 

s1   21nu1  2 cos 2u 2


1

s2   21nu1  2 sin 2u 2


1

Safety, Risk and Reliability, Heriot-Watt University 268


Appendix 4 – Information Sheet
APPENDIX 5 – PAST EXAM PAPERS

SCHOOL OF THE BUILT ENVIRONMENT

D21SR Safety, Risk and Reliability

Semester 2 – 2008/09

Date: Time: 2 hours

Answer all questions

269
Q1

(a) There is a concern that reinforcing steel in a concrete beam is subject to corrosion. The
severity of corrosion depends on the corrosion rate expressed in terms of the corrosion
current density, icorr, and classified as follows:

Low: icorr < 1 μA/cm2

Medium: 1 μA/cm2 ≤ icorr ≤ 3 μA/cm2

High: icorr > 3 μA/cm2

Based on the experience with similar beams in similar conditions and visual observations the
following probabilities that corrosion in the beam belongs to one of the above three classes
have been assigned: low – 0.35, medium – 0.60, and high – 0.05.

In order to obtain more information about the corrosion rate it has been decided to perform
corrosion measurements on-site. However, the device used for this is not very accurate and
when it shows that the corrosion rate belongs to a certain class there is a probability that it
can belong to another class. These probabilities are shown in the table below.

Corrosion Prior probabilities Likelihoods, P(M|Class)

rate M=Low M=Medium M=High

Low 0.35 0.80 0.19 0.01

Medium 0.60 0.20 0.70 0.10

High 0.05 0.02 0.23 0.75

The corrosion rate in the beam has been measured and the result was icorr=4 μA/cm2. Using
this new information, update the probabilities of the corrosion rate belonging to one of the
three classes.

(13 marks)

(b) The reinforced concrete beam from part (a) was initially designed to support the bending
moment with a mean of 27 kNm and a standard deviation of 4.5 kNm. The target safety index
used in the design was 3.8. It is also known that the standard deviation of the beam bending
strength used in the design was 5 kNm. Based on results of an on-site inspection it has been
estimated that due to corrosion the bending strength of the beam has decreased by 15%.

270
What is the safety index for the corrosion-damaged beam with regard to its failure in bending?
Assume that the standard deviation of the beam bending strength has remained unchanged.

(12 marks)

Q2 A steel pipeline is designed for a pressure, P, with a mean of 5 MN/m2 and a coefficient of
variation of 0.15. It is considered in the design that the pipeline fails due to yielding of steel
when hoop stress caused by pressure exceeds the yield stress of steel σy. The hoop stress can
be evaluated as

Pr

t

where r is the pipe radius and t is the pipe wall thickness. The yield stress of steel has a mean
value of 360 MN/m2 and a standard deviation of 30 MN/m2. The coefficient of variation of the
wall thickness t is 0.05. The pipe radius r can be treated as a deterministic parameter.
Determine the required mean pipe wall thickness for the pipeline with radius of 0.4 m if the
target safety index is 3.5. Use the FOSM method assuming that the pressure P, the yield stress
σy, and the wall thickness t are independent normal random variables.

(25 marks)

Q3 A simply supported steel beam with span L=8 m is loaded by uniformly distributed load w. The
yield stress of the steel, fy, is lognormally distributed with a mean of 375 N/mm2 and a
coefficient of variation (COV) of 0.07. The bending resistance of the beam is Zfy, where Z – the
plastic section modulus, is a normal random variable with a mean of 2×10-4 m3 and a COV of
0.04. The load intensity w has a Gumbel distribution with a mean of 5 kN/m and a COV of 0.20.
The probability of failure is calculated by Monte Carlo simulation. Perform one simulation
cycle using the following random numbers: u1=0.9003, u2=0.0705 and u3=0.6276.

(20 marks)

Q4

(a) An anchor chain of length 150 m is intended for use in a catenary mooring system. The chain
contains 450 links. Links identical to those in the chain have been tested and based on the test
results it has been determined that the strength of a single link can be modelled by a
lognormal random variable with a mean of 8 kN and a standard deviation of 0.8 kN. Before the

271
use the chain is proof tested with a load of 5.5 kN. What is the probability that the chain fails
the proof test?

(15 marks)

(b) The anchor chain is deployed on a drilling rig, which is installed in the open sea. The load in the
anchor chain is due to passage of waves and can be determined as C×H, where H is the weight
height and C is a force coefficient. It is expected that the worst 3-hour storm during the service
life of the drilling rig will have a significant wave height of 11 m and mean wave period of 12
seconds. Assume that the wave heights can be modelled by a Rayleigh distribution and C and
the chain strength R can be treated as deterministic parameters: C=0.25 kN/m and R=6 kN.
What is the probability of failure of the anchor chain during the extreme storm?

(15 marks)

End of Paper

272
SCHOOL OF THE BUILT ENVIRONMENT

D21SR Safety, Risk and Reliability

August – 2008/09

Date: Time: 2 hours

Answer all questions

273
Q1

(a) There is a set of timber members. It is known that 35% of the members have the tensile

strength above 30 N/mm2. 4/5 members out of these 35% have the modulus of elasticity

higher than 14 kN/mm2. From the other 65% of the members (with the tensile strength 30
N/mm2 and lower) only 1/3 have the modulus of elasticity higher than 14 kN/mm 2. If a timber
member has the modulus of elasticity higher than 14 kN/mm 2, what is the probability that the
tensile strength of this member is above 30 kN/mm 2? If a timber member has the modulus of
elasticity lower than 14 kN/mm2, what is the probability that the tensile strength of this
member is above 30 kN/mm2?

(12 marks)

(b) The timber members are used to build a nine-member statically determinate truss, i.e., failure
of any one of its members constitutes failure of the whole truss. The annual probability of
failure of each of the truss members is 5×10 -4. What is the annual probability of failure of the
truss? What is the probability of failure of the truss in 50 years? What is the probability of
more than one failure of the truss in 50 years? Assume that failures of the truss members are
mutually independent events.

(13 marks)

Q2 A pin-ended slender column of height L=5 m is to be designed to support without buckling a


load of W with a mean of 80 kN and a coefficient of variation (COV) of 0.15. The column is
made of structural timber with the modulus of elasticity E, its mean is 10 kN/mm2 and
COV=0.10. The column has a square cross-section a×a, where a has a mean of 200 mm and a
standard deviation of 4 mm. Find the safety index and the corresponding probability of failure

using the FOSM method. Assume that the load W, the modulus of elasticity E and the side
dimension a are independent normal random variables. Critical buckling load is Pcr=π2EI/L2 and
for a square cross-section I=a4/12.

(25 marks)

274
Q3 An anchor chain of length 120 m is used in a catenary mooring system of a drilling rig installed
in the open sea. The chain contains 400 links and the strength of a single link can be modelled
by a normal random variable with a mean of 10 kN and a standard deviation of 1 kN. The load
in the anchor chain is due to passage of waves and can be determined as C×H, where H is the
weight height and C is a force coefficient, which is normally distributed with a mean of 0.30
kN/m and a coefficient of variation 0.20. It is expected that the worst storm during the service
life of the drilling rig will have a significant wave height of 12 m and there will be 900 waves
during 3-hour duration of this storm. The wave heights can be modelled by a Rayleigh
distribution. The probability of failure of the anchor chain during the worst storm is calculated
by Monte Carlo simulation. Perform one simulation cycle using the following random
numbers: u1=0.4753, u2=0.8741, u3=0.3046 and u4=0.5234.

(20 marks)

Q4

(a) An old pipeline is quite badly corroded over its entire surface. A detailed inspection reveals no
cracks or deep pits rather just a general loss of the wall thickness that varies randomly over
the length of the pipeline. In order to model the effect of corrosion the pipeline has been
divided into 3400 segments along its length. Of these 60 segments have been selected
randomly and their wall thickness has been measured giving a mean thickness of 8 mm with a
standard deviation of 0.3 mm. The sample data when plotted have showed that the variability
of the wall thickness of a one segment can be described by a normal distribution. Find the
corresponding extreme value distribution of the minimum wall thickness of the 3400
segments. What is the probability that the wall thickness of the most corroded segment of the
pipeline is less than 6 mm?

(15 marks)

(b) It is expected that over its remaining service life the pipeline from part (a) will experience 8000
pressure cycles. The pressure in one cycle is modelled by a normal random variable with a
mean of 6 MN/m2 and a standard deviation of 1 MN/m2. The pipeline can fail due to yielding
of steel when hoop stress caused by pressure exceeds the yield stress of steel σy. The hoop
stress can be evaluated as

Pr

t

275
where r is the pipe radius and t is the pipe wall thickness. Calculate the probability of failure of
the pipeline due to steel yielding if its radius r=0.2 m, the yield stress of steel σy=320 MN/m2
and the pipe wall thickness is also treated as a deterministic parameter equal to 6.5 mm.

(15 marks)

End of Paper

276

Potrebbero piacerti anche